You are on page 1of 107

Options for Questions 1-2

A Prolactinoma B Pre-mature ovarian failure


C Turners syndrome D Ovarian hyper-stimulation syndrome
E Polycystic ovary syndrome F Hypothyroidism
G Cushings syndrome H Congenital adrenal hyperplasia
I Hypothalamic amenorrhoea J Hyperthyroidism
K Androgen insensitivity syndrome L Pure gonadal dysgenesis

Instructions: For each of the case histories described below, choose the single most likely cause of
menstrual abnormalities from the above list. Each option may be used once, more than once, or not at all.

A 35 year old woman and her 40 year old partner have been
referred to the fertility clinic because of a failure to conceive after
3 years of unprotected intercourse. The semen analysis is
normal. The woman has irregular periods every 3-6 months but Your answer: A
Question 1 no other symptoms and her BMI is 27. The womans results are
as follows: FSH (day 3) = 20mIU/ml, LH (day 3) = 8mIU/ml, Correct answer: B
prolactin = 110 ng/ml, progesterone (day 21) = 1.5ng/ml,
testosterone = 2.0pg/ml, DHEA, DHEA-sulphate and thyroid
function tests were normal.

A 35 year old woman with one previous normal pregnancy 3


years earlier attends the gynaecology clinic because of irregular
periods every 4-6 months. On direct questioning, she complains Your answer: A
Question 2 of persistent headaches in the last 9 months which are worse
first thing in the morning and a milky discharge from her right Correct answer: A
breast. Her BMI is 27 and there is a bi-temporal hemi-anopia on
clinical examination.

| EXPLANATION |

Options for Questions 3-4

A Routine recall in 3 years B Routine recall in 5 years


C Hysteroscopy + D&C D Laser vapourisation
E Refer for colposcopy F Cold knife cone biopsy
G Repeat cervical smear as soon as possible H Cryotherapy
I No further cervical smears required J Repeat cervical smear in 6 months

Instructions: For each scenario described below, choose the single most appropriate management from the
above list of options. Each option may be used once, more than once, or not at all.

A 30 year old woman with previously normal cervical smears has Your answer: A
Question 3 a smear which is reported as inadequate because the specimen
was poorly fixed. Correct answer: G

A 35 year old woman had a cervical smear 3 years ago which Your answer: A
showed border-line nuclear abnormalities. She has since had two
Question 4
consecutive smears 12 months apart which have been reported
as negative Correct answer: A

| EXPLANATION |

Options for Questions 5-6


A Continue iv oxytocin B Deliver by caesarean section
C Evacuation of retained products of conception D No additional intervention at this stage
E Bimanual compression of the uterus F Venous access and resuscitation
G Examination under spinal anaesthesia H Lovsets manoeuvre
I Intravenous ritodrine J Manual replacement of the uterus
K Sub-cutaneous terbutalline L Administer uterotonic agent

Instructions: For each of the case histories described below, choose the single most appropriate initial
management from the above list. Each option may be used once, more than once, or not at all.

A 16 year old primigravida attends for induction of labour


following intra-uterine fetal death at 36 weeks gestation. Maternal
BP is 120/80 with no proteinuria and all blood tests are normal. Your answer: A
Question 5 The fetus is in a breech presentation. Prostaglandin is
administered and oxytocin commenced at 12:00 according to the Correct answer: A
unit protocol. At 00:00, the cervix remains 1cm long and the os is
closed. There are 3 uterine contractions every 10 minutes.

A 34 year old woman is bleeding heavily and collapses during Your answer: A
the third stage of labour. Her pulse is 40bpm and systolic BP is
Question 6
70mmHg. There is a mass at the introitus consistent with uterine
inversion Correct answer: F

| EXPLANATION |

Options for Questions 7-8

A Failure to relieve symptoms B Damage to bowel


C Removal of ovaries D Irregular bleeding for 3-4 months
E Failure to identify disease F Failure to visualise uterine cavity
G Shoulder-tip pain H Haemorrhage requiring return to theatre
I Laparotomy J Failure rate 1 in 200

Instructions: For each of the case histories described below, choose the single most relevant complication
that you must discuss with the patient when taking consent prior to surgery from the above list of options.
Each option may be used once, more than once, or not at all.

A 35 year old nulliparous woman with menorrhagia is having the Your answer: A
Question 7 levonorgestrel releasing intra-uterine system inserted under
general anaesthesia Correct answer: D

Your answer: A
A 40 year old woman with a previous failed TVT for genuine
Question 8
stress incontinence is having a Burch colposuspension
Correct answer: A

| EXPLANATION |

Options for Questions 9-10

A Stress incontinence B Urinary retention with overflow


C Urinary retention D Detrusor instability
E Detrusor hyper-reflexia F Interstitial cystitis
G Genuine stress incontinence H Mixed urinary incontinence
I Bladder tumour J Urinary tract infection
K Diabetes mellitus

Instructions: For each of the case histories described below, choose the single most likely diagnosis from
the above list of options. Each option may be used once, more than once, or not at all.

A 35 year old woman complains of urinary frequency, urgency,


urge incontinence and bladder pain. Urine dipstix showed Your answer: A
Question 9 haematruria. Her symptoms have not responded to conservative
and medical treatment. Cystoscopy showed petechial Correct answer: F
haemorrhages in the bladder mucosa

A 40 year old woman has been investigated extensively for


persistent microscopic haematuria which was initially detected Your answer: A
Question 10 during pregnancy. Over the last year, she has developed urinary
urgency with bladder pain. Bladder biopsy showed a chronic Correct answer: F
inflammatory infiltrate in the detrusor muscle

| EXPLANATION |

Options for Questions 11-12

A Counsel and offer evacuation of retained products of B Counsel and offer support group
conception
C Counsel and offer laparoscopy & salpingectomy D Counsel and offer investigation for recurrent
miscarriage
E Counsel and offer treatment with methotrexate F Counsel and offer laparotomy & salpingectomy
G Counsel and offer termination of pregnancy H Perform salpingectomy
I Proceed to laparotomy J Serial HCG assay

K Counsel and refer to early pregnancy assessment L Repeat trans-vaginal scan 7 days later
unit

Instructions: For each patient described below choose the single most appropriate initial management
option from the list. Each option may be used once, more than once, or not at all.

A 25 year old woman had a transvaginal scan following IVF


treatment during which 2 embryos were replaced and was found Your answer: A
to have a 6 week singleton intra-uterine pregnancy. She presents
Question 11
2 weeks later with abdominal pain and vaginal bleeding and is
found to have an empty uterus with a small amount of free fluid in Correct answer: B
the pouch of Douglas.

A 25 year old woman had a transvaginal scan following IVF


treatment during which 2 embryos were replaced and is found to Your answer: A
Question 12 have a 6 week singleton intra-uterine pregnancy with a 5cm
cystic structure adjacent to the right ovary with internal echoes Correct answer: C
consistent with an ectopic pregnancy.

| EXPLANATION |

Options for Questions 13-14

A Stop heparin on the evening before induction B Commence oral warfarin

C Await results of V/Q scan then commence D Await results of venogram then commence
treatment treatment
E Await results of D-dimers assay F Commence therapeutic dose of heparin

G Commence prophylactic dose of heparin post- H Oral aspirin therapy


partum
I Antenatal prophylactic treatment with heparin J Antenatal treatment with warfarin
K Prophylactic heparin for 6 weeks post-partum L Stop heparin therapy

Instructions: For each scenario described below, choose the single most appropriate management from the
above list of options. Each option may be used once, more than once, or not at all.

A 30 year old woman with a previous DVT presents for antenatal Your answer: A
care at 15 weeks gestation. There is no family history of VTE and
Question 13
her BMI is 25. She is otherwise fit and well with a negative
thrombophilia screen Correct answer: K

A 30 year old woman with recurrent first trimester miscarriages is


known to have the anti-phospholipid antibody syndrome and has Your answer: A
Question 14 had a spontaneous vaginal delivery at 39 weeks gestation. Her
BMI is 27 and she is otherwise fit and well with no family history Correct answer: G
of VTE.

| EXPLANATION |

Options for Questions 15-16

A 85% 5 year survival B 70% 5 year survival


C 60% 5 year survival D 40% 5 year survival
E 25% 5 year survival F Virtually 0% 5 year survival
G Almost 100% 5 year survival H Overall life-time risk ~1%
I Overall life-time risk ~2% J Overall life-time risk _1.5%
K Overall life-time risk ~3% L Overall life-time risk of 0.1%

Instructions: For each scenario described below, choose the single most appropriate information that should
be given to the woman from the above list of options. Each option may be used once, more than once, or not
at all.

A 68 year old woman attends the gynaecology clinic 4 weeks


after TAH + BSO for endometrial carcinoma. The histology Your answer: A
Question 15 shows that the tumour had extended to involve the cervix and
she has been advised to have radiotherapy. She enquires about Correct answer: B
the survival rate for women with similar tumours.

A 32 year old teacher has been referred to the gynaecology clinic


because of marked anxiety since her friend died suddenly from Your answer: A
Question 16 ovarian cancer at the age of 35 years. She has no family history
of ovarian cancer and enquires about the likelihood of any Correct answer: I
woman developing the disease.

| EXPLANATION |

Options for Questions 17-18

A Endocervical and urethral swabs for culture B EUA + cystoscopy


C MRI scan D Out-patient hysteroscopy and biopsy
E Hysteroscopy D&C F Vulval biopsy
G Cervical smear H Peritoneal fluid for cytology
I Serum CA-125 J Colposcopy

Instructions: For each scenario described below, choose the single most appropriate investigation from the
above list of options. Each option may be used once, more than once, or not at all.

A 56 year old woman with a 3 year history of vulval lichen Your answer: A
sclerosis complains of a firm lump on the labia majora which has
Question 17
increased in size over the last 3 months and is bleeding
intermittently Correct answer: F

A 34 year old woman complains of a 3 months history of post- Your answer: A


coital bleeding. The cervix appears normal and microbiology
Question 18
swabs are negative. Cervical smear shows atypical glandular
cells Correct answer: J

| EXPLANATION |

Options for Questions 19-20

A Congenital syphilis B Congenital varicella syndrome


C Parvovirus B19 infection D Turners syndrome
E Parder-Willy syndrome F Fetal hydantoin syndrome
G Group B streptococcal infection H Congenital rubella syndrome
I Cri-du-chat syndrome J Edwards syndrome
K Pataus syndrome L Downs syndrome

Instructions: For each of the case histories described below, choose the single most likely cause of fetal
abnormality from the above list of options. Each option may be used once, more than once, or not at all.

A 42 year old woman is late booking for antenatal care. The Your answer: A
estimated gestation age by ultrasound scan is 22 weeks and the
Question 19
fetus is found to have an atrio-ventricular septal defect and a
double bubble sign in the upper abdomen Correct answer: L

A 42 year old woman had bleeding in early pregnancy and was


found to have an 8 week viable pregnancy. Anomaly scan at 22 Your answer: A
Question 20 weeks showed a head and abdominal circumferences below the
third centile , bilateral choroids plexus cysts and rockerbottom Correct answer: J
feet

| EXPLANATION |

Options for Questions 21-22

A No additional intervention at this stage B Deliver by caesarean section


C Evacuation of retained products of conception D Artificial rupture of membranes
E Oxytocin induction of labour F Venous access and resuscitation
G Controlled artificial rupture of membranes in theatre H Intra-muscular analgesia
I Prostaglandin induction of labour J Manual replacement of the uterus
K Sub-cutaneous terbutalline L Delay induction of labour

Instructions: For each of the case histories described below, choose the single most appropriate
management from the above list. Each option may be used once, more than once, or not at all.

A 34 year old nulliparous woman presents in spontaneous labour Your answer: A


at 37 weeks gestation with the cervix 3cm dilated. Four hours
Question 21
later, the cervix is 8cm dilated and a frank breech presentation is
diagnosed. Correct answer: B
A 24 year old primigravida has been admitted for induction at 38
weeks gestation with a dichorionic twin pregnancy because of Your answer: A
maternal discomfort. The pregnancy has otherwise been
Question 22
uncomplicated and twin 1 is cephalic. The cervix is 2cm long,
posterior, soft and the os is closed. The presenting part is 3cm Correct answer: I
above the spines. CTG is reactive.

| EXPLANATION |

Options for Questions 23-24

A Cystoscopy B MRI scan


C Urodynamic studies D Bladder re-training
E Urine for cytology F Cystoscopy and biopsy
G Examination under anaesthesia H MSU for culture and sensitivity
I Urine electrolytes J Renal function tests

Instructions: For each of the case histories described below, choose the single most important investigation
from the above list of options. Each option may be used once, more than once, or not at all.

A 60 year old woman developed cervical cancer 12 months Your answer: A


earlier and was treated with combined chemotherapy and
Question 23
radiotherapy. She now complains of blood in her urine which
occurs all the time Correct answer: F

A 45 year old woman complained initially of urinary frequency


urgency and urge incontinence and also leaked urine on
coughing or straining. She was treated medically initially and her Your answer: A
Question 24 symptoms of urinary frequency and urgency have largely
resolved. Over the last 6 months, she is finding that urinary Correct answer: C
leakage on coughing and straining is increasingly affecting her
social life.

| EXPLANATION |

Options for Questions 25-26

A In-vitro fertilisation B Intra-uterine insemination


C Laparoscopy and dye test D Intra-cytoplasmic sperm injection
E Clomephene citrate F Gonadotrophin induction of ovulation
G Metformin H Oocyte donation
I Surrogacy J Weight reduction
K Weight gain L Measure serum androgen concentrations

Instructions: For each of the case histories described below, choose the single most appropriate initial
management from the above list. Each option may be used once, more than once, or not at all.

A 24 year old woman with her 35 year old partner has been
referred to the infertility clinic because of 2 years of primary
infertility. The woman has an irregular menstrual cycle every 2-4
months and her partners semen analysis is normal. The Your answer: A
Question 25 womans BMI is 26 and there is no significant past medical
history. Investigations have confirmed a diagnosis of polycystic Correct answer: C
ovary. The couple have not achieved a pregnancy despite a 6
months course of clomephene citrate with evidence of ovulation
based on day 21 progesterone concentration and ultrasound
scanning.

A 34 year old woman with her 35 year old partner has been
referred to the infertility clinic because of 2 years of primary
infertility. The woman has a regular 28 day cycle and her periods
are heavy with severe dysmenorrhoea. She also complains of Your answer: A
deep dyspareunia and intermittent lower abdominal and pelvic
Question 26
pain. Her BMI is 27. Investigations have shown normal LH, FHS,
Thyroid function tests, prolactin and day 21 progesterone Correct answer: A
concentration confirms ovulatory cycles. Laparoscopy and dye
test shows evidence of chronic pelvic inflammatory disease with
bilateral tubal occlusion. Her partners semen analysis is normal.

| EXPLANATION |

Options for Questions 27-28

A Counsel and recommend appropriate contraception B Counsel and offer termination of pregnancy
C Commence folic acid 0.4mg/day D Commence folic acid 4mg/day
E Change anti-hypertensive drug to captopril F Stop anti-hypertensive medication
G Stop propylthiouracil H Change therapy to carbimazole
I Reassure J Commence block and replace therapy
K Explain risk of neonatal thyrotoxicosis L Change therapy to propylthiouracil
M Increase dose of thyroxine N Explain 20% risk of neonatal hypothyroidism

Instructions: For each scenario described below, choose the single most appropriate management from the
above list of options. Each option may be used once, more than once, or not at all.

A 30 year old woman with Graves disease is adequately Your answer: A


Question 27 controlled on carbimazole is planning a pregnancy. She attends
the pre-conception clinic enquiring about the risks to her fetus Correct answer: K

A 25 year old insulin dependent diabetic with well controlled Your answer: A
Question 28 disease is planning a pregnancy and attends the pre-conception
clinic. All observations and investigations are within normal limits. Correct answer: D

| EXPLANATION |

Options for Questions 29-30

A TAH + BSO B Vaginal hysterectomy


C Radical abdominal hysterectomy D Laparoscopic assisted vaginal hysterectomy
E TAH + BSO + omentectomy F BSO
G TAH H Chemotherapy
I Radiotherapy J Combined chemo-radiotherapy
K Endometrial biopsy

Instructions: For each scenario described below, choose the single most appropriate management from the
above list of options. Each option may be used once, more than once, or not at all.

A 65 year old woman presents with a 6 months history of post-


menopausal bleeding. Endometrial biopsy showed an Your answer: A
Question 29 endometrial adenocarcinoma and MRI scan shows that the
tumour has infiltrated the outer 50% of the myometrium but does Correct answer: A
not involve the cervix or adnexal structures
An asymptomatic 56 years old woman is referred to the
gynaecology clinic because of a cervical smear showing atypical Your answer: A
Question 30 glandular cells. The trans-vaginal scan requested by the general
practitioner shows a normal size uterus with a regular 8mm Correct answer: K
endometrium with normal ovaries.

| EXPLANATION |

Options for Questions 31-32

A Maternal treatment with antibiotics B Detailed fetal anomaly scan


C Deliver by caesarean section D Active immunisation of neonate
E Avoid breast-feeding F Counsel and offer termination of pregnancy
G Delay delivery by 5-7 days H Administer corticosteroids
I Administer tocolytics J Reassurance
K Screen for maternal syphilis infection

Instructions: For each scenario described below, choose the single most appropriate management from the
above list of options. Each option may be used once, more than once, or not at all.

A 20 year old woman presents with intermittent abdominal pain Your answer: A
Question 31 and a febrile illness at 20 weeks gestation. Blood cultures are
positive for Listeria monocytogenes Correct answer: A

A 30 year old woman has the following results following routine Your answer: A
Question 32 antenatal screening at 15 weeks gestation: VDRL positive, FTA-
abs positive Correct answer: A

| EXPLANATION |

Options for Questions 33-34

A Cancel IVF cycle B Freeze embryos


C Admit for iv fluids and thromboprophylaxis D Termination of pregnancy
E Transfer to ITU F Fluid restriction
G Drain pleural effusion H Drain ascites
I Drain ovarian cysts J Proceed with embryo replacement
K Avoid unprotected sexual intercourse L Diuretics

Instructions: For each of the case histories described below, choose the single most appropriate
management from the above list. Each option may be used once, more than once, or not at all.

A 35 year old woman with PCOS is undergoing IVF treatment.


She attends for embryo replacement 6 days after HCG
administration and complains of abdominal distension and pain Your answer: A
Question 33 with nausea and vomiting 2-3 times per day over the previous
48h. Clinical examination showed a mildly distended abdomen Correct answer: B
and ultrasound scan confirmed bilateral ovarian enlargement 8-
10cm. All her blood tests were normal.

A 35 year old woman with primary infertility is undergoing IVF Your answer: A
treatment. She attends 10 days after embryo replacement and
Question 34
complains of abdominal distension and pain with nausea and
persistent severe vomiting over the previous 48h. Clinical Correct answer: C
examination showed a markedly distended abdomen with ascites
and pleural effusion. Abdominal and pelvic ultrasound scan
confirmed 12cm diameter bilateral cystic ovaries. All her blood
tests were normal.

| EXPLANATION |

Options for Questions 35-36

A Damage to bladder / ureter B Damage to bowel


C Failure rate 1 in 200 D Failure to gain entry into abdominal cavity
E Failure to identify disease F Failure to visualise uterine cavity
G Haemorrhage requiring blood transfusion H Haemorrhage requiring return to theatre
I Laparotomy J Uterine perforation

Instructions: For each of the case histories described below, choose the single most relevant complication
that you must discuss with the patient when taking consent prior to surgery from the above list of options.
Each option may be used once, more than once, or not at all.

A 48-year-old nulliparous woman is scheduled for vaginal Your answer: A


Question 35 hysterectomy because of menorrhagia. Her uterus is enlarged
equivalent to 14 weeks' gestation. Correct answer: I

A 30 year old woman with a 3 year history of abdominal and


pelvic pain that has failed to respond to medical treatment. Her Your answer: A
Question 36 pain is unrelated to menses and she has no bowel symptoms.
She is otherwise fit and well with a BMI of 25 and is listed for Correct answer: E
diagnostic laparoscopy.

| EXPLANATION |

Options for Questions 37-38

A Oestrogen cream B Testosterone cream


C Oral HRT D Topical anti-histamines
E Vaginoplasty F Radical vulvectomy
G Skinning vulvectomy H Topical anti-fungal cream
I Clobetasol cream J Excision biopsy

Instructions: For each clinical scenario below choose the single most appropriate treatment from the list
above. Each diagnosis may be used once, more than once, or not at all.

A 23-year-old woman presents with a two-year history of vulval, Your answer: A


perineal and perianal irritation. The vulva is red, excoriated and
Question 37
there areas of white, thickened skin. Application of 3% Acetic
acid shows areas of mosaic and coarse punctuation. Correct answer: J

A 47 year old woman complains of a 3 months history of vulval Your answer: A


irritation and superficial dyapareunia. The vulval skin is thin and
Question 38
white with fissures and narrowing of the introitus and fusion of
the labia minora over the clitoris Correct answer: I

| EXPLANATION |
Options for Questions 39-40

A Tranexamic acid B Danazol


C GnRH analogues D Laparoscopy and ablation of endometriosis
E Diagnostic laparoscopy F Mefenamic acid
G Pelvic ultrasound scan H MRI scan of the pelvis
I Dilatation and curettage J Endocervical and urethral swabs
K Combined oral contraceptive pill L Depo-medroxyprogesterone acetate
M Refer to pain clinic N Refer to psychiatrist

Instructions: For each of the case histories described below, choose the single most appropriate
management from the above list. Each option may be used once, more than once, or not at all.

A 35 year old woman attends the gynaecology clinic with her 14


year old daughter who is complaining of severe period pains
since the onset of menstruation at the age of 13 years. Menstrual Your answer: A
Question 39 loss is not heavy and she has no bowel symptoms. Her
symptoms are now interfering with her education as she takes Correct answer: F
several days off school every month. She does not require
contraception and clinical examination is normal.

A 33 year old mother of 3 children complains of progressively


painful periods over 5 years. There is occasional deep Your answer: A
Question 40 dyspareunia but she has no bowel symptoms. Her symptoms
have not responded to simple analgesia and non-steroidal anti- Correct answer: K
inflammatory drugs. Clinical examination is normal.

Options for Questions 1-2

A Raloxifene B Oral sequential oestrogen + progestogen


C Oral continuous oestrogen + progestogen D Oestrogen patches
E Vaginal oestrogen tablets F Levonorgestrel IUS + oral oestrogens
G Levonorgestrel IUS H Refer to special oncology clinic
I Counsel and recommend discontinuation of HRT J HRT not recommended

Instructions: For each scenario described below, choose the single most appropriate
management from the above list of options. Each option may be used once, more than
once, or not at all.

A 60 year old woman complains of vaginal dryness and Your answer: A


superficial / deep dyspareunia since discontinuing oral HRT
Question 1
because of media reports. She has used lubricants without
improvement Correct answer: E

A 40 year old woman has TAH + BSO for stage Ia endometroid Your answer: A
Question 2 adenocarcinoma of the ovary and complains of debilitating hot
flushes. Correct answer: D

| EXPLANATION |

Options for Questions 3-4


A Administer VZIG as soon as possible to mother B Administer VZIG to mother if maternal serology -
ve
C Administer VZIG to neonate D Detailed ultrasound examination

E Immediate caesarean section and transfer baby to F Advise avoid contact with other pregnant women
the neonatal unit and neonates
G Induction of labour H Reassurance
I Separate mother and baby after delivery J Serum for VZV IgM antibodies
K Give intravenous Aciclovir L Treat with oral Aciclovir

Instructions: For each scenario described below, choose the single most appropriate
management from the above list of options. Each option may be used once, more than
once, or not at all.

A 26-year-old Para 1+ 0 at 38 weeks gestation contacts her GP Your answer: A


immediately after hearing that a child in her son's nursery has
Question 3
developed chickenpox. On direct questioning, she admits to
having had chicken pox as a child. Correct answer: H

A 26-year-old Para 1+ 0 at 38 weeks gestation contacts her GP Your answer: A


Question 4 immediately after discovering that her husband has shingles. She
has no memory of having chicken-pox in the past. Correct answer: B

| EXPLANATION |

Options for Questions 5-6

A Genital swabs for viral culture B Immediate induction of labour


C Delay delivery by 5-7 days if possible D Offer prophylactic oral acyclovir
E Treat with intra-venous acyclovir F Deliver by caesarean section
G Treat neonate with intravenous acyclovir H Reassurance
I Avoid breast-feeding J Avoid contact with other pregnant women and neonates
K Refer to genito-urinary medicine clinic L Screen for other sexually transmitted infections

Instructions: For each scenario described below, choose the single most appropriate
management from the above list of options. Each option may be used once, more than
once, or not at all.

A 17 year old woman complains of vulval soreness at 18 weeks Your answer: A


Question 5 gestation. She is found to have vulval herpetic vesicles on
examination. There is no history of previous genital herpes. Correct answer: K

A 20 year old woman presents in spontaneous labour at 37 Your answer: A


Question 6 weeks gestation. She developed primary genital herpes at 34
weeks gestation but is now symptom-free. Correct answer: F

| EXPLANATION |

Options for Questions 7-8

A Amniocentesis and karyotype B Amniocentesis and PCR


C CVS and karyotype D CVS and PCR
E Amniocentesis and enzyme assay F Fetal blood sampling and karyotype

G Fetal blood sampling and measurement of PO2 H Fetal blood sampling and haemoglobin
electrophoresis

I Fetal blood sampling and haemoglobin J Fetal blood sampling and viral culture
concentration
K Reassurance

Instructions: For each of the case histories described below, choose the single most
appropriate management from the above list of options. Each option may be used once,
more than once, or not at all.

A 30 year old woman with myotonic dystrophy is 11 weeks Your answer: A


Question 7 pregnant and would like to know whether or not her fetus has
inherited the condition Correct answer: D

A 21 year old woman with 3 previous consecutive first trimester


miscarriages is found to be a t(14:21) balanced translocation Your answer: A
Question 8 carrier. She is 10 weeks pregnant and would like to know
whether or not her fetus has inherited an unbalanced Correct answer: C
translocation.

| EXPLANATION |

Options for Questions 9-10

A Commence iv oxytocin B Deliver by caesarean section


C Fetal blood sampling D No additional intervention at this stage
E Umbilical artery Doppler F Reduce oxytocin dose
G Uterine artery Doppler H Forceps delivery
I Intravenous ritodrine J Administer maternal facial oxygen
K Sub-cutaneous terbutalline L Administer uterotonic agent

Instructions: For each of the case histories described below, choose the single most
appropriate initial management from the above list. Each option may be used once, more
than once, or not at all.

You have been asked to review a 35 year old woman half an


hour after spontaneous vaginal delivery of her first child because Your answer: A
Question 9 the membranes are ragged although the placenta appears
complete. The uterus is well contracted, the lochia are normal Correct answer: D
and all maternal observations are satisfactory.

A 35 year old mother of two children presents in spontaneous


labour at 39 weeks gestation. She has one previous caesarean
section for breech presentation followed by a spontaneous Your answer: A
Question 10 vaginal delivery. At 12:00, the cervix is 7cm dilated with a direct
occipito-anterior position with 3:10 strong contractions. At 16:00, Correct answer: B
the cervix is 7cm dilated and the contractions are inco-ordinate
occurring 2-3:10. The CTG is normal

| EXPLANATION |
Options for Questions 11-12

A Oestrogen cream B Testosterone cream


C Oral HRT D Topical anti-histamines
E Vaginoplasty F Radical vulvectomy
G Skinning vulvectomy H Topical anti-fungal cream
I Clobetasol cream J Excision biopsy

Instructions: For each clinical scenario below choose the single most appropriate treatment
from the list above. Each diagnosis may be used once, more than once, or not at all.

A 23-year-old woman presents with a two-year history of vulval, Your answer: A


perineal and perianal irritation. The vulva is red, excoriated and
Question 11
there areas of white, thickened skin. Application of 3% Acetic
acid shows areas of mosaic and coarse punctuation. Correct answer: J

A 47 year old woman complains of a 3 months history of vulval Your answer: A


irritation and superficial dyapareunia. The vulval skin is thin and
Question 12
white with fissures and narrowing of the introitus and fusion of
the labia minora over the clitoris Correct answer: I

| EXPLANATION |

Options for Questions 13-14

A Administer regional analgesia if APTT is B Administer regional analgesia if APTT and PT are
normal normal

C Administer protamine sulphate then regional D Advice that regional analgesia is contra-indicated
analgesia
E Administer regional analgesia F Administer prophylactic dose of heparin

G Remove epidural catheter H Check APTT then remove epidural catheter if result is
normal

I J Check anti-Xa levels then administer regional


Advise against removal of epidural catheter
analgesia if normal
K Wait for 12h then administer heparin

Instructions: For each scenario described below, choose the single most appropriate
management from the above list of options. Each option may be used once, more than
once, or not at all.

A 40 year old woman with a BMI of 39 has had an emergency


caesarean section for failure to progress at 6cm dilatation under Your answer: A
combined spinal epidural (CSE) analgesia. The procedure was
Question 13
uncomplicated with a blood loss of 700ml. A prophylactic does of
LMWH was administered at noon, 6h after insertion of CSE. She Correct answer: I
requests removal of the epidural catheter at 18:00h.

A 25 year old woman with a BMI of 38 has an emergency


caesarean section at full dilatation under spinal anaesthesia Your answer: A
Question 14 because of fetal distress. The procedure was uncomplicated with
blood loss of 700ml. She is now 8h post-surgery and your Correct answer: F
attention is drawn to instructions on post-partum
thromboprophylaxis.

| EXPLANATION |

Options for Questions 15-16

A Quadruple test B Integrated test


C Nuchal transluscency D Serum integrated test
E Triple test F Anomaly scan
G Fluorescent in-situ hybridisation H Amniocentesis
I Chorionic villus sampling J PCR
K No investigation L Fetal blood sampling

Instructions: For each of the case histories described below, choose the single most
important investigation from the above list of options. Each option may be used once, more
than once, or not at all.

A 30 year old woman has one previous termination of pregnancy Your answer: A
Question 15 at 22 weeks gestation for Edwards syndrome now presents at 10
weeks gestation. Correct answer: I

Your answer: A
A 30 year old woman is found to have a fetus with duodenal
Question 16
atresia on anomaly scan at 20 weeks gestation
Correct answer: H

| EXPLANATION |

Options for Questions 17-18

A In-vitro fertilisation B Intra-uterine insemination


C Laparoscopy and dye test D Laparoscopic ovarian drilling
E Clomephene citrate F Gonadotrophin induction of ovulation
G Metformin H Carbegolline
I Hystero-salpingogram J Weight reduction
K Weight gain L Measure serum androgen concentrations

Instructions: For each of the case histories described below, choose the single most
appropriate initial management from the above list. Each option may be used once, more
than once, or not at all.

A 34 year old woman with her 35 year old partner has been
referred to the infertility clinic because of 2 years of primary
infertility. The woman has a regular and normal 28 day cycle, her Your answer: A
Question 17 BMI is 26 and she has no other symptoms. Investigations have
shown normal LH, FHS, Thyroid function tests, prolactin and day Correct answer: I
21 progesterone concentration confirms ovulatory cycles. Her
partners semen analysis is normal.

A 36 year old woman with her 35 year old partner has been Your answer: A
Question 18
referred to the infertility clinic because of 2 years of primary
infertility. The woman has a regular and normal 28 day cycle, her Correct answer: B
BMI is 26 and she has no other symptoms. Investigations have
shown normal LH = 7.5mIU/ml, FHS = 17mIU/ml, Thyroid
function tests, prolactin and day 21 progesterone concentration
confirms ovulatory cycles. Her partners semen analysis is
normal. Hystero-salpingogram confirms bilateral patent fallopian
tubes.

| EXPLANATION |

Options for Questions 19-20

A Avoid breast-feeding B Administer VZIG to neonate and avoid breast-


feeding

C Administer VZIG to neonate but breast-feeding D Immediate induction of labour


encouraged
E Delay delivery with tocolytics F Offer termination of pregnancy
G Treat neonate with oral acyclovir H Treat neonate with intravenous acyclovir

I Examine neonate for evidence of congenital J Reassurance


anomalies
K Delay delivery for 5-7 days if possible L Detailed ultrasound examination

Instructions: For each scenario described below, choose the single most appropriate
management from the above list of options. Each option may be used once, more than
once, or not at all.

A 20 year old woman is in hospital 2 days after vaginal delivery


of her first baby and is informed that her 8 year old step-daughter Your answer: A
has chicken-pox. She does not remember having chicken pox in
Question 19
the past and is worried about taking her new baby home.
Investigations show that she is varicella zoster IgG positive and Correct answer: J
IgM negative

A 20 year old woman is being treated in the infectious diseases


unit for chicken pox and reports having missed a period with a Your answer: A
Question 20 positive pregnancy test. Dating scan shows that she has a viable
11 week pregnancy and she is discharged following resolution of Correct answer: L
her symptoms

| EXPLANATION |

Options for Questions 21-22

A Serial beta-HCG assay B Laparoscopy


C Repeat trans-vaginal scan in 24h D Repeat trans-vaginal scan in 7 days
E Counsel and discharge F Offer surgical evacuation of products of conception
G Offer medical treatment with methotrexate H Laparotomy
I Serum progesterone assay J Serum AFP assay
K Refer to molar pregnancy centre L Refer to other specialty

Instructions: Instructions: For each patient described below choose the single most
appropriate initial management option from the list. Each option may be used once, more
than once, or not at all.

An asymptomatic 30 year old woman with a previous ectopic


pregnancy presents at 6 weeks amenorrhoea for trans-vaginal Your answer: A
scan to exclude a repeat ectopic pregnancy. The scan is
Question 21
reported to show a 14mm endometrium with no gestation sac.
There are no adnexal masses and no free fluid in the pouch of Correct answer: D
Douglas. HCG concentration is 500IU and 1600IU 48h later

An asymptomatic 30 year old woman with a previous ectopic


pregnancy presents at 6 weeks amenorrhoea for trans-vaginal
scan to exclude a repeat ectopic pregnancy. Her pregnancy test Your answer: A
Question 22 had been positive 2 weeks earlier. The scan is reported to show
a 14mm endometrium with no gestation sac. There is a 4cm Correct answer: B
cystic structure in the right adnexum suggestive of an ectopic
pregnancy but no free fluid in the pouch of Douglas

| EXPLANATION |

Options for Questions 23-24

A Bilateral oophrectomy B CA-125 assays every 6-12 months


C TAH + BSO D Cyst aspiration for cytology
E Refer to cancer centre F Refer to palliative care team
G Reassure H Measure serum CA-125
I Yearly pelvic ultrasound scans J Repeat pelvic ultrasound scan in 4-6 months

Instructions: For each scenario described below, choose the single most appropriate
management from the above list of options. Each option may be used once, more than
once, or not at all.

A 40 year old woman presents with a 12 months history of vague Your answer: A
abdominal discomfort that did not respond to simple analgesia.
Question 23
Ultrasound scan shows bilateral complex ovarian cysts with right
sided hydronephrosis and ascites. Her CA-125 is 1500iu Correct answer: E

A 20 year old woman is admitted with sudden onset left sided


pelvic pain 23 days after her LMP. Her pregnancy test is negative Your answer: A
Question 24 and her symptoms are improving with simple analgesia. Pelvic
ultrasound scan shows a 5cm left ovarian cyst with internal Correct answer: J
echoes consistent with a haemorrhagic cyst.

| EXPLANATION |

Options for Questions 25-26

A Closure of visceral peritoneum B Closure of parietal peritoneum


C Closure of Scarpas fascia D Non-closure of the rectus sheath
E Instillation of local anaesthetic into pelvis F Supra-pubic catheterisation
G Mass closure of abdominal incision H Prophylactic heparin 2h pre-op
I Application of methylene blue to vagina J Closure of vaginal vault
K Excision of vaginal cuff L Use of prophylactic antibiotics after delivery
Instructions: For each of the case histories described below, choose the single most
appropriate intervention that would reduce peri-operative morbidity and mortality from the
above list. Each option may be used once, more than once, or not at all.

Your answer: A
A 25 year old woman is undergoing an emergency caesarean
Question 25
section at 9cm dilatation for failure to progress
Correct answer: L

Your answer: A
A 42 year old woman is to undergo TAH for dysfunctional uterine
Question 26
bleeding. She is otherwise fit and well with a BMI of 35.
Correct answer: H

| EXPLANATION |

Options for Questions 27-28

A Atrophic vulvovaginitis B Human papilloma virus infection


C Benign mucous membrane pemphigoid D Candida infection
E Lichen sclerosus F Contact dermatitis
G Lichen simplex et chronicus H Herpes simplex infection
I Vulval intraepithelial neoplasia J Vulvodynia

Instructions: For each clinical scenario below choose the single most likely diagnosis from
the list above. Each diagnosis may be used once, more than once, or not at all.

A 23-year-old woman presents with a two-year history of vulval, Your answer: A


perineal and perianal irritation. The vulva is red, excoriated and
Question 27
there areas of white, thickened skin. Application of 3% Acetic
acid shows areas of mosaic and coarse punctuation. Correct answer: I

A 78-year-old woman presents with vulval irritation and soreness.


On examination the vulva is red in colour, slightly oedematous Your answer: A
Question 28 and there are small, red papules scattered randomly beyond the
perimeter of the vulva. She also complains of soreness and Correct answer: D
irritation under the breasts.

| EXPLANATION |

Options for Questions 29-30

A No additional intervention at this stage B Deliver by caesarean section


C Increase dose of oxytocin D Lovsets manoeuvre
E Decrease dose of oxytocin F Fetal blood sampling
G Forceps delivery H Delivery of the posterior arm
I Episiotomy J Inhaled salbutamol
K McRoberts manoeuvre L Administer maternal facial oxygen

Instructions: For each of the case histories described below, choose the single most
appropriate management from the above list. Each option may be used once, more than
once, or not at all.

A 34 year old woman with 3 previous vaginal deliveries presents


in spontaneous labour at 39 weeks gestation. The cervix is fully
dilated with thick meconium stained liquor. The CTG shows a Your answer: A
Question 29 base-line fetal heart rate of 135bpm with variability of 3-4bpm
with variable decelerations. There are 3-4 uterine contractions Correct answer: G
every 10 minutes. The fetus is in a direct occipito-anterior
position 1cm below the ischial spines.

You have been called into a delivery room by the senior midwife
because of difficulties delivering the fetal shoulders. On your Your answer: A
Question 30 arrival, the woman is in lithotomy and the head had been
delivered 2 minutes earlier but moderate traction has thus far Correct answer: K
failed to deliver the shoulders.

| EXPLANATION |

Options for Questions 31-32

A Fetal blood sampling B Triple test


C Quadruple test D Integrated test
E Chorionic villus sampling F Serum integrated test
G Amniocentesis H Nuchal transluscency
I Anomaly scan J Fluorescent in-situ hybridisation
K PCR L Restriction fragment length polymorphisms
M Karyotype

Instructions: For each of the conditions described below, choose the single most
appropriate diagnostic tests from the above list of options. Each option may be used once,
more than once, or not at all.

Your answer: A
Question 31 Fetal anaemia
Correct answer: A

Your answer: A
Question 32 Fetal gastroschisis
Correct answer: I

| EXPLANATION |

Options for Questions 33-34

A Emergency caesarean section B Ultrasound scan for placental site


C Induction of labour with prostaglandins D Induction of labour by amniotomy
E Oxytocin augmentation of labour F Fetal scalp blood sampling
G Vaginal operative delivery H Maternal blood transfusion
I Transfer to high dependency unit J Expectant management
K Arrange antenatal clinic follow-up L Treatment with tocolytics
M Umbilical artery Doppler N Perform Kleihauer test
Instructions: For each of the case histories described below, choose the single most
appropriate management from the above list of options. Each option may be used once,
more than once, or not at all.

A 42 year old woman had an elective caesarean section at 37 Your answer: A


weeks gestation for major placenta previa. She returned to
Question 33
theatre 6h later because of suspected intra-abdominal bleeding
and has undergone a total abdominal hysterectomy Correct answer: I

A 35 year old Rhesus positive woman presented with fresh


vaginal bleeding and intermittent abdominal pain at 30 weeks Your answer: A
gestation. Maternal pulse on admission was 90bpm with BP
Question 34
120/80. The fetal heart rate was normal. She has been in
hospital for 48h and complains of a slight brown discharge but no Correct answer: K
other symptoms and has had no further bleeding.

| EXPLANATION |

Options for Questions 35-36

A Increase iv fluids B Insert central venous pressure line

C Intravenous magnesium sulphate D Measure serum aspartate transaminase


immediately
E Measure serum magnesium F Blood transfusion
G Monitor patellar reflex every 15 minutes H Provide a fluid challenge with colloids
I Provide intravenous Hartmann J Decrease iv fluids

K Immediate dose of 10ml 10% calcium gluconate L Carry out visual field assessment
intravenously

Instructions: For each patient described below choose the single most appropriate
management option from the list. Each option may be used once, more than once, or not at
all.

A 20-year-old primigravida is 30 weeks' pregnant and has been


transferred to the delivery suite with severe gestational
proteinuric hypertension. She complains of severe frontal Your answer: A
Question 35 headache but has no other symptoms. She has a normal
respiratory rate and her urine output has been satisfactory. Her Correct answer: C
blood pressure is 140/100mmHg. There are five beats of bilateral
ankle clonus.

A 20-year-old primigravida delivered a live infant 5 hours


previously by emergency caesarean section. She has developed
severe gestational proteinuric hypertension. Blood loss at
caesarean section was estimated at 1,000 ml. BP = Your answer: A
Question 36 160/100mmHg on oral anti-hypertensive treatment and pulse =
95bpm. She has been given one litre of Hartmann's solution Correct answer: B
intravenously since her delivery but has passed 40ml of urine
since delivery. Her clotting, liver enzymes and serum creatinine
are normal.

| EXPLANATION |
Options for Questions 37-38

A Uterine perforation B Laparotomy


C Bladder injury D Bleeding requiring transfusion
E Continuing pregnancy F Infertility
G Wound infection H Failure to identify any pathology
I Failure to gain access to abdominal cavity J Risk of failure of the procedure

Instructions: For each of the case histories described below, choose the single most
relevant complication that you must discuss with the patient when taking consent prior to
surgery from the above list of options. Each option may be used once, more than once, or
not at all.

A 34 year old woman with a previous left salpingectomy for Your answer: A
ectopic pregnancy presents with abdominal pain and vaginal
Question 37
bleeding and is found to have an empty uterus with a right
adnexal mass consistent with an ectopic pregnancy. Correct answer: F

A 35 year old woman with a 10 week missed miscarriage is Your answer: A


Question 38 undergoing evacuation of retained products of conception under
general anaesthesia Correct answer: A

| EXPLANATION |

Options for Questions 39-40

A Rocker bottom feet B Exomphalos


C Holoprosencephaly D Cystic hygroma
E Gastroschisis F atrio-ventricular septal defect
G Intra-cranial calcification H Duodenal atresia
I Spina bifida J Polyhydramnios
K Echogenic bowel

Instructions: For each of the conditions described below, choose the single most
characteristic abnormality from the above list of options. Each option may be used once,
more than once, or not at all.

Your answer: A
Question 39 Turners syndrome
Correct answer: D

Your answer: A
Question 40 Trisomy 13
Correct answer: C
Your score is 8% for this EMQ Exam
You answered 3 out of 40 questions correctly.

40 random questions for EMQ Mock Exam

| EXPLANATION |

Options for Questions 1-2

A No additional intervention at this stage B Deliver by caesarean


section
Welcome SEMSEM
C Evacuation of retained products of D Artificial rupture of
conception membranes Your last visit was on
Wednesday Apr 05th,
E Bimanual compression of the uterus F Venous access and 2006
resuscitation

G Controlled artificial rupture of membranes H Intra-muscular analgesia You have been a member
in theatre for 1 days.
I J Manual replacement of the
Prostaglandin induction of labour
uterus
Account Manager
K Sub-cutaneous terbutalline L Commence iv oxytocin
Edit my details
MCQ test scores
Instructions: For each of the case histories described below, choose the single
Mock MCQ scores
most appropriate management from the above list. Each option may be used
EMQ test scores
once, more than once, or not at all.
Mock EMQ scores
Logout
You have been asked to review a 25 year old
woman half an hour after spontaneous vaginal Your answer: A
delivery of her first child because a placental BUSY SpR
Question 1
cotyledon is retained. The uterus is well Correct
contracted, the lochia are normal and all answer: C Home
maternal observations are satisfactory.
About us

A 23 year old woman with two previous vaginal Signup/Join


deliveries is admitted for induction of labour at Your answer: A
42 weeks gestation. The pregnancy has been Lost password?
Question 2 uncomplicated and the fetal head is three fifths
Correct Medical forums
palpable. The cervix is 1cm long, central, 2cm
dilated with bulging membranes and the fetal answer: D
Contributors
head is 2cm above the ischial spines.
Bibliography

| EXPLANATION | Contact us

Recommend us
Options for Questions 3-4
Privacy
A Tranexamic acid B Danazol
C GnRH analogues D TAH + BSO
E Diagnostic laparoscopy F Medroxyprogesterone acetate
G Pelvic ultrasound scan H MRI scan of the pelvis
I Dilatation and curettage J Endocervical and urethral swabs

K Combined oral contraceptive L Levonorgestrel releasing intra-uterine


pill system
M Refer to pain clinic N Refer to gastroenterologist

Instructions: For each of the case histories described below, choose the single
most appropriate management from the above list. Each option may be used
once, more than once, or not at all.

A 22 year old woman is known to have


endometriosis diagnosed at laparoscopy. She
is referred to the gynaecology clinic because of Your answer: A
progressively severe pelvic pain,
Question 3 dysmenorrhoea and deep dyspareunia. There
is a past medical history of focal migraine. Correct
Clinical examination showed a tender pelvis answer: F
with a normal size anteverted uterus and no
adnexal masses.

A 22 year old woman is referred to the


gynaecology clinic because of a 2 year history
of persistent pelvic pain which is worse during
and after intercourse and during menstruation. Your answer: A
She also complains of intermittent abdominal
Question 4 bloating with alternating constipation and
diarrhoea and urgency of defecation. There is Correct
occasional rectal bleeding which is unrelated to answer: N
her menses and usually occurs during episodes
of constipation. Clinical examination is normal
and there is no significant past medical history.

| EXPLANATION |

Options for Questions 5-6

A Raloxifene B Oral sequential oestrogen +


progestogen
C Oral continuous oestrogen + D Oestrogen patches
progestogen
E Vaginal oestrogen tablets F Tamoxifen
G Levonorgestrel IUS H Refer to special oncology clinic

I Counsel and recommend J HRT not recommended


discontinuation of HRT

Instructions: For each scenario described below, choose the single most
appropriate management from the above list of options. Each option may be
used once, more than once, or not at all.

Your answer: A
A 40 year old woman has TAH + BSO for stage
Question 5 Ia ovarian cancer and complains of distressing
hot flushes and vaginal dryness Correct
answer: D

A 52 year old woman with a previous PE Your answer: A


complains of infrequent periods and distressing
Question 6
hot flushes. Her sister died from PE while on Correct
the COCP at the age of 38 years. answer: J

| EXPLANATION |

Options for Questions 7-8

A Urinary retention B Damage to bowel


C Removal of ovaries D Failure to gain entry into abdominal
cavity
E Failure to identify disease F Failure to visualise uterine cavity

G Haemorrhage requiring blood H Haemorrhage requiring return to


transfusion theatre
I Laparotomy J Uterine perforation

Instructions: For each of the case histories described below, choose the single
most relevant complication that you must discuss with the patient when taking
consent prior to surgery from the above list of options. Each option may be used
once, more than once, or not at all.

Your answer: A
A 30 year old woman with genuine stress
Question 7 incontinence who is due to have a tension-free
vaginal tape Correct
answer: A

A 30 year old woman with primary infertility who Your answer: A


is due to undergo IVF treatment and is
Question 8 scheduled to have laparoscopy and ovarian
cystectomies for severe endometriosis with Correct
bilateral ovarian endometriomas. answer: C

| EXPLANATION |

Options for Questions 9-10

A Abdominal pressure due to B Bladder neck weakness


mass

C Pelvic floor muscle damage D Congenital weakness of the pelvic


floor
E Obstetric trauma F Oestrogen deficiency
G Neurological disease H Drug side-effects
I Tumour infiltration into bladder J Bacterial urinary tract infection
K Schistosomiasis L Radiation injury

Instructions: For each of the case histories described below, choose the single
most likely cause of urinary tract symptoms from the above list of options. Each
option may be used once, more than once, or not at all.

A 35 year old woman complains of a 2 month


history of urinary frequency, urgency and Your answer: A
haematuria. She had an abdominal X-ray 2
Question 9 weeks earlier for an unrelated complaint which
showed evidence of bladder calcification. Correct
Cystoscopy showed numerous polypoid lesions answer: K
in the bladder

A 52 year old woman complains of


progressively worsening urinary frequency and Your answer: A
urgency 3 years after radical abdominal
Question 10
hysterectomy and radiotherapy for cervical Correct
cancer. There is no haematuria and all answer: F
investigations including cystoscopy are normal

| EXPLANATION |
Options for Questions 11-12

A Commence oral warfarin B Await results of D-dimers assay

C Commence therapeutic dose of D Prophylactic dose of heparin for 3-5


heparin days post-partum

E Antenatal prophylactic treatment F Antenatal treatment with warfarin


with heparin

G Prophylactic heparin for 6 weeks H Continue s/c heparin for 6 weeks


post-partum

I J Continue heparin therapy for 6


Stop heparin therapy
months

K No treatment required L Continue heparin prophylaxis during


labour

Instructions: For each scenario described below, choose the single most
appropriate management from the above list of options. Each option may be
used once, more than once, or not at all.

A 20 year old woman is known to be a carrier of


the factor V Leiden mutation following Your answer: A
screening because of a DVT in her sister. She
Question 11
attends the antenatal clinic at 15 weeks in her Correct
first pregnancy. Her BMI is 22 and she is answer: G
otherwise fir and well.

A 30 year old woman complains of a painful Your answer: A


swollen left lower limb at 32 weeks gestation
Question 12 and is treated with heparin. Subsequent
Doppler studies have shown a left popliteal vein Correct
thrombosis. answer: J

| EXPLANATION |

Options for Questions 13-14

A Stop heparin on the evening before B Commence oral warfarin


induction

C Await results of V/Q scan then D Await results of venogram then


commence treatment commence treatment

E Await results of D-dimers assay F Commence therapeutic dose of


heparin

G Commence prophylactic dose of H Oral aspirin therapy


heparin post-partum

I Antenatal prophylactic treatment J Antenatal treatment with warfarin


with heparin

K Prophylactic heparin for 6 weeks L Stop heparin therapy


post-partum

Instructions: For each scenario described below, choose the single most
appropriate management from the above list of options. Each option may be
used once, more than once, or not at all.

A 30 year old woman with a previous DVT Your answer: A


presents for antenatal care at 15 weeks
Question 13 gestation. There is no family history of VTE and
her BMI is 25. She is otherwise fit and well with Correct
a negative thrombophilia screen answer: K
A 30 year old woman with recurrent first
trimester miscarriages is known to have the Your answer: A
anti-phospholipid antibody syndrome and has
Question 14 had a spontaneous vaginal delivery at 39
weeks gestation. Her BMI is 27 and she is Correct
otherwise fit and well with no family history of answer: G
VTE.

| EXPLANATION |

Options for Questions 15-16

A Take pill as soon as possible B Take pill as soon as possible +


barrier contraception for 7 days

C Take pill as soon as possible and D Administer emergency contraceptive


omit pill-free week pill
E Insert copper IUCD F Await results of pregnancy test

G Replace detached patch with a H Replace with a new patch and use
new patch barrier contraception for 7 days
I Apply new patch and use barrier J Apply new patch but keep the same
contraception for 7 days change date

K Apply new patch and delay L Apply new patch and delay change
change day by 24h day by 36h

Instructions: For each of the case histories described below, choose the single
most appropriate advice from the above list. Each option may be used once,
more than once, or not at all.

A 30 year old nulliparous woman is using the Your answer: A


combined contraceptive patch. She placed her
Question 15 second patch at 10pm before going to bed and
having sexual intercourse and realised at 10am Correct
that the patch had detached. answer: J

A 29 year old mother of 3 children is using the


combined contraceptive patch. She placed her Your answer: A
second patch at 10pm on a Wednesday and
Question 16
realised 3 days later that the patch had Correct
detached. She had sexual intercourse on answer: I
several occasions.

| EXPLANATION |

Options for Questions 17-18

A Chorionic villus sampling B Amniocentesis


C Fetal blood sampling D Triple test
E Karyotype F Integrated test
G Western blotting H Northern blotting
I Anomaly scan J Southern blotting
K PCR

Instructions: For each of the case histories described below, choose the single
most relevant investigation from the above list of options. Each option may be
used once, more than once, or not at all.
Your answer: A
A 35 year old woman with myotonic dystrophy
Question 17 is undergoing IVF treatment in order to ensure
that the disease is not passed on to her child Correct
answer: K

A 35 year old woman is found to have raised Your answer: A


maternal serum AFP at 16 weeks gestation.
Question 18 Anomaly scan is equivocal because of high
body mass index and she has opted to have Correct
amniocentesis answer: G

| EXPLANATION |

Options for Questions 19-20

A Tranexamic acid B Danazol

C GnRH analogues D Laparoscopy and ablation of


endometriosis
E Diagnostic laparoscopy F Mefenamic acid
G Pelvic ultrasound scan H MRI scan of the pelvis
I Dilatation and curettage J Endocervical and urethral swabs

K Combined oral contraceptive L Depo-medroxyprogesterone acetate


pill
M Refer to pain clinic N Refer to psychiatrist

Instructions: For each of the case histories described below, choose the single
most appropriate management from the above list. Each option may be used
once, more than once, or not at all.

A 35 year old woman attends the gynaecology


clinic with her 14 year old daughter who is
complaining of severe period pains since the
onset of menstruation at the age of 13 years. Your answer: A
Menstrual loss is not heavy and she has no
Question 19
bowel symptoms. Her symptoms are now Correct
interfering with her education as she takes answer: F
several days off school every month. She does
not require contraception and clinical
examination is normal.

A 33 year old mother of 3 children complains of


progressively painful periods over 5 years. Your answer: A
There is occasional deep dyspareunia but she
Question 20 has no bowel symptoms. Her symptoms have
not responded to simple analgesia and non- Correct
steroidal anti-inflammatory drugs. Clinical answer: K
examination is normal.

| EXPLANATION |

Options for Questions 21-22

A Counsel and offer evacuation of B Counsel and offer support group


retained products of conception

C Counsel and offer laparoscopy & D Counsel and offer investigation


salpingectomy for recurrent miscarriage
E Counsel and offer treatment with F Counsel and offer laparotomy &
methotrexate salpingectomy

G Counsel and offer termination of H Perform salpingectomy


pregnancy
I Proceed to laparotomy J Serial HCG assay

K Counsel and refer to early pregnancy L Repeat trans-vaginal scan 7 days


assessment unit later

Instructions: For each patient described below choose the single most
appropriate initial management option from the list. Each option may be used
once, more than once, or not at all.

A 25 year old woman has a transvaginal scan Your answer: A


following IVF treatment during which 2 embryos
Question 21 were replaced and is found to have a 6 week
singleton intra-uterine pregnancy with no fetal Correct
heart activity. answer: A

A 25 year old nulliparous woman has had 2 Your answer: A


previous first trimester miscarriages presents at
Question 22 8 weeks gestation for an early pregnancy scan.
She is found to have a 7 week missed Correct
miscarriage. answer: A

| EXPLANATION |

Options for Questions 23-24

A Congenital adrenal hyperplasia B Maternal androgen therapy


C Downs syndrome D Turners syndrome
E Maternal diabetes mellitus F Maternal lithium therapy
G Maternal warfarin therapy H Maternal heparin therapy
I Group B streptococcus infection J Maternal anti-epileptic drug therapy
K Maternal smoking L Maternal cocaine abuse

Instructions: For each of the case histories described below, choose the single
most likely cause of neonatal collapse from the above list of options. Each option
may be used once, more than once, or not at all.

Your answer: A
A 6 hour old neonate delivered at 35 weeks
Question 23 gestation following prolonged pre-term rupture
of the membranes Correct
answer: I

A 6 hour old neonate delivered by rotational Your answer: A


forceps delivery for prolonged second stage of
Question 24 labour. Initial examination showed epicanthic
folds with up-slanting palpebral fissures and a Correct
loud systolic murmur answer: C

| EXPLANATION |

Options for Questions 25-26

A Endometriosis B Acute PID


C Chronic PID D Irritable bowel syndrome
E Residual ovary syndrome F Ovarian remnant syndrome
G Diverticular disease H Interstitial cystitis
I Primary dysmenorrhoea J Adenomyosis
K Uterine fibroids L Polycystic ovary syndrome
M Inflammatory bowel disease N Chronic constipation

Instructions: For each of the case histories described below, choose the single
most likely cause of chronic pelvic pain from the above list. Each option may be
used once, more than once, or not at all.

A 23 year old woman has been referred to the


gynaecology clinic with a 6 months history of
lower abdominal and pelvic pain and deep Your answer: A
dyspareunia. Her symptoms are particularly
Question 25 worse during menstruation. Over the last 4
months, she has experienced episodes of Correct
bloody diarrhoea and has lost about 5kg in answer: M
weight. Her Hb = 10.0g/dl, white cell count = 15
x 10E9/l and platelet count = 450 x 10E9/l.

A 35 year old woman complains of a 2 year


history of progressively painful periods with
occasional deep pain during intercourse. She Your answer: A
denies any bowel or urinary symptoms. Clinical
Question 26 examination showed a bulky smooth and
tender uterus with no adnexal masses and no Correct
adnexal tenderness. Her symptoms have not answer: J
responded to non-steroidal anti-inflammatory
agents and diagnostic laparoscopy was normal.

| EXPLANATION |

Options for Questions 27-28

A Commence iv oxytocin B Deliver by caesarean section

C Evacuation of retained products of D Artificial rupture of


conception membranes

E Bimanual compression of the uterus F Venous access and


resuscitation
G Examination under spinal anaesthesia H Intra-muscular analgesia

I J Manual replacement of the


Intravenous ritodrine
uterus
K Sub-cutaneous terbutalline L Administer uterotonic agent

Instructions: For each of the case histories described below, choose the single
most appropriate management from the above list. Each option may be used
once, more than once, or not at all.

A 34 year old woman is bleeding heavily during


the third stage of labour. Her pulse is 60bpm Your answer: A
and systolic BP is 100/40mmHg. There is a
Question 27
mass at the introitus consistent with uterine Correct
inversion. She has venous access with answer: J
adequate fluid resuscitation

A 23 year old primigravida is admitted for Your answer: A


induction of labour at 39 weeks gestation
Question 28
because of symphysis pubis diastasis. Three Correct
hours after vaginal prostaglandin answer: H
administration, she complains of painful
contractions every minute. CTG shows
contractions 8:10 lasting ~30s. The fetal heart
rate is 140bpm with good variability and
accelerations.

| EXPLANATION |

Options for Questions 29-30

A Intravenous labetalol B Immediate delivery by caesarean


section

C Intravenous magnesium D Measure serum aspartate transaminase


sulphate immediately

E Measure FBC and clotting F Blood transfusion


profile
G Immediate induction of labour H Provide a fluid challenge with colloids
I Antihypertensive treatment J Administer iv phenytoin

K Measure 24h urine protein L Arrange in-utero transfer to tertiary


excretion centre

Instructions: For each patient described below choose the single most
appropriate management option from the list. Each option may be used once,
more than once, or not at all.

A 30 year old primigravida attends as an


emergency at 32 weeks gestation with a twin
pregnancy and complains of 12h of severe
vomiting and upper abdominal pain. Her BP is
130/80mmHg with a trace of proteinuria. A Your answer: A
growth scan one week earlier had shown
Question 29
normal fetal growth. Investigations show a Correct
serum aspartate transaminase concentration of answer: B
600iu/l, alkaline phosphatase = 1550iu/l, serum
uric acid = 0.57mmol/l and serum creatinine =
130micromol/l. FBC and coagulation profile are
normal.

A 32 year old primigravida with a previously


normal pregnancy is seen in the antenatal clinic Your answer: A
at 39 weeks gestation with a BP of
Question 30
160/95mmHg and 3+ proteinuria. The uterus is Correct
appropriate for gestation age with a cephalic answer: G
presentation.

| EXPLANATION |

Options for Questions 31-32

A Routine recall in 3 years B Routine recall in 5 years


C Hysteroscopy + D&C D Laser vapourisation
E Refer for colposcopy F Cold knife cone biopsy
G Radical trachylectomy H Cryotherapy
I No further cervical smears required J Repeat cervical smear in 6 months

Instructions: For each scenario described below, choose the single most
appropriate management from the above list of options. Each option may be
used once, more than once, or not at all.

A 40 year old nulliparous woman had a cervical Your answer: A


smear showing severe dyskaryosis.
Question 31 Colposcopy with diathermy loop excision has
been performed and the histology shows CIN III Correct
which has been completely excised answer: J

A 53 year old asymptomatic post menopausal


woman has a cervical smear showing no Your answer: A
dyskaryotic cells but no evidence of
Question 32 transformation zone sampling. The cervix was
well visualised and normal and she has Correct
previously had regular smears which have all answer: B
been negative.

| EXPLANATION |

Options for Questions 33-34

A Take pill as soon as possible B Take pill as soon as possible +


barrier contraception for 7 days

C Take pill as soon as possible and D Administer emergency contraceptive


omit pill-free week pill
E Insert copper IUCD F Await results of pregnancy test

G Replace detached patch with a H Replace with a new patch and use
new patch barrier contraception for 7 days

I Apply new patch and use barrier J Apply new patch but keep the same
contraception for 7 days change date

K Apply new patch and delay L Apply new patch and delay change
change day by 24h day by 36h

Instructions: For each of the case histories described below, choose the single
most appropriate advice from the above list. Each option may be used once,
more than once, or not at all.

A 30 year old nulliparous woman is using the Your answer: A


combined contraceptive patch. She placed her
Question 33 second patch at 10pm before going to bed and
having sexual intercourse and realised 36h Correct
later that the patch had detached answer: I

Your answer: A
A 35 year old woman is using the contraceptive
Question 34 patch and telephones for advice because she
has missed her patch change date by 24h Correct
answer: J

| EXPLANATION |

Options for Questions 35-36

A Bilateral oophrectomy B CA-125 assays every 6-12 months


C TAH + BSO D Cyst aspiration for cytology
E Refer to cancer centre F Ovarian cystectomy
G Reassure H Measure serum CA-125
I Refer to palliative care team J Paracentesis for cytology
K Chemotherapy L Radiotherapy

Instructions: For each scenario described below, choose the single most
appropriate management from the above list of options. Each option may be
used once, more than once, or not at all.

A 56 year old post-menopausal woman with


diverticular disease has an ultrasound scan Your answer: A
because of a suspected diverticular abscess.
Question 35
There is no evidence of pelvic abscess but the Correct
scan shows a 4cm septate left ovarian cyst with answer: H
no free fluid.

A 30 year old nulliparous woman complains of Your answer: A


a 6 months history of deep dyspareunia that
Question 36 has not responded to medical treatment. Pelvic
ultrasound scan shows a 6cm complex right Correct
ovarian mass consistent with a dermoid cyst. answer: F

| EXPLANATION |

Options for Questions 37-38

A Atrophic vulvovaginitis B Human papilloma virus


infection

C Benign mucous membrane D Candida infection


pemphigoid
E Lichen sclerosus F Contact dermatitis
G Lichen simplex et chronicus H Herpes simplex infection
I Vulval intraepithelial neoplasia J Vulvodynia

Instructions: For each clinical scenario below choose the single most likely
diagnosis from the list above. Each diagnosis may be used once, more than
once, or not at all.

A 23-year-old woman presents with a two-year


history of vulval, perineal and perianal irritation. Your answer: A
The vulva is red, excoriated and there areas of
Question 37
white, thickened skin. Application of 3% Acetic Correct
acid shows areas of mosaic and coarse answer: I
punctuation.

A 78-year-old woman presents with vulval


irritation and soreness. On examination the Your answer: A
vulva is red in colour, slightly oedematous and
Question 38 there are small, red papules scattered
randomly beyond the perimeter of the vulva. Correct
She also complains of soreness and irritation answer: D
under the breasts.

| EXPLANATION |

Options for Questions 39-40

A Failure to relieve symptoms B Damage to bowel


C Removal of ovary D Irregular bleeding for 3-4 months
E Failure to identify disease F Failure to visualise uterine cavity
G Shoulder-tip pain H Haemorrhage requiring return to theatre
I Laparotomy J Failure rate 1 in 200

Instructions: For each of the case histories described below, choose the single
most relevant complication that you must discuss with the patient when taking
consent prior to surgery from the above list of options. Each option may be used
once, more than once, or not at all.

An 18 year old woman is known to have an Your answer: A


8cm simple ovarian cyst and admitted as an
Question 39 emergency with torsion. She is due to undergo
laparoscopic ovarian cystectomy. She is Correct
otherwise healthy with a BMI of 27. answer: C

A 56 year old woman had successful trans-


cervical resection of the endometrium eight Your answer: A
years earlier for menorrhagia and now presents
Question 40
with post-menopausal bleeding. She is Correct
otherwise healthy and has been listed for answer: F
hysteroscopy D&C

Home | Signup | Subscribe | Contact us | Medical forums | Privacy | Legal Notices | Copyright
Busy SpR
Busy SpR Ltd. is a Registered Company of the United Kingdom. Company Registration Number:4407908
Developed by Last Digital
Options for Questions 1-2

A Congenital adrenal B Maternal androgen therapy


hyperplasia
C Downs syndrome D Turners syndrome
E Maternal diabetes mellitus F Maternal lithium therapy
G Maternal warfarin therapy H Maternal heparin therapy

I Group B streptococcus J Maternal anti-epileptic drug


infection therapy
K Maternal smoking L Maternal cocaine abuse

Instructions: For each of the case histories described below, choose


the single most likely cause of neonatal collapse from the above list
of options. Each option may be used once, more than once, or not at Welcome SEMSEM
all. Your last visit was on
Wednesday Apr 05th,
Your answer: 2006
A
A 6 hour old neonate with ambiguous You have been a member
Question 1
genitalia for 1 days.
Correct
answer: A
Account Manager
Your answer: Edit my details
A MCQ test scores
A 3 hour old neonate weighing 4900g at
Question 2 Mock MCQ scores
birth
Correct EMQ test scores
answer: E Mock EMQ scores
Logout

| EXPLANATION |
BUSY SpR
Options for Questions 3-4
Home

A Congenital adrenal B Maternal androgen therapy About us


hyperplasia
Signup/Join
C Downs syndrome D Turners syndrome
E Maternal diabetes mellitus F Maternal lithium therapy Lost password?
G Maternal warfarin therapy H Maternal heparin therapy Medical forums

I Group B streptococcus J Maternal anti-epileptic drug Contributors


infection therapy
K Maternal smoking L Maternal cocaine abuse Bibliography

Contact us
Instructions: For each of the case histories described below, choose
the single most likely cause of neonatal collapse from the above list Recommend us
of options. Each option may be used once, more than once, or not at
Privacy
all.

Your answer:
A 6 hour old neonate delivered at 35 A
Question 3 weeks gestation following prolonged pre-
term rupture of the membranes Correct
answer: I

A 6 hour old neonate delivered by Your answer:


rotational forceps delivery for prolonged A
second stage of labour. Initial
Question 4
examination showed epicanthic folds
with up-slanting palpebral fissures and a Correct
loud systolic murmur answer: C

Home | Signup | Subscribe | Contact us | Med


Options for Questions 1-2

A Take pill as soon as possible B Take pill as soon as possible +


barrier contraception for 7 days
C Take pill as soon as possible D Administer emergency
and omit pill-free week contraceptive pill
E Insert copper IUCD F Await results of pregnancy test
Replace with a new patch and
G Replace detached patch H use barrier contraception for 7
with a new patch
days
Apply new patch and use
I barrier contraception for 7 J Apply new patch but keep the
same change date
days

K Apply new patch and delay L Apply new patch and delay Welcome SEMSEM
change day by 24h change day by 36h
Your last visit was on
Wednesday Apr 05th,
Instructions: For each of the case histories described below, choose 2006
the single most appropriate advice from the above list. Each option
may be used once, more than once, or not at all. You have been a member
for 1 days.
A 30 year old nulliparous woman is using Your answer:
Question 1
the combined contraceptive patch. She A
placed her second patch at 10pm before Account Manager
going to bed and having sexual Correct
intercourse and realised 36h later that the Edit my details
answer: I
patch had detached MCQ test scores
Mock MCQ scores
EMQ test scores
Your answer:
A 35 year old woman is using the Mock EMQ scores
A
contraceptive patch and telephones for Logout
Question 2
advice because she has missed her
patch change date by 24h Correct
answer: J BUSY SpR

Home
| EXPLANATION |
About us
Options for Questions 3-4 Signup/Join

Lost password?
A Commence iv oxytocin B Deliver by caesarean
section Medical forums

C Evacuation of retained products of D Artificial rupture of Contributors


conception membranes
Bibliography
E Bimanual compression of the F Venous access and
uterus resuscitation Contact us
G Examination under spinal H Intra-muscular analgesia Recommend us
anaesthesia

I J Manual replacement of Privacy


Intravenous ritodrine
the uterus

K Sub-cutaneous terbutalline L Administer uterotonic


agent

Instructions: For each of the case histories described below, choose


the single most appropriate management from the above list. Each
option may be used once, more than once, or not at all.

A 34 year old woman is bleeding heavily


during the third stage of labour. Her pulse Your answer:
is 60bpm and systolic BP is A
Question 3 100/40mmHg. There is a mass at the
introitus consistent with uterine inversion. Correct
She has venous access with adequate answer: J
fluid resuscitation

A 23 year old primigravida is admitted for


induction of labour at 39 weeks gestation
because of symphysis pubis diastasis. Your answer:
Three hours after vaginal prostaglandin A
Question 4 administration, she complains of painful
contractions every minute. CTG shows Correct
contractions 8:10 lasting ~30s. The fetal answer: H
heart rate is 140bpm with good variability
and accelerations.

| EXPLANATION |

Options for Questions 5-6

A No additional intervention at this B Deliver by caesarean


stage section

C Increase dose of oxytocin D Artificial rupture of


membranes
E Decrease dose of oxytocin F Fetal blood sampling

G Controlled artificial rupture of H Intra-muscular analgesia


membranes in theatre
I Continue oxytocin at current dose J Inhaled salbutamol

K Sub-cutaneous terbutalline L Administer maternal


facial oxygen

Instructions: For each of the case histories described below, choose


the single most appropriate management from the above list. Each
option may be used once, more than once, or not at all.

A 24 year old woman with 2 previous


vaginal deliveries presents in
spontaneous labour at 40 weeks
gestation. She progresses to 9cm Your answer:
dilatation and there is thick old meconium A
Question 5 stained liquor. The CTG shows a base-
line fetal heart rate of 135bpm with Correct
variability of ~10bpm, with occasional answer: A
accelarations and no decelerations.
There are 3-4 uterine contractions every
10 minutes.

A 34 year old primigravida presents in


spontaneous labour at 39 weeks
gestation. The cervix is 4cm dilated with Your answer:
thin meconium stained liquor. The CTG A
Question 6 over 25 minutes shows a base-line fetal
heart rate of 135bpm with variability of 3- Correct
4bpm, no accelerations and variable answer: F
decelerations. There are 3-4 uterine
contractions every 10 minutes.

| EXPLANATION |

Options for Questions 7-8

A No additional intervention at this B Deliver by caesarean


stage section
C Evacuation of retained products of D Artificial rupture of
conception membranes

E Oxytocin induction of labour F Venous access and


resuscitation

G Controlled artificial rupture of H Intra-muscular analgesia


membranes in theatre

I J Manual replacement of
Prostaglandin induction of labour
the uterus
K Sub-cutaneous terbutalline L Delay induction of labour

Instructions: For each of the case histories described below, choose


the single most appropriate management from the above list. Each
option may be used once, more than once, or not at all.

A 34 year old nulliparous woman Your answer:


presents in spontaneous labour at 37 A
weeks gestation with the cervix 3cm
Question 7
dilated. Four hours later, the cervix is
8cm dilated and a frank breech Correct
presentation is diagnosed. answer: B
A 24 year old primigravida has been
admitted for induction at 38 weeks
gestation with a dichorionic twin Your answer:
pregnancy because of maternal A
discomfort. The pregnancy has otherwise
Question 8
been uncomplicated and twin 1 is
cephalic. The cervix is 2cm long, Correct
posterior, soft and the os is closed. The answer: I
presenting part is 3cm above the spines.
CTG is reactive.

| EXPLANATION |

Options for Questions 9-10

A No additional intervention at this B Deliver by caesarean


stage section

C Evacuation of retained products of D Artificial rupture of


conception membranes
E Commence iv oxytocin regimen F Venous access and
resuscitation

G Controlled artificial rupture of H Intra-muscular analgesia


membranes in theatre

I J Manual replacement of
Prostaglandin induction of labour
the uterus
K Sub-cutaneous terbutalline L Delay induction of labour

Instructions: For each of the case histories described below, choose


the single most appropriate management from the above list. Each
option may be used once, more than once, or not at all.

A 25 year old primigravida presents for


induction of labour at 42 weeks gestation.
Artificial rupture of the membranes (ARM) Your answer:
is performed at 09:00 at 1cm dilatation A
and oxytocin commenced at 10:00. An
Question 9
epidural is subsequently inserted and
oxytocin is increased to a maximum of Correct
32mu/min according to the unit protocol. answer: B
12h after ARM, the cervix is 1cm dilated
and partially effaced.

A 34 year old woman with four previous


vaginal deliveries has been admitted for
induction of labour at 40 weeks gestation Your answer:
because she has a history of precipitate A
labours with her last baby being delivered
Question 10
in the car. The fetal head is five fifths
palpable and the cervix is 2cm long, soft Correct
and posterior with a multiparous os. The answer: E
presenting part is not reached on vaginal
examination.

| EXPLANATION |

Options for Questions 11-12

A Continue iv oxytocin B Deliver by caesarean


section

C Evacuation of retained products D No additional intervention


of conception at this stage

E Bimanual compression of the F Venous access and


uterus resuscitation

G Examination under spinal H Lovsets manoeuvre


anaesthesia

I J Manual replacement of the


Intravenous ritodrine
uterus
K Sub-cutaneous terbutalline L Administer uterotonic agent

Instructions: For each of the case histories described below, choose


the single most appropriate initial management from the above list.
Each option may be used once, more than once, or not at all.

A 16 year old primigravida attends for


induction of labour following intra-uterine
fetal death at 36 weeks gestation.
Maternal BP is 120/80 with no proteinuria Your answer:
and all blood tests are normal. The fetus A
Question 11 is in a breech presentation. Prostaglandin
is administered and oxytocin commenced Correct
at 12:00 according to the unit protocol. At answer: A
00:00, the cervix remains 1cm long and
the os is closed. There are 3 uterine
contractions every 10 minutes.

A 34 year old woman is bleeding heavily Your answer:


and collapses during the third stage of A
Question 12 labour. Her pulse is 40bpm and systolic
BP is 70mmHg. There is a mass at the Correct
introitus consistent with uterine inversion answer: F

| EXPLANATION |

Options for Questions 13-14

A Take pill as soon as possible B Take pill as soon as possible +


barrier contraception for 7 days
C Take pill as soon as possible D Administer emergency
and omit pill-free week contraceptive pill
E Insert copper IUCD F Await results of pregnancy test
Replace with a new patch and
G Replace detached patch H use barrier contraception for 7
with a new patch
days
Apply new patch and use
I barrier contraception for 7 J Apply new patch but keep the
same change date
days

K Apply new patch and delay L Apply new patch and delay
change day by 24h change day by 36h

Instructions: For each of the case histories described below, choose


the single most appropriate advice from the above list. Each option
may be used once, more than once, or not at all.

A 30 year old nulliparous woman is using Your answer:


Question 13 the combined contraceptive patch. She A
placed her second patch at 10pm before
going to bed and having sexual Correct
intercourse and realised at 10am that the answer: J
patch had detached.

A 29 year old mother of 3 children is Your answer:


using the combined contraceptive patch. A
She placed her second patch at 10pm on
Question 14
a Wednesday and realised 3 days later
that the patch had detached. She had Correct
sexual intercourse on several occasions. answer: I

| EXPLANATION |

Options for Questions 15-16

A Commence iv oxytocin B Deliver by caesarean section

C Fetal blood sampling D No additional intervention at this


stage
E Umbilical artery Doppler F Reduce oxytocin dose
G Uterine artery Doppler H Forceps delivery
I Intravenous ritodrine J Administer maternal facial oxygen

K Sub-cutaneous L Administer uterotonic agent


terbutalline

Instructions: For each of the case histories described below, choose


the single most appropriate initial management from the above list.
Each option may be used once, more than once, or not at all.

You have been asked to review a 35 year


old woman half an hour after Your answer:
spontaneous vaginal delivery of her first A
child because the membranes are ragged
Question 15
although the placenta appears complete.
The uterus is well contracted, the lochia Correct
are normal and all maternal observations answer: D
are satisfactory.

A 35 year old mother of two children


presents in spontaneous labour at 39
weeks gestation. She has one previous
caesarean section for breech Your answer:
presentation followed by a spontaneous A
Question 16 vaginal delivery. At 12:00, the cervix is
7cm dilated with a direct occipito-anterior Correct
position with 3:10 strong contractions. At answer: B
16:00, the cervix is 7cm dilated and the
contractions are inco-ordinate occurring
2-3:10. The CTG is normal

| EXPLANATION |

Options for Questions 17-18

A Increase oxytocin dose B Deliver by caesarean section


C Fetal blood sampling D Continuous CTG monitoring
E Umbilical artery Doppler F Reduce oxytocin dose
G Uterine artery Doppler H Inhaled salbutamol
I Intravenous ritodrine J Administer maternal facial oxygen
K Sub-cutaneous terbutalline L Administer uterotonic agent

Instructions: For each of the case histories described below, choose


the single most appropriate initial management from the above list.
Each option may be used once, more than once, or not at all.

A 34 year old primigravida attends for


induction of labour at 42 weeks gestation.
She progresses normally to 8cm Your answer:
dilatation at which point the dose of A
Question 17 oxytocin is 16mu/min. The CTG shows 7
uterine contractions every 10 minutes Correct
lasting 30-40s with a base-line fetal heart answer: F
rate of 160bpm with good variability and
variable decelerations.

A 23 year old primigravida is admitted for


induction of labour at 42 weeks gestation. Your answer:
Three hours after vaginal prostaglandin A
administration, she complains of painful
Question 18
contractions every minute. CTG shows
contractions 8:10 lasting ~30s. The fetal Correct
heart rate is 140bpm with good variability answer: K
and deep variable decelerations.

| EXPLANATION |

Options for Questions 19-20

A No additional intervention at this B Deliver by caesarean


stage section
C Increase dose of oxytocin D Lovsets manoeuvre
E Decrease dose of oxytocin F Fetal blood sampling
G Forceps delivery H Delivery of the posterior arm
I Episiotomy J Inhaled salbutamol

K McRoberts manoeuvre L Administer maternal facial


oxygen

Instructions: For each of the case histories described below, choose


the single most appropriate management from the above list. Each
option may be used once, more than once, or not at all.

A 34 year old woman with 3 previous


vaginal deliveries presents in
spontaneous labour at 39 weeks
gestation. The cervix is fully dilated with Your answer:
thick meconium stained liquor. The CTG A
Question 19 shows a base-line fetal heart rate of
135bpm with variability of 3-4bpm with Correct
variable decelerations. There are 3-4 answer: G
uterine contractions every 10 minutes.
The fetus is in a direct occipito-anterior
position 1cm below the ischial spines.

You have been called into a delivery Your answer:


room by the senior midwife because of A
Question 20
difficulties delivering the fetal shoulders.
On your arrival, the woman is in lithotomy Correct
and the head had been delivered 2 answer: K
minutes earlier but moderate traction has
thus far failed to deliver the shoulders.

| EXPLANATION |

Options for Questions 21-22

A No additional intervention at this B Deliver by caesarean


stage section
C Evacuation of retained products of D Artificial rupture of
conception membranes
E Commence iv oxytocin regimen F Fetal blood sampling

G Controlled artificial rupture of H Intra-muscular analgesia


membranes in theatre
I Prostaglandin induction of labour J Inhaled salbutamol

K Sub-cutaneous terbutalline L Administer maternal


facial oxygen

Instructions: For each of the case histories described below, choose


the single most appropriate management from the above list. Each
option may be used once, more than once, or not at all.

A 34 year old primigravida presents in


spontaneous labour at 39 weeks
gestation. The cervix is 4cm dilated with Your answer:
thin meconium stained liquor. The CTG A
Question 21 over 25 minutes shows a base-line fetal
heart rate of 135bpm with variability of 3- Correct
4bpm, no accelerations and no answer: A
decelerations. There are 3-4 uterine
contractions every 10 minutes.

A 34 year old primigravida presents in


spontaneous labour at 39 weeks
gestation. The cervix is 4cm dilated with Your answer:
thin meconium stained liquor. The CTG A
Question 22 over shows a base-line fetal heart rate of
135bpm with variability of 3-4bpm, no Correct
accelerations and no decelerations over a answer: F
period of 120minutes. There are 3-4
uterine contractions every 10 minutes.

| EXPLANATION |

Options for Questions 23-24

A No additional intervention at this B Deliver by caesarean


stage section
C Increase dose of oxytocin D Artificial rupture of
membranes
E Decrease dose of oxytocin F Fetal blood sampling

G Controlled artificial rupture of H Intra-muscular analgesia


membranes in theatre
I Continue oxytocin at current dose J Inhaled salbutamol

K Sub-cutaneous terbutalline L Administer maternal


facial oxygen
Instructions: For each of the case histories described below, choose
the single most appropriate management from the above list. Each
option may be used once, more than once, or not at all.

A 34 year old woman attends for


induction of labour at 42 weeks gestation.
4hours after prostaglandin administration, Your answer:
she complains of painful contractions. A
The CTG shows contractions 6:10 lasting
Question 23
30-45s with a baseline fetal heart rate of
170bpm and late decelerations. The Correct
cervix is 2cm dilated and there is thick answer: B
fresh meconium on artificial rupture of
membranes.

A 40 year woman with three previous


vaginal deliveries attends for induction of
labour at 42 weeks. She has artificial
rupture of membranes at 09:00
(multiparous os) and oxytocin
commenced at 11:00. Epidural analgesia Your answer:
is inserted at 13:00 but is ineffective. As a A
Question 24 consequence, the maximum dose of
oxytocin administered is 4mu/min. Correct
Epidural is re-sited at 19:00. At 23:00, answer: I
she has adequate analgesia, the oxytocin
dose is 16mu/min and she has 3-4
contractions every 10 minutes. The cervix
is fully effaced, 2cm dilated and the CTG
is reactive.

| EXPLANATION |

Options for Questions 25-26

A No additional intervention at this B Deliver by caesarean


stage section
C Increase dose of oxytocin D Lovsets manoeuvre
E Decrease dose of oxytocin F Fetal blood sampling
G Forceps delivery H Delivery of the posterior arm
I Episiotomy J Inhaled salbutamol

K McRoberts manoeuvre L Administer maternal facial


oxygen

Instructions: For each of the case histories described below, choose


the single most appropriate management from the above list. Each
option may be used once, more than once, or not at all.

A 40 year woman with three previous


vaginal deliveries attends for induction of
labour at 42 weeks. She has artificial
rupture of membranes at 09:00 Your answer:
(multiparous os) and oxytocin A
Question 25 commenced at 11:00. Epidural analgesia
is inserted at 13:00 but is ineffective. As a Correct
consequence, the maximum dose of answer: I
oxytocin administered is 4mu/min.
Epidural is re-sited at 19:00. At 23:00,
she has adequate analgesia, the oxytocin
dose is 16mu/min and she has 3-4
contractions every 10 minutes. The cervix
is fully effaced, 2cm dilated and the CTG
is reactive.

A 40 year woman with three previous


vaginal deliveries attends for induction of
labour at 42 weeks. She has artificial
rupture of membranes at 09:00
(multiparous os) and oxytocin
commenced at 11:00. Epidural analgesia Your answer:
is inserted at 13:00 but is ineffective. As a A
Question 26 consequence, the maximum dose of
oxytocin administered is 4mu/min. Correct
Epidural is re-sited at 19:00. At 23:00, answer: I
she has adequate analgesia, the oxytocin
dose is 16mu/min and she has 3-4
contractions every 10 minutes. The cervix
is fully effaced, 2cm dilated and the CTG
is reactive.

| EXPLANATION |

Options for Questions 27-28

A No additional intervention at this B Deliver by caesarean


stage section

C Evacuation of retained products of D Artificial rupture of


conception membranes

E Bimanual compression of the uterus F Venous access and


resuscitation

G Controlled artificial rupture of H Intra-muscular analgesia


membranes in theatre

I J Manual replacement of
Prostaglandin induction of labour
the uterus
K Sub-cutaneous terbutalline L Commence iv oxytocin

Instructions: For each of the case histories described below, choose


the single most appropriate management from the above list. Each
option may be used once, more than once, or not at all.

You have been asked to review a 25 year


old woman half an hour after Your answer:
spontaneous vaginal delivery of her first A
Question 27 child because a placental cotyledon is
retained. The uterus is well contracted, Correct
the lochia are normal and all maternal answer: C
observations are satisfactory.

A 23 year old woman with two previous


vaginal deliveries is admitted for induction Your answer:
of labour at 42 weeks gestation. The A
pregnancy has been uncomplicated and
Question 28
the fetal head is three fifths palpable. The
cervix is 1cm long, central, 2cm dilated Correct
with bulging membranes and the fetal answer: D
head is 2cm above the ischial spines.
Home | Signup | Subscribe | Contact us | Medical forums | Privacy | Legal Notices | Copyright
Busy SpR
Busy SpR Ltd. is a Registered Company of the United Kingdom. Company Registration Number:4407908
Developed by Last Digital
Options for Questions 1-2

A Urinary retention B Damage to bowel


C Removal of ovaries D Failure to gain entry into abdominal cavity
E Failure to identify disease F Failure to visualise uterine cavity
G Haemorrhage requiring blood transfusion H Haemorrhage requiring return to theatre
I Laparotomy J Failure rate 1 in 200

Instructions: For each of the case histories described below, choose the single most relevant complication that
you must discuss with the patient when taking consent prior to surgery from the above list of options. Each
option may be used once, more than once, or not at all.

A 30 year old mother of 5 children who is scheduled to undergo Your answer: A


Question 1 laparoscopic sterilisation and has expressed a personal objection
to blood transfusion. Correct answer: G

A 35 year old mother of three children is using the combined oral Your answer: A
Question 2 contraceptive pill and is scheduled to undergo laparoscopic
sterilisation Correct answer: J

| EXPLANATION |

Options for Questions 3-3

A Closure of visceral peritoneum B Closure of parietal peritoneum


C Closure of Scarpas fascia D Non-closure of the rectus sheath
E Instillation of local anaesthetic into pelvis F Supra-pubic catheterisation
G Mass closure of abdominal incision H Prophylactic heparin 2h pre-op
I Application of methylene blue to vagina J Closure of vaginal vault
K Instillation of local anaesthetic into pelvis L Use of prophylactic antibiotics after delivery

Instructions: For each of the case histories described below, choose the single most appropriate intervention
that would reduce peri-operative morbidity and mortality from the above list. Each option may be used once,
more than once, or not at all.

Your answer: A
A 35 year old mother of 4 children is to undergo laparoscopic
Question 3
sterilisation. She is otherwise healthy with a BMI of 27.
Correct answer: K

| EXPLANATION |

Options for Questions 4-5

A Urinary retention B Damage to bowel


C Removal of ovaries D Failure to gain entry into abdominal cavity
E Failure to identify disease F Failure to visualise uterine cavity
G Haemorrhage requiring blood transfusion H Haemorrhage requiring return to theatre
I Laparotomy J Uterine perforation

Instructions: For each of the case histories described below, choose the single most relevant complication that
you must discuss with the patient when taking consent prior to surgery from the above list of options. Each
option may be used once, more than once, or not at all.

Your answer: A
A 30 year old woman with genuine stress incontinence who is due
Question 4
to have a tension-free vaginal tape
Correct answer: A

A 30 year old woman with primary infertility who is due to undergo Your answer: A
IVF treatment and is scheduled to have laparoscopy and ovarian
Question 5
cystectomies for severe endometriosis with bilateral ovarian
endometriomas. Correct answer: C

| EXPLANATION |

Options for Questions 6-7

A Damage to bladder / ureter B Damage to bowel


C Failure rate 1 in 200 D Failure to gain entry into abdominal cavity
E Failure to identify disease F Failure to visualise uterine cavity
G Haemorrhage requiring blood transfusion H Haemorrhage requiring return to theatre
I Laparotomy J Uterine perforation

Instructions: For each of the case histories described below, choose the single most relevant complication that
you must discuss with the patient when taking consent prior to surgery from the above list of options. Each
option may be used once, more than once, or not at all.

A 52-year-old woman with frequent heavy periods is listed for Your answer: A
Question 6 diagnostic hysteroscopy. She has had two children both delivered
by caesarean section. She is hypertensive and her BMI is 26. Correct answer: J

A 56-year-old woman is scheduled for laparotomy and possible


bilateral salpingo-oophorectomy for an ovarian mass. She had a Your answer: A
Question 7 total abdominal hysterectomy at the age of forty for fibroids and is
in discomfort with an ovarian mass which measures 15cm in Correct answer: B
diameter on ultrasound examination.

| EXPLANATION |

Options for Questions 8-9

A Failure to relieve symptoms B Damage to bowel


C Removal of ovary D Irregular bleeding for 3-4 months
E Failure to identify disease F Failure to visualise uterine cavity
G Shoulder-tip pain H Haemorrhage requiring return to theatre
I Laparotomy J Failure rate 1 in 200

Instructions: For each of the case histories described below, choose the single most relevant complication that
you must discuss with the patient when taking consent prior to surgery from the above list of options. Each
option may be used once, more than once, or not at all.

An 18 year old woman is known to have an 8cm simple ovarian Your answer: A
Question 8
cyst and admitted as an emergency with torsion. She is due to
undergo laparoscopic ovarian cystectomy. She is otherwise Correct answer: C
healthy with a BMI of 27.

A 56 year old woman had successful trans-cervical resection of Your answer: A


the endometrium eight years earlier for menorrhagia and now
Question 9
presents with post-menopausal bleeding. She is otherwise healthy
and has been listed for hysteroscopy D&C Correct answer: F

| EXPLANATION |

Options for Questions 10-11

A Failure to relieve symptoms B Damage to bowel


C Removal of ovaries D Irregular bleeding for 3-4 months
E Failure to identify disease F Failure to visualise uterine cavity
G Shoulder-tip pain H Haemorrhage requiring return to theatre
I Laparotomy J Failure rate 1 in 200

Instructions: For each of the case histories described below, choose the single most relevant complication that
you must discuss with the patient when taking consent prior to surgery from the above list of options. Each
option may be used once, more than once, or not at all.

A 35 year old nulliparous woman with menorrhagia is having the Your answer: A
Question 10 levonorgestrel releasing intra-uterine system inserted under
general anaesthesia Correct answer: D

Your answer: A
A 40 year old woman with a previous failed TVT for genuine stress
Question 11
incontinence is having a Burch colposuspension
Correct answer: A

| EXPLANATION |

Options for Questions 12-13

A Closure of visceral peritoneum B Closure of parietal peritoneum


C Closure of Scarpas fascia D Non-closure of the rectus sheath
E Instillation of local anaesthetic into pelvis F Supra-pubic catheterisation
G Mass closure of abdominal incision H Use of absorbable suture material
I Application of methylene blue to vagina J Closure of vaginal vault
K Excision of vaginal cuff L Non-closure of the peritoneum

Instructions: For each of the case histories described below, choose the single most appropriate intervention
that would reduce peri-operative morbidity and mortality from the above list. Each option may be used once,
more than once, or not at all.

A 70 year old woman with stage Ia endometrial cancer on MRI Your answer: A
Question 12 scan is to undergo TAH +BSO. She has no significant medical or
surgical history and her BMI is 27 Correct answer: L

A 45 year old woman is to undergo laparotomy for a 15cm Your answer: A


Question 13 complex ovarian mass. She is otherwise healthy and has a BMI of
40 Correct answer: G
| EXPLANATION |

Options for Questions 14-15

A Closure of visceral peritoneum B Closure of parietal peritoneum


C Closure of Scarpas fascia D Non-closure of the rectus sheath
E Instillation of local anaesthetic into pelvis F Supra-pubic catheterisation
G Mass closure of abdominal incision H Prophylactic heparin 2h pre-op
I Application of methylene blue to vagina J Closure of vaginal vault
K Excision of vaginal cuff L Use of prophylactic antibiotics after delivery

Instructions: For each of the case histories described below, choose the single most appropriate intervention
that would reduce peri-operative morbidity and mortality from the above list. Each option may be used once,
more than once, or not at all.

Your answer: A
A 25 year old woman is undergoing an emergency caesarean
Question 14
section at 9cm dilatation for failure to progress
Correct answer: L

Your answer: A
A 42 year old woman is to undergo TAH for dysfunctional uterine
Question 15
bleeding. She is otherwise fit and well with a BMI of 35.
Correct answer: H

| EXPLANATION |

Options for Questions 16-17

A Damage to bladder / ureter B Damage to bowel


C Failure rate 1 in 200 D Failure to gain entry into abdominal cavity
E Failure to identify disease F Failure to visualise uterine cavity
G Haemorrhage requiring blood transfusion H Haemorrhage requiring return to theatre
I Laparotomy J Uterine perforation

Instructions: For each of the case histories described below, choose the single most relevant complication that
you must discuss with the patient when taking consent prior to surgery from the above list of options. Each
option may be used once, more than once, or not at all.

A 48-year-old nulliparous woman is scheduled for vaginal Your answer: A


Question 16 hysterectomy because of menorrhagia. Her uterus is enlarged
equivalent to 14 weeks' gestation. Correct answer: I

A 30 year old woman with a 3 year history of abdominal and pelvic


pain that has failed to respond to medical treatment. Her pain is Your answer: A
Question 17 unrelated to menses and she has no bowel symptoms. She is
otherwise fit and well with a BMI of 25 and is listed for diagnostic Correct answer: E
laparoscopy.

Options for Questions 1-2


A Raloxifene B Oral sequential oestrogen + progestogen
C Oral continuous oestrogen + progestogen D Oestrogen patches
E Alteration of therapy not necessary F Levonorgestrel IUS + oral oestrogens
G Levonorgestrel IUS H Refer to special oncology clinic
I Counsel and recommend discontinuation of HRT J HRT not recommended

Instructions: For each scenario described below, choose the single most appropriate
management from the above list of options. Each option may be used once, more than once,
or not at all.
A 50 year old woman has been referred to the gynaecology clinic Your answer: A
because her 60 year old sister has developed an osteoporotic
Question 1
fracture. She is healthy with mild menopausal symptoms but is
concerned about the risk of osteoporosis Correct answer: J

A 55 year old woman attends the gynaecology clinic for oestrogen Your answer: A
implant. She had TAH + BSO at the age of 45 years for a torted
Question 2
benign ovarian cyst and has yearly oestrogen implants. She
currently has minimal menopausal symptoms Correct answer: I

| EXPLANATION |

Options for Questions 3-4


A Raloxifene B Oral sequential oestrogen + progestogen
C Oral continuous oestrogen + progestogen D Oestrogen patches
E Alteration of therapy not necessary F Levonorgestrel IUS + oral oestrogens
G Levonorgestrel IUS H Refer to special oncology clinic
I Counsel and recommend discontinuation of HRT J HRT not recommended

Instructions: For each scenario described below, choose the single most appropriate
management from the above list of options. Each option may be used once, more than once,
or not at all.
Your answer: A
A healthy 52 year old woman is due to be admitted for pelvic floor
Question 3
repair. She is taking continuous combined HRT and her BMI is 28.
Correct answer: E

A healthy 48 year old woman complains of infrequent periods and Your answer: A
severe hot flushes. She experienced mood swings, depression
Question 4
and fluid retention when she took the COCP in her thirties and had
to discontinue treatment. She uses the sheath for contraception. Correct answer: F

| EXPLANATION |

Options for Questions 5-6


A Commence iv erythromycin B Commence oral co-amoxiclav
C Commence oral erythromycin D Corticosteroids + oral erythromycin
E Administer corticosteroids for fetal lung maturity F Administer corticosteroids + tocolytic
G Arrange in-utero transfer to tertiary centre H Arrange ex-utero transfer to tertiary centre
I Induction of labour + iv benzyl penicillin J Counsel and recommend termination of pregnancy

Instructions: For each scenario described below, choose the single most appropriate
management from the above list of options. Each option may be used once, more than once,
or not at all.
A 32 year old primigravida was found to have group B Your answer: A
Question 5 streptococcal bacteriuria at 16 weeks gestation which was
successfully treated. She presents at 36 weeks gestation with Correct answer: I
spontaneous rupture of the membranes. All maternal and fetal
observations are within normal limits, there are no contractions
and the cervix is not dilated

An 18 year old primigravida presents with spontaneous rupture of


the membranes at 20 weeks gestation and is managed Your answer: A
expectantly. Seven days later, she complains of feeling generally
Question 6
unwell and is noted to have a temperature of 38C and pulse =
115bpm. The uterus is tender with mild contractions every 10 Correct answer: J
minutes and the liquor is offensive. The fetal heart is 120bpm

| EXPLANATION |

Options for Questions 7-8


A Raloxifene B Oral sequential oestrogen + progestogen

C Oral continuous oestrogen + progestogen D Check thrombophilia screen before commencing


HRT
E Vaginal oestrogen tablets F Levonorgestrel IUS + oral oestrogens
G Levonorgestrel IUS H Refer to special oncology clinic

I Counsel and recommend discontinuation of J HRT not recommended


HRT

Instructions: For each scenario described below, choose the single most appropriate
management from the above list of options. Each option may be used once, more than once,
or not at all.
A 52 year old woman on continuous combined HRT complains of Your answer: A
persistent vaginal bleeding but no other symptoms. Trans-vaginal
Question 7
ultrasound scan shows an endometrial thickness of 18mm with no
adnexal masses Correct answer: I

A 52 year old woman is wheel-chair bound because of an accident Your answer: A


Question 8 and complains of debilitating hot flushes. Her BMI is 29 and her
sister died from a pulmonary embolus at the age of 48 years. Correct answer: D

| EXPLANATION |

Options for Questions 9-10


A Raloxifene B Oral sequential oestrogen + progestogen
C Oral continuous oestrogen + progestogen D Oestrogen patches
E Vaginal oestrogen tablets F Levonorgestrel IUS + oral oestrogens
G Levonorgestrel IUS H Refer to special oncology clinic
I Counsel and recommend discontinuation of HRT J HRT not recommended

Instructions: For each scenario described below, choose the single most appropriate
management from the above list of options. Each option may be used once, more than once,
or not at all.
A 60 year old woman complains of vaginal dryness and superficial Your answer: A
Question 9 / deep dyspareunia since discontinuing oral HRT because of
media reports. She has used lubricants without improvement Correct answer: E

A 40 year old woman has TAH + BSO for stage Ia endometroid Your answer: A
Question 10 adenocarcinoma of the ovary and complains of debilitating hot
flushes. Correct answer: D
| EXPLANATION |

Options for Questions 11-12


A Raloxifene B Oral sequential oestrogen + progestogen
C Oral continuous oestrogen + progestogen D Oestrogen patches
E Vaginal oestrogen tablets F Tamoxifen
G Levonorgestrel IUS H Refer to special oncology clinic
I Counsel and recommend discontinuation of HRT J HRT not recommended

Instructions: For each scenario described below, choose the single most appropriate
management from the above list of options. Each option may be used once, more than once,
or not at all.
A 50 year old woman is known to have anti-thrombin III deficiency. Your answer: A
Question 11 She complains of distressing hot flushes and vaginal dryness 6
months after her last menstrual period Correct answer: J

A 50 year old woman had been on sequential HRT for 6 months


prior to a diagnosis and treatment for breast cancer. She is still Your answer: A
having the occasional menstrual period but complains of
Question 12
worsening menopausal symptoms. She wishes to re-commence
HRT which was discontinued at the time of breast cancer Correct answer: H
diagnosis

| EXPLANATION |

Options for Questions 13-14


A Commence iv erythromycin B Commence oral co-amoxiclav
C Commence oral erythromycin D Corticosteroids + oral erythromycin
E Administer corticosteroids for fetal lung maturity F Administer corticosteroids + tocolytic
G Arrange in-utero transfer to tertiary centre H Arrange ex-utero transfer to tertiary centre
I Immediate caesarean section J Recommend bed-rest

Instructions: For each scenario described below, choose the single most appropriate
management from the above list of options. Each option may be used once, more than once,
or not at all.
A 24 year old primigravida presents at 30 weeks gestation with Your answer: A
painful uterine contractions every 3 minutes but no other
Question 13
symptoms. The cervix is partially effaced and the os is closed.
You have been informed that there are no SCBU cots in the unit. Correct answer: F

A 32 year old P 3+0 with two previous caesarean sections


presents with spontaneous rupture of the membranes at 32 weeks Your answer: A
Question 14 gestation. Her temperature is 37.8C, pulse = 110bpm and the
baseline fetal heart rate is 180bpm. The uterus is tender with mild Correct answer: I
contractions every 10 minutes.

| EXPLANATION |

Options for Questions 15-16


A Commence iv erythromycin B Commence oral co-amoxiclav
C Commence oral erythromycin D Treat with antenatal iv benzyl-penicillin
E Treat with antenatal oral benzyl-penicillin F No treatment required
G Arrange in-utero transfer to tertiary centre H Arrange ex-utero transfer to tertiary centre
I Insert rescue cervical suture J Recommend bed-rest

Instructions: For each scenario described below, choose the single most appropriate
management from the above list of options. Each option may be used once, more than once,
or not at all.
A 30 year old primigravida presents with spontaneous rupture of Your answer: A
the membranes at 28 weeks gestation. All maternal and fetal
Question 15
observations are within normal limits, there are no uterine
contractions and the cervix is not dilated Correct answer: C

A 32 year old woman presents with spontaneous rupture of the


membranes at 28 weeks gestation. All maternal and fetal Your answer: A
observations are within normal limits and despite tocolytic therapy,
Question 16
uterine contractions persist and the cervix is now 4cm dilated. The
local neonatal unit is unable to accommodate neonates born Correct answer: H
before 32 weeks

| EXPLANATION |

Options for Questions 17-18


A Commence iv erythromycin B Commence oral co-amoxiclav
C Commence oral erythromycin D Corticosteroids + oral erythromycin
E Administer corticosteroids for fetal lung maturity F No treatment required
G Arrange in-utero transfer to tertiary centre H Arrange ex-utero transfer to tertiary centre
I Insert rescue cervical suture J Recommend bed-rest

Instructions: For each scenario described below, choose the single most appropriate
management from the above list of options. Each option may be used once, more than once,
or not at all.
A 24 year old P4+0 with four previous term vaginal deliveries
presents at 27 weeks gestation with fresh vaginal bleeding and Your answer: A
uterine contractions every 3 minutes. The cervix is partially
Question 17
effaced and 3cm dilated with intact membranes. The placenta is
fundal and all other maternal and fetal observations are within Correct answer: E
normal limits. SCBU cots are available.

A 40 year woman with one previous pre-term delivery at 30 weeks


gestation presents with regular painful contractions at 32 weeks
gestation. Corticosteroids had been administered 5 days earlier Your answer: A
Question 18 when she presented with uterine contractions. The cervix is now
fully effaced and 5cm dilated with intact membranes. All other Correct answer: F
maternal and fetal observations are within normal limits and SCBU
cots are available.

| EXPLANATION |

Options for Questions 19-20


A Raloxifene B Oral sequential oestrogen + progestogen
C Oral continuous oestrogen + progestogen D Oestrogen patches
E Vaginal oestrogen tablets F Tamoxifen
G Levonorgestrel IUS H Refer to special oncology clinic
I Counsel and recommend discontinuation of HRT J HRT not recommended
Instructions: For each scenario described below, choose the single most appropriate
management from the above list of options. Each option may be used once, more than once,
or not at all.
Your answer: A
A 40 year old woman has TAH + BSO for stage Ia ovarian cancer
Question 19
and complains of distressing hot flushes and vaginal dryness
Correct answer: D

A 52 year old woman with a previous PE complains of infrequent Your answer: A


Question 20 periods and distressing hot flushes. Her sister died from PE while
on the COCP at the age of 38 years. Correct answer: J

Options for Questions 1-2


A Counsel and offer amniocentesis B Counsel and offer chorionic villus
sampling
C Counsel and offer pregnancy termination D Counsel and offer intra-uterine therapy
E Reassure F Counsel and offer fetal surgery

G Counsel and offer fetal blood sampling H Counsel and offer intra-uterine
transfusion

I Counsel and offer induction of labour at 36-38 weeks J Counsel and offer serial fetal growth
gestation scans
K Counsel and refer for neonatal assessment after delivery

Instructions: For each of the case histories described below, choose the single most
appropriate management from the above list of options. Each option may be used once, more
than once, or not at all.
A 20 year old woman is found to have a fetus with an isolated Your answer: A
Question 1 choroids plexus cyst at 21 weeks gestation. Integrated test
showed a risk of Downs syndrome of 1:5000 Correct answer: E

Your answer: A
A 20 year old woman is found to have a fetus with gastroschisis
Question 2
but no other anomalies at 21 weeks gestation.
Correct answer: J

| EXPLANATION |

Options for Questions 3-4


A Tranexamic acid B Hysteroscopy + polypectomy
C Danazol D Fibroid embolisation
E Mefenamic acid F GnRH analogues
G Trans-cervical resection of fibroid H Levonorgestrel IUS
I Combined oral contraceptive pill J Pipelle endometrial biopsy

Instructions: For each scenario described below, choose the single most appropriate
management from the above list of options. Each option may be used once, more than once,
or not at all.
A 17 year old woman complains of painful periods since menarche Your answer: A
Question 3 which are now interfering with her studies. She is not sexually
active and does not wish to take contraceptives. Correct answer: E
A 30 year old mother of 2 children complains of heavy regular Your answer: A
periods that were improved by cyclical progestogens. She has,
Question 4
however, discontinued treatment because of weight gain. She was
sterilised 2 years ago. Correct answer: H

| EXPLANATION |

Options for Questions 5-6


A Myomectomy B Hysteroscopy + polypectomy
C Danazol D Fibroid embolisation
E TAH + BSO F GnRH analogues
G Trans-cervical resection of fibroid H Tranexamic acid
I Combined oral contraceptive pill J Pipelle endometrial biopsy

Instructions: For each scenario described below, choose the single most appropriate
management from the above list of options. Each option may be used once, more than once,
or not at all.
A 45 year old mother of 4 children complains of progressively
heavy but regular periods. Clinical examination shows a 15 weeks Your answer: A
Question 5 size fibroid uterus and endometrial biopsy showed proliferative
endometrium. Medical therapy has failed but she does not wish to Correct answer: D
have major surgery

A 35 year old mother of 4 children complains of progressively Your answer: A


heavy periods. Over the last 2 months, she has bled severely and
Question 6
required admission for blood transfusion. She has been listed for
total abdominal hysterectomy in 3 months time Correct answer: F

| EXPLANATION |

Options for Questions 7-8


A Myomectomy B Hysteroscopy + polypectomy
C Danazol D Fibroid embolisation
E TAH + BSO F GnRH analogues
G Trans-cervical resection of fibroid H Levonorgestrel IUS
I Combined oral contraceptive pill J Pipelle endometrial biopsy

Instructions: For each scenario described below, choose the single most appropriate
management from the above list of options. Each option may be used once, more than once,
or not at all.
A 35 year old woman has been trying for a pregnancy for 12 Your answer: A
months and complains of heavy periods. Clinical examination and
Question 7
pelvic ultrasound scan confirm the presence of a 15 weeks size
fibroid uterus but no other abnormalities Correct answer: A

A 30 year old woman complains of progressively heavy periods Your answer: A


over 2 years. Pelvic ultrasound scan shows a slightly enlarged
Question 8
uterus with multiple small fibroids. The endometrium is distorted
by a sub-mucous fibroid Correct answer: G

| EXPLANATION |
Options for Questions 9-10
A Sequential combined HRT B Hysteroscopy + polypectomy
C Danazol D Fibroid embolisation
E TAH + BSO F GnRH analogues
G Trans-cervical resection of fibroid H Levonorgestrel IUS
I Combined oral contraceptive pill J Pipelle endometrial biopsy

Instructions: For each scenario described below, choose the single most appropriate
management from the above list of options. Each option may be used once, more than once,
or not at all.
A 30 year old woman complains of a 2 year history of
progressively heavy periods. She was on the oral contraceptive Your answer: A
Question 9 pill until 3 years ago when she developed a DVT during
pregnancy. Clinical examination is normal and she does not wish Correct answer: H
to take tablets.

A 43 year old mother of 5 children complains of a 5 year history of


progressively heavy periods that have failed to respond to medical Your answer: A
Question 10 treatment. Trans-vaginal ultrasound scan performed in the
proliferative phase shows a normal pelvis with a regular 14mm Correct answer: J
endometrium

| EXPLANATION |

Options for Questions 11-12


A Counsel and offer amniocentesis B Counsel and offer chorionic villus
sampling
C Counsel and offer pregnancy termination D Counsel and offer intra-uterine therapy
E Reassure F Counsel and offer fetal surgery

G Counsel and offer fetal blood sampling H Counsel and offer intra-uterine
transfusion

I Counsel and offer induction of labour at 36-38 weeks J Counsel and offer serial fetal growth
gestation scans
K Counsel and refer for neonatal assessment after delivery L Counsel and offer fetal echocardiography

Instructions: For each of the case histories described below, choose the single most
appropriate management from the above list of options. Each option may be used once, more
than once, or not at all.
Your answer: A
A 35 year old woman is found to have a fetus with a cleft lip and
Question 11
palate on anomaly scan at 20 weeks gestation
Correct answer: A

A 42 year old woman has had amniocentesis with a normal Your answer: A
Question 12 karyotype at 16 weeks gestation. The fetus is found to have
talipes equinovarus on anomaly scan at 20 weeks gestation Correct answer: K

| EXPLANATION |

Options for Questions 13-14


A Sequential combined HRT B Hysteroscopy + polypectomy
C Danazol D Fibroid embolisation
E TAH + BSO F GnRH analogues
G Trans-cervical resection of fibroid H Tranexamic acid
I Combined oral contraceptive pill J Hysteroscopy D&C

Instructions: For each scenario described below, choose the single most appropriate
management from the above list of options. Each option may be used once, more than once,
or not at all.
A 48 year old woman complains of a 6 months history of Your answer: A
progressively heavy and irregular periods (occurring every 4-7
Question 13
weeks) with occasional hot flushes. Trans-vaginal ultrasound scan
shows a normal pelvis with a 7mm regular endometrium Correct answer: A

A 48 year old woman complains of continuous vaginal bleeding Your answer: A


over the last 6 months, varying from slight staining to heavy
Question 14
period-like bleeding. Trans-vaginal ultrasound scan shows a
normal pelvis with a 7mm regular endometrium Correct answer: J

| EXPLANATION |

Options for Questions 15-16


A Sequential combined HRT B Hysteroscopy + polypectomy
C Danazol D Fibroid embolisation
E TAH + BSO F GnRH analogues
G Trans-cervical resection of fibroid H Mefenamic acid
I Combined oral contraceptive pill J Hysteroscopy D&C

Instructions: For each scenario described below, choose the single most appropriate
management from the above list of options. Each option may be used once, more than once,
or not at all.
A 30 year old woman complains of a 2 year history of Your answer: A
progressively heavy periods. She was on the oral contraceptive
Question 15
pill until 3 years ago when she developed a DVT during
pregnancy. Clinical examination is normal. Correct answer: H

A 30 year old nulliparous woman is referred to the gynaecology Your answer: A


clinic because of progressively heavy periods. She has been
Question 16
treated with oral progestogens, mefenamic acid and tranexamic
acid without improvement. Correct answer: J

| EXPLANATION |

Options for Questions 17-18


A Counsel and offer amniocentesis B Counsel and offer chorionic villus
sampling
C Counsel and offer pregnancy termination D Counsel and offer intra-uterine therapy
E Reassure F Counsel and offer fetal surgery

G Counsel and offer fetal blood sampling H Counsel and offer intra-uterine
transfusion

I Counsel and offer induction of labour at 36-38 weeks J Counsel and offer serial fetal growth
gestation scans
K Counsel and refer for neonatal assessment after delivery L Counsel and offer fetal echocardiography

Instructions: For each of the case histories described below, choose the single most
appropriate management from the above list of options. Each option may be used once, more
than once, or not at all.
Your answer: A
A 35 year old woman with phenylketonuria presents for anomaly
Question 17
scan at 21 weeks gestation.
Correct answer: L

Your answer: A
A 20 year old woman with a history of tetralogy of Fallot corrected
Question 18
in childhood is referred for antenatal care at 10 weeks gestation.
Correct answer: D

| EXPLANATION |

Options for Questions 19-20


A Counsel and offer amniocentesis B Counsel and offer chorionic villus
sampling
C Counsel and offer pregnancy termination D Counsel and offer intra-uterine therapy
E Reassure F Counsel and offer fetal surgery

G Counsel and offer fetal blood sampling H Counsel and offer intra-uterine
transfusion

I Counsel and offer induction of labour at 36-38 weeks J Counsel and offer serial fetal growth
gestation scans
K Counsel and refer for neonatal assessment after delivery

Instructions: For each of the case histories described below, choose the single most
appropriate management from the above list of options. Each option may be used once, more
than once, or not at all.
Your answer: A
A 20 year old woman is found to have a fetus with an exomphalos
Question 19
at 21 weeks gestation
Correct answer: A

Your answer: A
A 30 year old Rhesus negative woman with Rhesus D antibodies
Question 20
is found to have a hydropic fetus at 26 weeks gestation
Correct answer: H
Options for Questions 1-2
A Chorionic villus sampling B Amniocentesis
C Fetal blood sampling D Triple test
E Karyotype F Integrated test
G Western blotting H Northern blotting
I Anomaly scan J Southern blotting
K PCR

Instructions: For each of the case histories described below, choose the single most relevant
investigation from the above list of options. Each option may be used once, more than once,
or not at all.
Your answer: A
A 20 year old HIV positive woman presents for antenatal care at
Question 1
20 weeks gestation
Correct answer: I

A 35 year old woman is known to be a balanced 14:21 Your answer: A


Question 2 translocation carrier and has undergone amniocentesis at 16
weeks gestation Correct answer: E
| EXPLANATION |

Options for Questions 3-4


A Le Fort procedure B Boari flap procedure
C Catheterisation D Short bladder distension
E Urethral dilatation F Peri-urethral collagen injection
G Urodynamic studies H Cystoscopy
I Anterior colporrhaphy J Pelvic floor physiotherapy
K Sacro-colpopexy L Long-term intermittent self-catheterisation

Instructions: For each scenario described below, choose the single most appropriate
management from the above list of options. Each option may be used once, more than once,
or not at all.
A 35 year old woman re-attends 6 days after TAH for menorrhagia
because of persistent leakage of clear fluid from the vagina. Your answer: A
Question 3 Speculum examination showed the presence of urine in the
vagina and a three swab test showed the presence of dye in the Correct answer: C
vagina.

A healthy 57 years old woman complains of a vaginal lump 20 Your answer: A


Question 4 years after total anbdominal hysterectomy for a large fibroid
uterus. Clinical examination showed marked vault prolapse. Correct answer: K

| EXPLANATION |

Options for Questions 5-6


A Duloxetine B Tolterodine
C Amitriptyline D Short bladder distension
E Urethral dilatation F Tolterodine + bladder re-training
G Urodynamic studies H Cystoscopy
I Prolonged bladder distension J Pelvic floor physiotherapy
K Solifenacine L Long-term intermittent self-catheterisation

Instructions: For each scenario described below, choose the single most appropriate
management from the above list of options. Each option may be used once, more than once,
or not at all.
A 33 year old woman complains of debilitating urinary frequency, Your answer: A
urgency and urge incontinence but no other symptoms. Clinical
Question 5
examination is normal and urine microscopy and culture are
negative. Correct answer: F

A 34 year old nulliparous woman complains of urinary frequency, Your answer: A


Question 6 urgency, deep dyspareunia and bladder pain. Urine dipstix shows
persistent haematuria but culture is negative. Correct answer: H

| EXPLANATION |

Options for Questions 7-8


A Cystoscopy and cystodistension B Bladder re-training
C Tension-free vaginal tape D Anterior repair + TVT
E Posterior repair F Paravaginal repair
G Para-urethral collagen injection H Hysterectomy
I Pelvic floor physiotherapy J Burch colposuspension
K Intermittent self-catherterisation L Oestrogen replacement therapy

Instructions: For each of the case histories described below, choose the single most
appropriate treatment option from the above list of options. Each option may be used once,
more than once, or not at all.
A 45 year old woman complains of urinary frequency, urgency and Your answer: A
Question 7 urge incontinence. Clinical examination showed an 18 weeks size
fibroid uterus with a small cystocele but no rectocele Correct answer: H

A 45 year old woman remains in hospital 8 days after Burch Your answer: A
colposuspension because she is unable to empty her bladder
Question 8
spontaneously. The supre-pubic catheter has been removed and
she now has a urethral catheter Correct answer: K

| EXPLANATION |

Options for Questions 9-10


A Cystoscopy B MRI scan
C Urodynamic studies D Bladder re-training
E Urine for cytology F Cystoscopy and biopsy
G Examination under anaesthesia H MSU for culture and sensitivity
I Urine electrolytes J Renal function tests

Instructions: For each of the case histories described below, choose the single most
important investigation from the above list of options. Each option may be used once, more
than once, or not at all.
A 60 year old woman developed cervical cancer 12 months earlier Your answer: A
Question 9 and was treated with combined chemotherapy and radiotherapy.
She now complains of blood in her urine which occurs all the time Correct answer: F

A 45 year old woman complained initially of urinary frequency


urgency and urge incontinence and also leaked urine on coughing Your answer: A
or straining. She was treated medically initially and her symptoms
Question 10
of urinary frequency and urgency have largely resolved. Over the
last 6 months, she is finding that urinary leakage on coughing and Correct answer: C
straining is increasingly affecting her social life.

| EXPLANATION |

Options for Questions 11-12


A Congenital syphilis B Congenital varicella syndrome
C Parvovirus B19 infection D Turners syndrome
E Parder-Willy syndrome F Fetal hydantoin syndrome
G Group B streptococcal infection H Congenital rubella syndrome
I Cri-du-chat syndrome J Edwards syndrome
K Pataus syndrome L Downs syndrome

Instructions: For each of the case histories described below, choose the single most likely
cause of fetal abnormality from the above list of options. Each option may be used once,
more than once, or not at all.
A 42 year old woman is late booking for antenatal care. The Your answer: A
estimated gestation age by ultrasound scan is 22 weeks and the
Question 11
fetus is found to have an atrio-ventricular septal defect and a
double bubble sign in the upper abdomen Correct answer: L

A 42 year old woman had bleeding in early pregnancy and was Your answer: A
found to have an 8 week viable pregnancy. Anomaly scan at 22
Question 12
weeks showed a head and abdominal circumferences below the
third centile , bilateral choroids plexus cysts and rockerbottom feet Correct answer: J

| EXPLANATION |

Options for Questions 13-14


A Stress incontinence B Urinary retention with overflow
C Urinary retention D Detrusor instability
E Detrusor hyper-reflexia F Interstitial cystitis
G Genuine stress incontinence H Mixed urinary incontinence
I Bladder tumour J Urinary tract infection
K Diabetes mellitus

Instructions: For each of the case histories described below, choose the single most likely
diagnosis from the above list of options. Each option may be used once, more than once, or
not at all.
A 30 year old woman complains of severe lower abdominal pain 3
days after hysterectomy. The urethral catheter had been removed Your answer: A
Question 13 12h earlier and she has voided spontaneously on 2 occasions
(total volume 75ml). Clinical examination shows a 14 week size Correct answer: C
tender supra-pubic mass.

Your answer: A
A 35 year old woman with multiple sclerosis complains of urinary
Question 14
frequency, urgency and urge incontinence
Correct answer: E

| EXPLANATION |

Options for Questions 15-16


A Amniocentesis and karyotype B Amniocentesis and PCR
C CVS and karyotype D CVS and PCR
E Amniocentesis and enzyme assay F Fetal blood sampling and karyotype

G Fetal blood sampling and measurement of PO2 H Fetal blood sampling and haemoglobin
electrophoresis

I Fetal blood sampling and haemoglobin J Fetal blood sampling and viral culture
concentration
K Reassurance

Instructions: For each of the case histories described below, choose the single most
appropriate management from the above list of options. Each option may be used once, more
than once, or not at all.
A 30 year old woman with sickle cell disease is concerned about Your answer: A
Question 15 passing the disease onto her child. On haemoglobin
electrophoresis, her partner is found to have haemoglobin AA. Correct answer: K
A 30 year old woman had amniocentesis because of a high risk of
Downs syndrome on serum screening. The karyotype is reported Your answer: A
as 46XX. She is now 20 weeks pregnant and has discovered that
Question 16
she is a carrier of Duchenne muscular dystrophy after her sisters
son was diagnosed with the condition. She is worried about Correct answer: K
passing the condition to her child.

| EXPLANATION |

Options for Questions 17-18


A Autosomal dominant B Autosomal recessive
C X-linked recessive D X-linked dominant
E Autosomal dominant with incomplete penetrance F Autosomal dominant with variable expressivity
G Polygeneic transmission H Confined placental mosaicism
I Non-mendelian inheritance J Inheritance through mitochondrial DNA
K X-linked recessive with lyonisation L X-linked dominant with lyonisation

Instructions: For each of the case histories described below, choose the single most likely
mode of inheritance from the above list of options. Each option may be used once, more
than once, or not at all.
A 25 year old woman has been referred for pre-natal diagnosis.
Her father and brother suffer from an inherited condition. She is Your answer: A
Question 17 found to be mildly affected by the condition and will require
assessment and treatment during labour. She is found to be Correct answer: K
carrying a female fetus and has been reassured.

A 25 year old primigravida is married to her first cousin. At 20 Your answer: A


Question 18 weeks gestation, anomaly scan shows multiple fetal abnormalities.
Both parents have been found to have normal karyotypes. Correct answer: B

| EXPLANATION |

Options for Questions 19-20


A Autosomal dominant B Autosomal recessive
C X-linked recessive D X-linked dominant
E Autosomal dominant with incomplete penetrance F Autosomal dominant with variable expressivity
G Polygeneic transmission H Confined placental mosaicism
I Non-mendelian inheritance J Inheritance through mitochondrial DNA
K X-linked recessive with lyonisation L X-linked dominant with lyonisation

Instructions: For each of the case histories described below, choose the single most likely
mode of inheritance from the above list of options. Each option may be used once, more
than once, or not at all.
A 25 year old woman is known to be mildly affected by an
inherited condition. Her father suffered from a severe form of the Your answer: A
Question 19 same condition and died at the age 30 years. She has been found
to have a female fetus with a severe form of the condition and is Correct answer: F
requesting termination of pregnancy

A 25 year old woman has been referred for pre-natal diagnosis. Your answer: A
Question 20 Her father and brother suffer from an inherited condition. She is
found to be carrying a female fetus and has been reassured. Correct answer: C
| EXPLANATION |

Options for Questions 21-22


A Rocker bottom feet B Exomphalos
C Holoprosencephaly D Cystic hygroma
E Gastroschisis F atrio-ventricular septal defect
G Intra-cranial calcification H Duodenal atresia
I Spina bifida J Polyhydramnios
K Echogenic bowel

Instructions: For each of the conditions described below, choose the single most
characteristic ultrasound marker from the above list of options. Each option may be used
once, more than once, or not at all.
Your answer: A
Question 21 Trisomy 18
Correct answer: A

Your answer: A
Question 22 Cystic fibrosis
Correct answer: K

| EXPLANATION |

Options for Questions 23-24


A Amniocentesis B Chorionic villus sampling
C Fetal blood sampling D Triple test
E Quadruple test F Integrated test
G Serum integrated test H Nuchal transluscency
I Anomaly scan J Fluorescent in-situ hybridisation
K PCR L Restriction fragment length polymorphisms

Instructions: For each of the case histories described below, choose the single most
appropriate investigation from the above list of options. Each option may be used once,
more than once, or not at all.
A 17 year old woman books for antenatal care at 16 weeks Your answer: A
Question 23 gestation and would like to know whether or not her baby has
Downs syndrome Correct answer: E

A 30 year old woman is known to be a carrier of Haemophilia A. Your answer: A


Question 24 Amniocentesis at 16 weeks gestation shows that she is carrying
an unaffected male fetus. She is now 20 weeks pregnant. Correct answer: I

| EXPLANATION |

Options for Questions 25-26


A Chorionic villus sampling B Amniocentesis
C Fetal blood sampling D Triple test
E Karyotype F Integrated test
G Western blotting H Northern blotting
I Anomaly scan J Southern blotting
K PCR

Instructions: For each of the case histories described below, choose the single most relevant
investigation from the above list of options. Each option may be used once, more than once,
or not at all.
A 35 year old woman with myotonic dystrophy is undergoing IVF Your answer: A
Question 25 treatment in order to ensure that the disease is not passed on to
her child Correct answer: K

A 35 year old woman is found to have raised maternal serum AFP Your answer: A
Question 26 at 16 weeks gestation. Anomaly scan is equivocal because of high
body mass index and she has opted to have amniocentesis Correct answer: G

| EXPLANATION |

Options for Questions 27-28


A Risk of Downs syndrome of ~1:10 B Risk of culture failure

C Risk of miscarriage of 0.5-1% above D Risk of miscarriage of 1-2% above background


background
E Risk of fetal death F Risk of confined placental mosaicism

G Risk of contamination with maternal blood H Risk of fetal bradycardia requiring emergency caesarean
section
I Risk of Rhesus iso-immunisation J Risk of trisomy 18
K Risk of infection L Risk of fetal needle injury

Instructions: Instructions: For each of the case histories described below, choose the single
most relevant complication that you must discuss with the patient when taking consent prior
to invasive testing from the above list of options. Each option may be used once, more than
once, or not at all.
A 42 year old primigravida is found to have a risk of having a baby Your answer: A
Question 27 with Downs syndrome of 1:10 following integrated testing and is
scheduled to undergo amniocentesis at 17 weeks gestation Correct answer: C

A 42 year old woman with a previous baby with cystic fibrosis. She Your answer: A
and her partner have both been found to be carriers of the cystic
Question 28
fibrosis gene mutation and she is scheduled to undergo chorionic
villus sampling at 11 weeks gestation Correct answer: D

| EXPLANATION |

Options for Questions 29-30


A Amniocentesis B Chorionic villus sampling
C Fetal blood sampling D Triple test
E Quadruple test F Integrated test
G Serum integrated test H Nuchal transluscency
I Anomaly scan J Fluorescent in-situ hybridisation
K PCR L Restriction fragment length polymorphisms

Instructions: For each of the case histories described below, choose the single most
appropriate investigation from the above list of options. Each option may be used once,
more than once, or not at all.
A 30 year old Rhesus positive woman is found to have a hydropic Your answer: A
Question 29 fetus at 22 weeks gestation. No other anomalies have been
identified on detailed ultrasound examination. Correct answer: C

A 30 year old woman is found to have a high risk of having a Your answer: A
Question 30 baby with Downs syndrome following serum screening.
Amniocentesis has been performed at 18 weeks gestation. Correct answer: J

| EXPLANATION |

Options for Questions 31-32


A Fetal blood sampling B Triple test
C Quadruple test D Integrated test
E Chorionic villus sampling F Serum integrated test
G Amniocentesis H Nuchal transluscency
I Anomaly scan J Fluorescent in-situ hybridisation
K PCR L Restriction fragment length polymorphisms
M Karyotype

Instructions: For each of the conditions described below, choose the single most appropriate
diagnostic tests from the above list of options. Each option may be used once, more than
once, or not at all.
Your answer: A
Question 31 Fragile X-syndrome
Correct answer: M

Your answer: A
Question 32 Downs syndrome
Correct answer: M

| EXPLANATION |

Options for Questions 33-34


A Fetal blood sampling B Triple test
C Quadruple test D Integrated test
E Chorionic villus sampling F Serum integrated test
G Amniocentesis H Nuchal transluscency
I Anomaly scan J Fluorescent in-situ hybridisation
K PCR L Restriction fragment length polymorphisms
M Karyotype

Instructions: For each of the conditions described below, choose the single most appropriate
diagnostic tests from the above list of options. Each option may be used once, more than
once, or not at all.
Your answer: A
Question 33 Fetal anaemia
Correct answer: A

Question 34 Fetal gastroschisis Your answer: A


Correct answer: I

| EXPLANATION |

Options for Questions 35-36


A Cystoscopy and cystodistension B Bladder re-training
C Tension-free vaginal tape D Anterior repair
E Posterior repair F Paravaginal repair
G Para-urethral collagen injection H Tolterodine + bladder re-training
I Solifenacin J Duloxetine
K Amitriptyline L Oestrogen replacement therapy

Instructions: For each of the case histories described below, choose the single most
appropriate treatment option from the above list of options. Each option may be used once,
more than once, or not at all.
A 65 year old mother of 6 children presents with a 3 year history of Your answer: A
Question 35 urinary frequency, leakage on coughing and straining. Urodynamic
investigations confirm a diagnosis of genuine stress incontinence Correct answer: C

Your answer: A
A 35 year old woman complains of urinary frequency, urgency and
Question 36
urge incontinence but no other symptoms.
Correct answer: H

| EXPLANATION |

Options for Questions 37-38


A Abdominal pressure due to mass B Bladder neck weakness
C Pelvic floor muscle damage D Congenital weakness of the pelvic floor
E Obstetric trauma F Oestrogen deficiency
G Neurological disease H Drug side-effects
I Tumour infiltration into bladder J Bacterial urinary tract infection
K Schistosomiasis L Radiation injury

Instructions: For each of the case histories described below, choose the single most likely
cause of urinary tract symptoms from the above list of options. Each option may be used
once, more than once, or not at all.
A 35 year old woman complains of a 2 month history of urinary
frequency, urgency and haematuria. She had an abdominal X-ray Your answer: A
Question 37 2 weeks earlier for an unrelated complaint which showed evidence
of bladder calcification. Cystoscopy showed numerous polypoid Correct answer: K
lesions in the bladder

A 52 year old woman complains of progressively worsening Your answer: A


urinary frequency and urgency 3 years after radical abdominal
Question 38
hysterectomy and radiotherapy for cervical cancer. There is no
haematuria and all investigations including cystoscopy are normal Correct answer: F

| EXPLANATION |

Options for Questions 39-40


A Cystoscopy & cystodistension B Cystoscopy
C Urine microscopy, culture and sensitivity D Cystometry
E Pelvic ultrasound scan F Colposuspension
G Bladder retraining H Three swab test
I Intra-venous urogram J Return to theatre
K Catheterise L Pelvic floor exercises

Instructions: For each of the case histories described below, choose the single most
appropriate initial management from the above list of options. Each option may be used
once, more than once, or not at all.
A 60 year old woman complains of urinary frequency and urgency Your answer: A
Question 39 for 9 months and the presence of blood in her urine over the last 7
days Correct answer: B

A 35 year old woman complains of feeling wet all the time 10 days Your answer: A
Question 40 after an uneventful colposuspension. Her initial symptoms of
urinary leakage on coughing have improved significantly. Correct answer: H

| EXPLANATION |

Options for Questions 1-2


A Offer hepatitis B passive immunisation to B Offer hepatitis B active immunisation to woman
woman
C Deliver by caesarean section D Amniocentesis for fetal viraemia by PCR

E Fetal blood sampling for viral culture F Offer hepatitis B passive immunisation to neonate from
day 7

G Offer hepatitis B active immunisation to H Offer hepatitis B active & passive immunisation to
neonate neonate
I Avoid breast-feeding J Intra-venous acyclovir in labour
K Reassurance L Screen for hepatitis B infection

Instructions: For each scenario described below, choose the single most appropriate
management from the above list of options. Each option may be used once, more than once,
or not at all.
A 30 year old woman has hepatitis B screening at 14 weeks Your answer: A
Question 1 gestation with the following results: Hep B surface antigen
negative, antibodies to Hep B surface antigen negative Correct answer: K

A 25 year old woman with a history of hepatitis B infection Your answer: A


Question 2 presents in spontaneous labour at 39 weeks gestation. The cervix
is 2cm dilated with intact membranes. Correct answer: L

| EXPLANATION |

Options for Questions 3-4


A Avoid breast-feeding B Administer VZIG to neonate and avoid breast-
feeding
C Administer VZIG to neonate but breast-feeding D Immediate induction of labour
encouraged
E Delay delivery with tocolytics F Offer termination of pregnancy
G Treat neonate with oral acyclovir H Treat neonate with intravenous acyclovir

I Examine neonate for evidence of congenital J Reassurance


anomalies
K Delay delivery for 5-7 days if possible L Delay discharge for at least 7 days

Instructions: For each scenario described below, choose the single most appropriate
management from the above list of options. Each option may be used once, more than once,
or not at all.
A 30 year old woman has delivered her second child by
caesarean section for breech presentation. While she is in Your answer: A
Question 3 hospital, she is informed that her 5 year old daughter has
chickenpox. She has had chicken pox in the past but is concerned Correct answer: J
about taking her baby home

A 20 year old woman is in hospital 2 days after vaginal delivery of


her first baby and is informed that her 8 year old step-daughter Your answer: A
has chicken-pox. She does not remember having chicken pox in
Question 4
the past and is worried about taking her new baby home.
Investigations show that she is varicella zoster IgG negative and Correct answer: L
IgM negative

| EXPLANATION |

Options for Questions 5-6


A Endometriosis B Acute PID
C Chronic PID D Irritable bowel syndrome
E Residual ovary syndrome F Ovarian remnant syndrome
G Diverticular disease H Interstitial cystitis
I Primary dysmenorrhoea J Adenomyosis
K Uterine fibroids L Polycystic ovary syndrome
M Inflammatory bowel disease N Chronic constipation

Instructions: For each of the case histories described below, choose the single most likely
cause of chronic pelvic pain from the above list. Each option may be used once, more than
once, or not at all.
A 34 year old mother of 3 children with a 10 year menorrhagia
underwent a total abdominal hysterectomy with conservation of Your answer: A
the ovaries 2 years earlier. She now complains of persistent pelvic
Question 5
pain that is worse when her period would have been due and
deep dyspareunia. Pelvic ultrasound scan shows small bilateral Correct answer: E
ovarian cysts 2-3cm in diameter but no other abnormalities.

A 35 year old woman complains of a 10 months history of pelvic


pain, dysmenorrhoea and deep dyspareunia since discontinuing
the combined oral contraceptive pill. She also complains of rectal Your answer: A
Question 6 bleeding during menstruation but has a regular bowel habit and
there is no recent weight loss. Clinical examination showed a Correct answer: A
bulky tender uterus with decreased mobility and palpable nodules
within the pouch of Douglas. Rectal examination was normal.

| EXPLANATION |

Options for Questions 7-8


A Tranexamic acid B Danazol
C GnRH analogues D Laparoscopy and ablation of endometriosis
E Diagnostic laparoscopy F Mefenamic acid
G Pelvic ultrasound scan H MRI scan of the pelvis
I Dilatation and curettage J Endocervical and urethral swabs
K Combined oral contraceptive pill L Depo-medroxyprogesterone acetate
M Refer to pain clinic N Refer to psychiatrist

Instructions: For each of the case histories described below, choose the single most
appropriate management from the above list. Each option may be used once, more than
once, or not at all.
A 35 year old woman attends the gynaecology clinic with her 14
year old daughter who is complaining of severe period pains since
the onset of menstruation at the age of 13 years. Menstrual loss is Your answer: A
Question 7 not heavy and she has no bowel symptoms. Her symptoms are
now interfering with her education as she takes several days off Correct answer: F
school every month. She does not require contraception and
clinical examination is normal.

A 33 year old mother of 3 children complains of progressively


painful periods over 5 years. There is occasional deep Your answer: A
Question 8 dyspareunia but she has no bowel symptoms. Her symptoms
have not responded to simple analgesia and non-steroidal anti- Correct answer: K
inflammatory drugs. Clinical examination is normal.

| EXPLANATION |

Options for Questions 9-10


A Maternal treatment with antibiotics B Detailed fetal anomaly scan
C Deliver by caesarean section D Active immunisation of neonate
E Avoid breast-feeding F Counsel and offer termination of pregnancy
G Delay delivery by 5-7 days H Administer corticosteroids
I Administer tocolytics J Reassurance
K Screen for maternal syphilis infection

Instructions: For each scenario described below, choose the single most appropriate
management from the above list of options. Each option may be used once, more than once,
or not at all.
A 20 year old woman presents with intermittent abdominal pain Your answer: A
Question 9 and a febrile illness at 20 weeks gestation. Blood cultures are
positive for Listeria monocytogenes Correct answer: A

A 30 year old woman has the following results following routine Your answer: A
Question 10 antenatal screening at 15 weeks gestation: VDRL positive, FTA-
abs positive Correct answer: A

| EXPLANATION |

Options for Questions 11-12


A Counsel and offer termination of pregnancy B Counsel and offer rubella immune globulin
C Treat with intravenous zidovudine D Counsel and offer intra-uterine therapy
E Offer amniocentesis for karyotype F Reassurance
G Immunise neonate against rubella H Immunise mother against rubella
I Treat with intravenous acyclovir J Amniocentesis and viral culture
K Amniocentesis and PCR for fetal viraemia

Instructions: For each scenario described below, choose the single most appropriate
management from the above list of options. Each option may be used once, more than once,
or not at all.
A 20 year old student has been immunised against Rubella 2
weeks before travelling to the UK. She attends as an emergency Your answer: A
Question 11 because she has missed a period and is found to be 10 weeks
pregnant on ultrasound scan. She is concerned about the effect of Correct answer: F
the vaccine on her fetus

A 30 year old school teacher is 32 weeks pregnant and is


concerned because she has been in contact with a child who has Your answer: A
Question 12 been diagnosed with rubella. Antenatal screening had revealed
low antibody titres and post-partum immunisation had been Correct answer: F
recommended.

| EXPLANATION |

Options for Questions 13-14


A Endometriosis B Acute PID
C Chronic PID D Irritable bowel syndrome
E Residual ovary syndrome F Ovarian remnant syndrome
G Diverticular disease H Interstitial cystitis
I Primary dysmenorrhoea J Adenomyosis
K Uterine fibroids L Polycystic ovary syndrome
M Inflammatory bowel disease N Chronic constipation

Instructions: For each of the case histories described below, choose the single most likely
cause of chronic pelvic pain from the above list. Each option may be used once, more than
once, or not at all.
A 23 year old woman has been referred to the gynaecology clinic
with a 6 months history of lower abdominal and pelvic pain and
deep dyspareunia. Her symptoms are particularly worse during Your answer: A
Question 13 menstruation. Over the last 4 months, she has experienced
episodes of bloody diarrhoea and has lost about 5kg in weight. Correct answer: M
Her Hb = 10.0g/dl, white cell count = 15 x 10E9/l and platelet
count = 450 x 10E9/l.

A 35 year old woman complains of a 2 year history of


progressively painful periods with occasional deep pain during
intercourse. She denies any bowel or urinary symptoms. Clinical Your answer: A
Question 14 examination showed a bulky smooth and tender uterus with no
adnexal masses and no adnexal tenderness. Her symptoms have Correct answer: J
not responded to non-steroidal anti-inflammatory agents and
diagnostic laparoscopy was normal.

| EXPLANATION |

Options for Questions 15-16


A Tranexamic acid B Danazol
C GnRH analogues D TAH + BSO
E Diagnostic laparoscopy F Medroxyprogesterone acetate
G Pelvic ultrasound scan H MRI scan of the pelvis
I Dilatation and curettage J Endocervical and urethral swabs
K Combined oral contraceptive pill L Levonorgestrel releasing intra-uterine system
M Refer to pain clinic N Refer to gastroenterologist

Instructions: For each of the case histories described below, choose the single most
appropriate management from the above list. Each option may be used once, more than
once, or not at all.
A 22 year old woman is known to have endometriosis diagnosed
at laparoscopy. She is referred to the gynaecology clinic because Your answer: A
of progressively severe pelvic pain, dysmenorrhoea and deep
Question 15
dyspareunia. There is a past medical history of focal migraine.
Clinical examination showed a tender pelvis with a normal size Correct answer: F
anteverted uterus and no adnexal masses.

A 22 year old woman is referred to the gynaecology clinic because


of a 2 year history of persistent pelvic pain which is worse during
and after intercourse and during menstruation. She also
complains of intermittent abdominal bloating with alternating Your answer: A
Question 16 constipation and diarrhoea and urgency of defecation. There is
occasional rectal bleeding which is unrelated to her menses and Correct answer: N
usually occurs during episodes of constipation. Clinical
examination is normal and there is no significant past medical
history.

| EXPLANATION |

Options for Questions 17-18


A Refer to surgical team B Danazol
C GnRH analogues D TAH + BSO
E Diagnostic laparoscopy F Medroxyprogesterone acetate
G Pelvic ultrasound scan H MRI scan of the pelvis
I Dilatation and curettage J Endocervical and urethral swabs
K Combined oral contraceptive pill L Levonorgestrel releasing intra-uterine system
M Refer to pain clinic N Refer to gastroenterologist

Instructions: For each of the case histories described below, choose the single most
appropriate management from the above list. Each option may be used once, more than
once, or not at all.
A 78 year old woman has been investigated by the gynaecologists
over a 2 year period for persistent pelvic pain and was admitted
overnight because of severe pain with nausea and vomiting. Her
LMP was over 20 years ago and there is no post-menopausal Your answer: A
Question 17 bleeding. On admission, she had a temperature of 38C, pulse
110bpm with marked left iliac fossa tendernedd. Her CRP and Correct answer: A
white cell count are elevated and pelvic ultrasound scan shows a
normal size uterus with normal ovaries and a 7cm complex mass
adjacent to the left ovary.

A 34 year old woman with severe endometriosis which has not


responded to the combined oral contraceptive pill and Your answer: A
Question 18 progestogens has undergone laparoscopy with laser ablation of
endometriosis. Six months later, she complains of a relapse with Correct answer: C
severe pelvic pain, dysmenorrhoea and deep dyspareunia
| EXPLANATION |

Options for Questions 19-20


A Tranexamic acid B Danazol
C GnRH analogues D TAH + BSO
E Diagnostic laparoscopy F Mefenamic acid
G Pelvic ultrasound scan H MRI scan of the pelvis
I Dilatation and curettage J Endocervical and urethral swabs
K Combined oral contraceptive pill L Levonorgestrel releasing intra-uterine system
M Refer to pain clinic N Refer to psychiatrist

Instructions: For each of the case histories described below, choose the single most
appropriate management from the above list. Each option may be used once, more than
once, or not at all.
A 37 year old woman complains of severe pain during
menstruation over the last 12 months. She has a regular 28 day Your answer: A
cycle and denies any pelvic or abdominal pain at other times. She
Question 19
has no bowel symptoms. She had been taking the combined oral
contraceptive pill until 12 months ago when it was discontinued Correct answer: F
because she is a smoker. Clinical examination is normal.

A 38 year old woman complains of a 12 months history of severe


pelvic pain, deep pain during sexual intercourse and pain on
defecation. Her symptoms are particularly worse during Your answer: A
Question 20 menstruation but are present at other times and have not
responded to simple analgesia. Clinical examination showed a Correct answer: E
markedly tender pelvis with a fixed retroverted uterus but no
adnexal masses.

| EXPLANATION |

Options for Questions 21-22


A Counsel and offer termination of pregnancy B Counsel and offer rubella immune globulin
C Treat with intravenous zidovudine D Counsel and offer intra-uterine therapy
E Offer amniocentesis for karyotype F Reassurance
G Immunise neonate against rubella H Immunise mother against rubella
I Treat with intravenous acyclovir J Amniocentesis and viral culture
K Amniocentesis and PCR for fetal viraemia

Instructions: For each scenario described below, choose the single most appropriate
management from the above list of options. Each option may be used once, more than once,
or not at all.
A 30 year old woman is known to be rubella immune but is Your answer: A
Question 21 concerned about taking her 3 day old baby home because her 10
year old step-daughter has rubella Correct answer: F

A 20 year old woman has returned from holiday with a rash which Your answer: A
is subsequently diagnosed as rubella. Following resolution of her
Question 22
symptoms, she complains of missing a period 2 weeks later and is
found to be 11 weeks pregnant on ultrasound scan Correct answer: A

| EXPLANATION |
Options for Questions 23-24
A Administer VZIG as soon as possible to mother B Administer VZIG to mother if maternal serology -
ve
C Administer VZIG to neonate D Detailed ultrasound examination

E Immediate caesarean section and transfer baby to F Advise avoid contact with other pregnant women
the neonatal unit and neonates
G Induction of labour H Reassurance
I Separate mother and baby after delivery J Serum for VZV IgM antibodies
K Give intravenous Aciclovir L Treat with oral Aciclovir

Instructions: For each scenario described below, choose the single most appropriate
management from the above list of options. Each option may be used once, more than once,
or not at all.
A 35 year old woman was administered VZIG three weeks ago Your answer: A
because she had close contact with a child with chickenpox at 12
Question 23
weeks gestation and was found to be susceptible on serological
testing Correct answer: J

A 20 year old woman has just returned from holiday and is


surprised to find that she is 10 weeks pregnant based on Your answer: A
ultrasound scan. She reports having been in contact with a child
Question 24
with chickenpox while on holiday 3 weeks earlier and serological
tests show that she is varicella zoster IgM negative but IgG Correct answer: H
positive

| EXPLANATION |

Options for Questions 25-26


A Avoid breast-feeding B Administer VZIG to neonate and avoid breast-
feeding

C Administer VZIG to neonate but breast-feeding D Immediate induction of labour


encouraged
E Delay delivery with tocolytics F Offer termination of pregnancy
G Treat neonate with oral acyclovir H Treat neonate with intravenous acyclovir

I Examine neonate for evidence of congenital J Reassurance


anomalies
K Delay delivery for 5-7 days if possible L Detailed ultrasound examination

Instructions: For each scenario described below, choose the single most appropriate
management from the above list of options. Each option may be used once, more than once,
or not at all.
A 20 year old woman is in hospital 2 days after vaginal delivery of
her first baby and is informed that her 8 year old step-daughter Your answer: A
has chicken-pox. She does not remember having chicken pox in
Question 25
the past and is worried about taking her new baby home.
Investigations show that she is varicella zoster IgG positive and Correct answer: J
IgM negative

A 20 year old woman is being treated in the infectious diseases


unit for chicken pox and reports having missed a period with a Your answer: A
Question 26 positive pregnancy test. Dating scan shows that she has a viable
11 week pregnancy and she is discharged following resolution of Correct answer: L
her symptoms
| EXPLANATION |

Options for Questions 27-28


A Endometriosis B Acute PID
C Chronic PID D Irritable bowel syndrome
E Residual ovary syndrome F Ovarian remnant syndrome
G Diverticular disease H Interstitial cystitis
I Primary dysmenorrhoea J Adenomyosis
K Uterine fibroids L Polycystic ovary syndrome
M Inflammatory bowel disease N Chronic constipation

Instructions: For each of the case histories described below, choose the single most likely
cause of chronic pelvic pain from the above list. Each option may be used once, more than
once, or not at all.
A 34 year old woman with one previous ectopic pregnancy
complains of a 3 year history of progressive pelvic and lower
abdominal pain, dysmenorrhoea and deep dyspareunia. She has
no bowel symptoms but has two previous laparoscopies for Your answer: A
Question 27 drainage of tubo-ovarian abscesses. Endocervical swabs for
Chlamydia are negative and her white cell count and CRP are Correct answer: C
within normal limits. Pelvic ultrasound scan shows a normal size
anteverted uterus with normal ovaries and a small amount of free
fluid in the pouch of Douglas.

A 19 year old woman complains of progressively painful periods


since menarche with occasional deep dyspareunia. She normally
opens her bowels 2-3 times a week and denies any bowel Your answer: A
Question 28 symptoms. She is sexually active and has one previous surgical
termination of an unwanted pregnancy at 11 weeks gestation at Correct answer: I
the age of 17 years. Her symptoms have not responded to simple
analgesia.

| EXPLANATION |

Options for Questions 29-30


A Maternal treatment with antibiotics B Detailed fetal anomaly scan
C Deliver by caesarean section D Active immunisation of neonate
E Avoid breast-feeding F Counsel and offer termination of pregnancy
G Delay delivery by 5-7 days H Administer corticosteroids
I Administer tocolytics J Reassurance
K Screen for maternal syphilis infection

Instructions: For each scenario described below, choose the single most appropriate
management from the above list of options. Each option may be used once, more than once,
or not at all.
A 32 year old woman had a diagnosis of secondary syphilis at 16 Your answer: A
weeks gestation which was successfully treated. She presents in
Question 29
spontaneous labour at 38 weeks gestation. The cervix is 2cm
dilated with intact membranes. Correct answer: J

A 30 year old woman who has not booked for antenatal care Your answer: A
presents with fetal death in-utero. She has a vaginal delivery of a
Question 30
fetus weighing 2700g. Post-mortem shows features consistent
with congenital syphilis. Correct answer: K
| EXPLANATION |

Options for Questions 31-31


A Refer to surgical team B Perform adhesiolysis and ovarian cystectomy
C GnRH analogues D TAH + BSO
E Perform laser ablation of endometriosis F Perform diathermy ablation of endometriosis

G Arrange clinic appointment to discuss further H MRI scan of the pelvis


treatment
I Dilatation and curettage J Endocervical and urethral swabs

K Commence combined oral contraceptive pill L Insert Levonorgestrel releasing intra-uterine


system
M Refer to pain clinic N Refer to gastroenterologist

Instructions: For each of the case histories described below, choose the single most
appropriate management from the above list. Each option may be used once, more than
once, or not at all.
A 23 year old woman with a 3 year history of pelvic pain and deep
dyspareunia has been consented for a diagnostic laparoscopy Your answer: A
because her symptoms have not responded to medical treatment.
Question 31
At laparoscopy, she is found to have deposits of endometriosis in
the ovarian fissae, utero-sacral ligaments and bilateral 3-4cm Correct answer: G
ovarian endometriomas with adhesions.

| EXPLANATION |

Options for Questions 32-33


A Genital swabs for viral culture B Immediate induction of labour
C Delay delivery by 5-7 days if possible D Offer prophylactic oral acyclovir
E Treat with intra-venous acyclovir F Deliver by caesarean section
G Treat neonate with intravenous acyclovir H Reassurance
I Avoid breast-feeding J Avoid contact with other pregnant women and neonates
K Refer to genito-urinary medicine clinic L Screen for other sexually transmitted infections

Instructions: For each scenario described below, choose the single most appropriate
management from the above list of options. Each option may be used once, more than once,
or not at all.
A 17 year old woman complains of generalised abdominal pain
and vulval soreness at 38 weeks gestation. On examination, she Your answer: A
is in urinary retention with florid genital herpetic vesicles. There
Question 32
are uterine contractions 3:10 and the cervix is 2cm dilated with
bulging membranes. There is no previous history of genital Correct answer: F
herpes.

A 30 year old woman with previous genital herpes presents in Your answer: A
spontaneous labour at 39 weeks gestation. There are no recurrent
Question 33
lesions and no prodromal symptoms. She is worried about her
baby contracting the infection Correct answer: H

| EXPLANATION |

Options for Questions 34-34


A Endometriosis B Acute PID
C Chronic PID D Irritable bowel syndrome
E Residual ovary syndrome F Ovarian remnant syndrome
G Diverticular disease H Interstitial cystitis
I Primary dysmenorrhoea J Adenomyosis
K Uterine fibroids L Polycystic ovary syndrome
M Inflammatory bowel disease N Chronic constipation

Instructions: For each of the case histories described below, choose the single most likely
cause of chronic pelvic pain from the above list. Each option may be used once, more than
once, or not at all.
A 77 year old woman presents with a 5 year history of left iliac
fossa pain and deep dyspareunia. Her LMP was 30 years ago and
there has not been any post menopausal bleeding. She has a long
history of constipation but her bowel habit has been regular in Your answer: A
recent years with the use of laxatives. There is no significant past
Question 34
medical history and clinical examination is normal. Her white cell
count, CRP and ESR are elevated and pelvic ultrasound scan Correct answer: G
shows a normal size anteverted uterus with normal ovaries and
two small (2-3cm) complex cystic lesions posterior to the uterus
slightly to the left of the midline.

| EXPLANATION |

Options for Questions 35-36


A Administer VZIG as soon as possible to mother B Administer VZIG to mother if maternal serology -
ve
C Administer VZIG to neonate D Detailed ultrasound examination

E Immediate caesarean section and transfer baby to F Advise avoid contact with other pregnant women
the neonatal unit and neonates
G Induction of labour H Reassurance
I Separate mother and baby after delivery J Serum for VZV IgM antibodies
K Give intravenous Aciclovir L Treat with oral Aciclovir

Instructions: For each scenario described below, choose the single most appropriate
management from the above list of options. Each option may be used once, more than once,
or not at all.
A 26-year-old Para 1+ 0 at 38 weeks gestation contacts her GP Your answer: A
immediately after hearing that a child in her son's nursery has
Question 35
developed chickenpox. She has no memory of having the disease
herself. Correct answer: B

Mrs Jones is seen in the antenatal clinic at 40 weeks. She has a


cough and smokes 20 cigarettes per day. She has a rash and Your answer: A
feels generally unwell. Her sister's child has developed
Question 36
chickenpox. They spent a weekend together two weeks ago. She
does not think she has ever had chickenpox. Serological Correct answer: K
investigation shows that she is susceptible to varicella zoster .

| EXPLANATION |

Options for Questions 37-38


A Genital swabs for viral culture B Immediate induction of labour
C Delay delivery by 5-7 days if possible D Offer prophylactic oral acyclovir
E Treat with intra-venous acyclovir F Deliver by caesarean section
G Treat neonate with intravenous acyclovir H Reassurance
I Avoid breast-feeding J Avoid contact with other pregnant women and neonates
K Refer to genito-urinary medicine clinic L Screen for other sexually transmitted infections

Instructions: For each scenario described below, choose the single most appropriate
management from the above list of options. Each option may be used once, more than once,
or not at all.
A 17 year old woman complains of vulval soreness at 18 weeks Your answer: A
Question 37 gestation. She is found to have vulval herpetic vesicles on
examination. There is no history of previous genital herpes. Correct answer: K

A 20 year old woman presents in spontaneous labour at 37 weeks Your answer: A


Question 38 gestation. She developed primary genital herpes at 34 weeks
gestation but is now symptom-free. Correct answer: F

| EXPLANATION |

Options for Questions 39-40


A Offer hepatitis B passive immunisation to B Offer hepatitis B active immunisation to woman
woman
C Deliver by caesarean section D Amniocentesis for fetal viraemia by PCR

E Fetal blood sampling for viral culture F Offer hepatitis B passive immunisation to neonate from
day 7

G Offer hepatitis B active immunisation to H Offer hepatitis B active & passive immunisation to
neonate neonate
I Avoid breast-feeding J Intra-venous acyclovir in labour
K Reassurance L Screen for hepatitis B infection

Instructions: For each scenario described below, choose the single most appropriate
management from the above list of options. Each option may be used once, more than once,
or not at all.
A 25 year old nurse has been screened for hepatitis B infection at Your answer: A
15 weeks gestation with the following results: Hep B surface
Question 39
antigen negative, Hep B e antigen negative, antibodies to Hep B
surface antigen positive Correct answer: K

A 20 year old woman has been screened for hepatitis B infection Your answer: A
at 15 weeks gestation with the following results: Hep B surface
Question 40
antigen positive, Hep B e antigen positive, antibodies to Hep B
surface antigen positive Correct answer: H

Welcome to Busy SpR - Your professional on-line learning experience for Click here to
Obstetricians and Gynaecologists. signout
MRCOG I PASS-RATE ANALYSIS
Possible metastatic breast cancer peritoneal fluid for cytology would provide tissue
diagnosis
Tissue diagnosis required and best obtained by biopsy of cervical lesion

Copyright Busy SpR

Your score is 8% for this EMQ Exam


You answered 3 out of 40 questions correctly.

40 random questions for EMQ Test

| EXPLANATION |

Options for Questions 1-2

A Routine recall in 3 years B Routine recall in 5 years


C Hysteroscopy + D&C D Laser vapourisation
E Refer for colposcopy F Cold knife cone biopsy
G Radical trachylectomy H Cryotherapy
I No further cervical smears required J Repeat cervical smear in 6 months

Instructions: For each scenario described below, choose the single most appropriate
management from the above list of options. Each option may be used once, more than
once, or not at all.

A 40 year old nulliparous woman had a cervical Your answer: A


smear showing severe dyskaryosis. Colposcopy with
Question 1 diathermy loop excision has been performed and the
histology shows CIN III which has been completely Correct answer:
excised J

A 53 year old asymptomatic post menopausal


woman has a cervical smear showing no dyskaryotic Your answer: A
cells but no evidence of transformation zone
Question 2
sampling. The cervix was well visualised and normal Correct answer:
and she has previously had regular smears which B
have all been negative.

| EXPLANATION |

Options for Questions 3-4

A Induction of labour B Discharge home


C Community midwife follow-up D Anti-hypertensive therapy
E Caesarean section at 39 weeks F Manage as out-patient with day assessment unit
G External cephalic version H Induction of labour at 40 weeks
I Induction of labour at 42 weeks J Measure 24h urine protein excretion

Instructions: For each patient described below choose the single most appropriate
management option from the list. Each option may be used once, more than once, or not
at all.

A 35 year old primigravida is an in-patient because of


Question 3 Your answer: A
hypertension and 1+ proteinuria on stix testing at 37
weeks gestation. Her BP is 130/80mmHg on oral
methyldopa and all her blood tests are within normal Correct answer:
limits. 24h urine protein excretion is 200mg/24h. F

A 20 year old woman is an in-patient because of


proteinuric hypertension at 37 weeks gestation. Her Your answer: A
BP is 140/85mmHg on oral methyldopa and all her
Question 4 blood tests are within normal limits. 24h urine protein
excretion is 800mg. Fetal growth scan shows the Correct answer:
head and abdominal circumferences on the 50th A
centile with normal liquor volume.

| EXPLANATION |

Options for Questions 5-6

A Routine recall in 3 years B Routine recall in 5 years


C Hysteroscopy + D&C D Laser vapourisation
E Refer for colposcopy F Cold knife cone biopsy
G Radical trachylectomy H Cryotherapy
I No further cervical smears required J Repeat cervical smear in 6 months

Instructions: For each scenario described below, choose the single most appropriate
management from the above list of options. Each option may be used once, more than
once, or not at all.

Your answer: A
A 35 year old nulliparous woman has a cervical
Question 5
smear showing severe dyskaryosis Correct answer:
E

A 68 year old woman attends the gynaecology clinic


because she is concerned that she has not been Your answer: A
Question 6 called for a cervical smear. She has always attended
for 3 yearly smears over the previous 15 years which Correct answer: I
have all been negative and she is asymptomatic

| EXPLANATION |

Options for Questions 7-8

A Increase iv fluids B Insert central venous pressure line

C Intravenous magnesium sulphate D Measure serum aspartate


transaminase immediately
E Measure serum magnesium F Blood transfusion
G Monitor patellar reflex every 15 minutes H Provide a fluid challenge with colloids
I Provide intravenous Hartmann J Decrease iv fluids

K Immediate dose of 10ml 10% calcium L Carry out visual field assessment
gluconate intravenously

Instructions: For each patient described below choose the single most appropriate
management option from the list. Each option may be used once, more than once, or not
at all.

Question 7 A 20-year-old primigravida is 30 weeks' pregnant and Your answer: A


has been transferred to the delivery suite with severe
gestational proteinuric hypertension. She complains Correct answer:
of severe frontal headache but has no other C
symptoms. She has a normal respiratory rate and her
urine output has been satisfactory. Her blood
pressure is 140/100mmHg. There are five beats of
bilateral ankle clonus.

A 20-year-old primigravida delivered a live infant 5


hours previously by emergency caesarean section.
She has developed severe gestational proteinuric
hypertension. Blood loss at caesarean section was Your answer: A
estimated at 1,000 ml. BP = 160/100mmHg on oral
Question 8
anti-hypertensive treatment and pulse = 95bpm. She Correct answer:
has been given one litre of Hartmann's solution B
intravenously since her delivery but has passed 40ml
of urine since delivery. Her clotting, liver enzymes
and serum creatinine are normal.

| EXPLANATION |

Options for Questions 9-10

A Routine recall in 3 years B Routine recall in 5 years


C Hysteroscopy + D&C D Laser vapourisation
E Refer for colposcopy F Cold knife cone biopsy

G Repeat cervical smear as soon as H Repeat cervical & endocervical smear in


possible 6 months

I Wertheims hysterectomy + J Repeat cervical smear in 6 months


lymphadenectomy

Instructions: For each scenario described below, choose the single most appropriate
management from the above list of options. Each option may be used once, more than
once, or not at all.

A 32 year old woman has a cervical smear showing


atypical glandular cells. She has colposcopy and a Your answer: A
cone biopsy and the histology shows cervical
Question 9
glandular intra-epithelial neoplasia which has been Correct answer:
completely excised. She has not completed her H
family.

A 35 year old woman has a cervical smear showing Your answer: A


Question 10 atypical glandular cells. Cone biopsy shows micro-
invasive cervical adenocarcinoma. Correct answer: I

| EXPLANATION |

Options for Questions 11-12

A Antihypertensive treatment B Insert central venous pressure line

C Intravenous magnesium sulphate D Measure serum aspartate


transaminase immediately
E Measure serum magnesium F Transfer to intensive treatment unit

G Monitor patellar reflex every 15 minutes H Provide a fluid challenge with


colloids
I Provide intravenous Hartmann's solution at J Calculate the mean arterial blood
the rate of 85ml per hour pressure

K Immediate dose of 10ml 10% calcium L Carry out visual field assessment
gluconate intravenously

Instructions: For each patient described below choose the single most appropriate initial
treatment option from the list. Each option may be used once, more than once, or not at
all.

A 20-year-old primigravida had a normal delivery of a


live infant 12 hours previously. She has developed
severe gestational proteinuric hypertension, her
clotting is normal, serum albumin is 43g/dl, there is Your answer: A
no ankle clonus and her blood pressure is
Question 11
160/100mmHg. She has been given one litre of Correct answer:
Hartmann's solution intravenously since her delivery F
and has been anuric. The central venous pressure is
+10mmHg, serum sodium 132mmol/l, serum
potassium 7.1mmol/l and serum urea 22mmol/l.

A 20-year-old primigravida delivered a live infant 24


hours previously. She has developed severe Your answer: A
gestational proteinuric hypertension. Treatment with
Question 12 intravenous magnesium was required. Her fluid
balance is satisfactory and serum urea, electrolytes Correct answer:
and clotting profile are all normal. Her respiratory rate K
falls to 6 per minute and she is drowsy but rousable.

| EXPLANATION |

Options for Questions 13-13

A Bilateral oophrectomy B CA-125 assays every 6-12 months


C TAH + BSO D Cyst aspiration for cytology
E Refer to cancer centre F Ovarian cystectomy
G Reassure H Measure serum CA-125
I Refer to palliative care team J TAH + BSO + Omentectomy
K Chemotherapy L Radiotherapy

Instructions: For each scenario described below, choose the single most appropriate
management from the above list of options. Each option may be used once, more than
once, or not at all.

A 40 year old nulliparous woman has had a


laparotomy for suspected ovarian cancer. The Your answer: A
findings were of disseminated malignancy with
Question 13 tumour involving both ovaries, the surface of the liver
and bowel with a large omental cake. De-bulking was Correct answer:
not possible and tissue for histology shows a poorly K
differentiated serous adenocarcinoma of the ovary.

| EXPLANATION |

Options for Questions 14-15

A Administer iv frusemide B Immediate delivery by caesarean section

C Intravenous magnesium D Measure serum aspartate transaminase


sulphate immediately
E Measure serum magnesium F Blood transfusion
G Immediate induction of labour H Provide a fluid challenge with colloids
I Provide intravenous Hartmann J Decrease iv fluids

K Measure 24h urine protein L Carry out visual field assessment


excretion

Instructions: For each patient described below choose the single most appropriate
management option from the list. Each option may be used once, more than once, or not
at all.

A 20-year-old primigravida is 35 weeks' pregnant and


has been transferred to the delivery suite with severe Your answer: A
gestational proteinuric hypertension. She complains
Question 14 of severe epigastric pain with nausea and vomiting.
Her respiratory rate and her urine output are normal. Correct answer:
Her blood pressure is 140/90mmHg on treatment D
with methyldopa

An 18-year-old primigravida is 36 weeks' pregnant


and has been referred to the delivery by the
community midwife because of a 3 day history of Your answer: A
feeling generally unwell. Her BP on admission is
Question 15 145/95mmHg with 3+ proteinuria. The uterine size is
appropriate for gestation age with a cephalic Correct answer:
presentation. Her serum aspartate transaminase G
concentration is 245 iu/l but all other investigations
are within normal limits.

| EXPLANATION |

Options for Questions 16-17

A 85% 5 year survival B 70% 5 year survival


C 60% 5 year survival D 40% 5 year survival
E 25% 5 year survival F Virtually 0% 5 year survival
G Almost 100% 5 year survival H Overall life-time risk ~1%
I Overall life-time risk ~2% J Overall life-time risk _1.5%
K Overall life-time risk ~3% L Overall life-time risk of 0.1%

Instructions: For each scenario described below, choose the single most appropriate
information that should be given to the woman from the above list of options. Each option
may be used once, more than once, or not at all.

A 68 year old woman attends the gynaecology clinic


4 weeks after TAH + BSO for endometrial carcinoma. Your answer: A
The histology shows that the tumour had extended to
Question 16
involve the cervix and she has been advised to have Correct answer:
radiotherapy. She enquires about the survival rate for B
women with similar tumours.

A 32 year old teacher has been referred to the


gynaecology clinic because of marked anxiety since Your answer: A
her friend died suddenly from ovarian cancer at the
Question 17
age of 35 years. She has no family history of ovarian
cancer and enquires about the likelihood of any Correct answer: I
woman developing the disease.
| EXPLANATION |

Options for Questions 18-19

A Oestrogen cream B Testosterone cream


C Oral HRT D Topical anti-histamines
E Vaginoplasty F Radical vulvectomy
G Skinning vulvectomy H Topical anti-fungal cream
I Clobetasol cream J Excision biopsy

Instructions: For each clinical scenario below choose the single most appropriate
treatment from the list above. Each diagnosis may be used once, more than once, or not
at all.

A 23-year-old woman presents with a two-year Your answer: A


history of vulval, perineal and perianal irritation. The
Question 18 vulva is red, excoriated and there areas of white,
thickened skin. Application of 3% Acetic acid shows Correct answer:
areas of mosaic and coarse punctuation. J

A 47 year old woman complains of a 3 months


history of vulval irritation and superficial dyapareunia. Your answer: A
Question 19 The vulval skin is thin and white with fissures and
narrowing of the introitus and fusion of the labia Correct answer: I
minora over the clitoris

| EXPLANATION |

Options for Questions 20-21

A Bilateral oophrectomy B CA-125 assays every 6-12 months


C TAH + BSO D Cyst aspiration for cytology
E Refer to cancer centre F Ovarian cystectomy
G Reassure H Measure serum CA-125
I Refer to palliative care team J TAH + BSO + Omentectomy
K Chemotherapy L Radiotherapy

Instructions: For each scenario described below, choose the single most appropriate
management from the above list of options. Each option may be used once, more than
once, or not at all.

A healthy 20 year old woman has been referred by


the on-call surgeon following admission for Your answer: A
suspected appendicitis. Her LMP was 12 days earlier
Question 20 and a pelvic ultrasound scan shows a 2cm simple
cyst on her right ovary with a small amount of free Correct answer:
fluid in the pouch of Douglas. She has a regular 28 G
day cycle and her pregnancy test is negative.

A 56 year old woman has been admitted for


laparotomy for suspected ovarian cancer. Your answer: A
Laparotomy shows a 10cm mobile complex left
Question 21 ovarian mass with a small amount of free fluid and a
normal right ovary. The bowel, omentum, surface of Correct answer:
the liver and diaphragmatic peritoneum are normal. J
There is no obvious lymphadenopathy.
| EXPLANATION |

Options for Questions 22-23

A Bilateral oophrectomy B CA-125 assays every 6-12 months


C TAH + BSO D Cyst aspiration for cytology
E Refer to cancer centre F Ovarian cystectomy
G Reassure H Measure serum CA-125
I Refer to palliative care team J Paracentesis for cytology
K Chemotherapy L Radiotherapy

Instructions: For each scenario described below, choose the single most appropriate
management from the above list of options. Each option may be used once, more than
once, or not at all.

A 35 year old nulliparous woman has had an


oophrectomy after emergency admission with a Your answer: A
torted left ovarian mass. The histology report shows
Question 22
a stage Ia serous cystadenocarcinoma. Her CA-125 Correct answer:
on admission was 650iu and she is keen to maintain B
her reproductive potential

A 35 year mother of 3 children is referred to the


gynaecology clinic because she has been found to Your answer: A
be a carrier of the BRCA-1 mutation following breast
Question 23
cancer in her sister. She has completed her family Correct answer:
and would like the most effective way of minimising A
the risk of ovarian cancer

| EXPLANATION |

Options for Questions 24-25

A Intravenous labetalol B Immediate delivery by caesarean section

C Intravenous magnesium sulphate D Measure serum aspartate transaminase


immediately
E Measure serum magnesium F Blood transfusion
G Immediate induction of labour H Provide a fluid challenge with colloids
I Antihypertensive treatment J Administer iv phenytoin

K Measure 24h urine protein L Place in left lateral position


excretion

Instructions: For each patient described below choose the single most appropriate
management option from the list. Each option may be used once, more than once, or not
at all.

A 20-year-old primigravida has an emergency


caesarean section at 30 weeks gestation because of
severe gestational proteinuric hypertension and
IUGR. Her FBC, LFTs and U&E pre-op were within Your answer: A
normal limits. 12 hours after delivery, investigations
Question 24
show that her serum lactate dehydrogenase Correct answer:
concentration is markedly elevated, serum aspartate C
transaminase concentration is 224 iu/l, platelet count
is 80 x 10E9 /l and her renal function is normal. Her
BP is 140/90 on oral anti-hypertensive therapy and
her urine out-put is 80ml/h.

An asymptomatic 40 year old primigravida attends


the antenatal clinic at 35 weeks gestation with a Your answer: A
previously uncomplicated pregnancy. Her BP is
Question 25
170/95 on two occasions 15 minutes apart and there
is no proteinuria. The uterine size is appropriate for Correct answer: I
gestation age with a cephalic presentation.

| EXPLANATION |

Options for Questions 26-27

A Bilateral oophrectomy B CA-125 assays every 6-12 months


C TAH + BSO D Cyst aspiration for cytology
E Refer to cancer centre F Refer to palliative care team
G Reassure H Measure serum CA-125
I Yearly pelvic ultrasound scans J Repeat pelvic ultrasound scan in 4-6 months

Instructions: For each scenario described below, choose the single most appropriate
management from the above list of options. Each option may be used once, more than
once, or not at all.

A 40 year old woman presents with a 12 months Your answer: A


history of vague abdominal discomfort that did not
Question 26 respond to simple analgesia. Ultrasound scan shows
bilateral complex ovarian cysts with right sided Correct answer:
hydronephrosis and ascites. Her CA-125 is 1500iu E

A 20 year old woman is admitted with sudden onset


left sided pelvic pain 23 days after her LMP. Her Your answer: A
pregnancy test is negative and her symptoms are
Question 27
improving with simple analgesia. Pelvic ultrasound Correct answer:
scan shows a 5cm left ovarian cyst with internal J
echoes consistent with a haemorrhagic cyst.

| EXPLANATION |

Options for Questions 28-29

A Intravenous labetalol B Immediate delivery by caesarean section

C Intravenous magnesium sulphate D Measure serum aspartate transaminase


immediately
E Measure FBC and clotting profile F Blood transfusion
G Immediate induction of labour H Provide a fluid challenge with colloids
I Antihypertensive treatment J Administer iv phenytoin

K Measure 24h urine protein L Arrange in-utero transfer to tertiary centre


excretion

Instructions: For each patient described below choose the single most appropriate
management option from the list. Each option may be used once, more than once, or not
at all.

A 30 year old primigravida attends as an emergency Your answer: A


Question 28 at 32 weeks gestation with a twin pregnancy and
complains of 12h of severe vomiting and upper Correct answer:
abdominal pain. Her BP is 130/80mmHg with a trace B
of proteinuria. A growth scan one week earlier had
shown normal fetal growth. Investigations show a
serum aspartate transaminase concentration of
600iu/l, alkaline phosphatase = 1550iu/l, serum uric
acid = 0.57mmol/l and serum creatinine =
130micromol/l. FBC and coagulation profile are
normal.

A 32 year old primigravida with a previously normal Your answer: A


pregnancy is seen in the antenatal clinic at 39 weeks
Question 29 gestation with a BP of 160/95mmHg and 3+
proteinuria. The uterus is appropriate for gestation Correct answer:
age with a cephalic presentation. G

| EXPLANATION |

Options for Questions 30-31

A Atrophic vulvovaginitis B Human papilloma virus infection


C Benign mucous membrane pemphigoid D Candida infection
E Lichen sclerosus F Contact dermatitis
G Lichen simplex et chronicus H Herpes simplex infection
I Vulval intraepithelial neoplasia J Lichen planus

Instructions: For each clinical scenario below choose the single most likely diagnosis
from the list above. Each diagnosis may be used once, more than once, or not at all.

A 47 year old woman complains of a 3 months Your answer: A


history of vulval irritation and superficial dyapareunia.
Question 30 The vulval skin is thin and white with fissures and
narrowing of the introitus and fusion of the labia Correct answer:
minora over the clitoris E

A 42 year old woman with ulcerative colitis complains


of a 6 months history of vulval irritation and a burning Your answer: A
sensation during intercourse. Examination showed
Question 31
popular eruptions on the flexor surface of her upper Correct answer:
limbs, vulva and vagina with ulcerative lesions in the J
mouth.

| EXPLANATION |

Options for Questions 32-33

A Platelet transfusion B Insert central venous pressure line


C Intravenous magnesium sulphate D Continue iv fluids at current rate
E Measure FBC and clotting profile F Blood transfusion
G Increase iv fluids H Provide a fluid challenge with colloids
I Antihypertensive treatment J Measure 24h urine protein excretion
K Administer iv frusemide L Administer iv albumin

Instructions: For each patient described below choose the single most appropriate
management option from the list. Each option may be used once, more than once, or not
at all.
A 32-year-old primigravida had a normal delivery of a
live infant 10 hours previously. She has developed
severe gestational proteinuric hypertension, her
clotting and FBC are normal, serum albumin is 27g/dl Your answer: A
serum sodium 132mmol/l, serum potassium
Question 32 7.1mmol/l and serum urea 22mmol/l. Her blood
pressure is 160/90mmHg, she has passed 50ml of Correct answer:
urine and been given 1000ml of Hartmann's solution B
since delivery. She complains of being short of
breadth and her oxygen saturation is 93% on 3l/min
oxygen.

A 20-year-old primigravida is 32 weeks pregnant and


has been transferred to the delivery suite for
emergency caesarean section because of gestational Your answer: A
proteinuric hypertension and IUGR. Her BP is
Question 33
160/100 on oral methyldopa. Her platelet count is 42 Correct answer:
x10E9/l, Hb 13.5g/dl, AST 57iu/l and her clotting and A
renal function are normal. She has no symptoms and
her urine out-put is satisfactory.

| EXPLANATION |

Options for Questions 34-35

A Platelet transfusion B Administer fresh frozen plasma


C Intravenous magnesium sulphate D Continue iv fluids at current rate
E Measure FBC and clotting profile F Blood transfusion
G Increase iv fluids H Provide a fluid challenge with colloids
I Antihypertensive treatment J Measure 24h urine protein excretion
K Administer iv frusemide L Decrease iv fluids

Instructions: For each patient described below choose the single most appropriate
management option from the list. Each option may be used once, more than once, or not
at all.

A 40-year-old primigravida had an emergency


caesarean section 10 hours previously because of
severe gestational proteinuric hypertension at 33
weeks gestation. Estimated blood loss was 500ml.
She has marked peripheral oedema and her clotting Your answer: A
and FBC are normal. Serum albumin is 35g/dl serum
Question 34
sodium 132mmol/l, serum potassium 7.1mmol/l and Correct answer:
serum urea 22mmol/l. Her blood pressure is L
160/90mmHg, pulse 78bpm. She has passed 50ml of
urine and been given 1000ml of Hartmann's solution
since delivery. She has also taken 250ml of fluid
orally.

A 20-year-old primigravida has an emergency


caesarean section at 34 weeks gestation because of
severe gestational proteinuric hypertension and
IUGR. The estimated blood loss at caesarean section Your answer: A
was 2000ml. Four hours post-op, her BP is 165/100,
Question 35
pulse 110bpm and she has passed 20ml of urine Correct answer:
since delivery. Her Hb taken 2h after delivery is F
9.0g/dl, and her platelet count and renal function are
normal. She is asymptomatic and there is 20ml of
fluid in the abdominal drain.
| EXPLANATION |

Options for Questions 36-36

A TAH + BSO B Vaginal hysterectomy


C Radical abdominal hysterectomy D Laparoscopic assisted vaginal hysterectomy
E TAH + BSO + omentectomy F BSO
G TAH H Chemotherapy
I Radiotherapy J Combined chemo-radiotherapy
K Endometrial biopsy

Instructions: For each scenario described below, choose the single most appropriate
management from the above list of options. Each option may be used once, more than
once, or not at all.

A 65 years old woman complains of profuse post-


coital bleeding 3 years after TAH + BSO for a stage
Ib endometrial carcinoma. Clinical examination Your answer: A
showed a polypoid lesion at the vaginal vault with
Question 36
contact bleeding and biopsy confirmed recurrent
endometrial adenocarcinoma. MRI scan of the pelvis Correct answer: I
and abdomen identified a 3cm lesion at the vault but
no other abnormalities.

| EXPLANATION |

Options for Questions 37-38

A Administer iv frusemide B Immediate delivery by caesarean section

C Intravenous magnesium D Measure serum aspartate transaminase


sulphate immediately
E Measure serum magnesium F Blood transfusion
G Immediate induction of labour H Provide a fluid challenge with colloids
I Provide intravenous Hartmann J Administer iv phenytoin

K Measure 24h urine protein L Place in left lateral position


excretion

Instructions: For each patient described below choose the single most appropriate
management option from the list. Each option may be used once, more than once, or not
at all.

A 20-year-old primigravida is 30 weeks' pregnant and


has been transferred to the delivery suite with severe
gestational proteinuric hypertension. Corticosteroids Your answer: A
had been administered 3 days earlier and her BP is
Question 37
170/115mmHg on oral methyldopa. Over the next 5 Correct answer:
hours, she is treated with oral nifedipine and B
intravenous labetalol and her BP remains
170/110mmHg. She is asymptomatic.

You have been asked to attend the antenatal ward as


an emergency because a 20 year old primigravida Your answer: A
with severe gestational proteinuric hypertension at 32
Question 38
weeks gestation is having a seizure. On your arrival, Correct answer:
the woman is actively fitting and there are two L
midwives in attendance.
| EXPLANATION |

Options for Questions 39-40

A Endocervical and urethral swabs for culture B EUA + cystoscopy


C MRI scan D Out-patient hysteroscopy and biopsy
E Hysteroscopy D&C F Cervical biopsy
G Cervical smear H Peritoneal fluid for cytology
I Serum CA-125 J Colposcopy

Instructions: For each scenario described below, choose the single most appropriate
investigation from the above list of options. Each option may be used once, more than
once, or not at all.

A 48 year old woman with a history of breast cancer Your answer: A


presents with a 6 months history of vague abdominal
Question 39 discomfort. Clinical examination is suggestive of
ascites which is confirmed on abdominal ultrasound Correct answer:
scan. The ovaries appeared normal. H

A 73 year old woman complains of a 3 week history Your answer: A


of persistent vaginal bleeding. Clinical examination
Question 40 shows a florid 5cm lesion on the anterior lip of her
cervix extending to the upper third of the vagina with Correct answer:
contact bleeding. F

Home | Signup | Subscribe | Contact us | Medical forums | Privacy | Legal Notices | Copyright
Busy SpR
Busy SpR Ltd. is a Registered Company of the United Kingdom. Company Registration Number:4407908
Developed by Last Digital

Welcome to Busy SpR - Your professional on-line learning experience for Click here to
Obstetricians and Gynaecologists. signout
MRCOG I PASS-RATE ANALYSIS
Thursday Apr 06th, 2006

PROBLEM BASED
LEARNING ZONE

Gynaecology
Your score is 10% for this EMQ Exam
Obstetrics

Surgical practice You answered 4 out of 40 questions correctly.


PART II
MRCOG e-COURSE 40 random questions for EMQ Test
Essays

Notes | EXPLANATION |

MCQ Options for Questions 1-2


EMQ
A Counsel and offer B Counsel and offer detailed
PART I termination of pregnancy anomaly scan
ON-LINE TUTORIAL
C Reassure D Commence folic acid
0.4mg/day
PAPER 1
E Commence folic acid F Maternal vitamin K from 36
4mg/day weeks
Anatomy & embryology G Neonatal oral vitamin K H Amniocentesis for karyotype

I Discontinue anti-epileptic J Monitor anti-epileptic drug


drug therapy levels every month
Endocrinology
K Advice against breast- L Advice against pregnancy
feeding
Immunology M Neonatal im vitamin K

Instructions: For each scenario described below, choose the single


Microbiology most appropriate management from the above list of options. Each
option may be used once, more than once, or not at all.

Pharmacology A 30 year old epileptic has been seizure- Your answer:


free for 12 months on sodium valproate. A
Question 1 She has a positive pregnancy test
MOCK following a missed period and is anxious Correct
because she has not bee taking folic acid answer: B

PAPER 2
Physiology (free) Your answer:
A 30 year old epileptic has been seizure- A
free for 12 months on sodium valproate.
Question 2
She is seen in the pre-conception clinic
Biochemistry because she is planning a pregnancy. Correct
answer: E

Genetics
| EXPLANATION |

Options for Questions 3-4


Pathology

A FSH & LH concentration on days B Serum prolactin


MOCK 2-5
C Day 21 progesterone D Random serum
oestradiol

E Neonatal 17-hydroxy-progesterone F Pelvic ultrasound scan


concentration
G Serum androgen concentrations H Progesterone challenge
test

I J MRI scan of abdomen


Dexamethasone suppression test
and pelvis

K Karyotype L Random serum FSH


concentration

Instructions: For each of the case histories described below, choose


the single most appropriate investigation from the above list. Each
option may be used once, more than once, or not at all.

A 20 year old woman has been referred


to the gynaecology clinic because she Your answer:
has not had any menstrual periods for A
over 2 years. She has no other
Question 3
symptoms. On examination, her height is
1.6m and BMI is 22. There was slight Correct
webbing of the neck, widely spaced answer: K
nipples and a wide carrying angle.
An 38 year old woman has just had a
forceps delivery and the neonate has
been found to have ambiguous external Your answer:
genitalia. Her medical records show that A
Question 4 she had amniocentesis at 18 weeks
gestation because of a high risk of Correct
Downs syndrome on serum screening answer: E
and the fetus was found to have a 46XX
karyotype.

| EXPLANATION |

Options for Questions 5-6

A Commence treatment with B Commence treatment with


metformin insulin

C Stop insulin D Commence insulin-glucose


regimen

E Recommend delivery by F Induce labour at 36 weeks


caesarean section at 38 weeks gestation

G Induce labour at 39-40 weeks H Increase insulin dose


gestation

I J Re-introduce pre-
Stop insulin-glucose regimen
pregnancy insulin regimen

K Counsel and offer termination of L Reassure


pregnancy
M Repeat GTT 6-12 weeks post-
partum

Instructions: For each scenario described below, choose the single


most appropriate management from the above list of options. Each
option may be used once, more than once, or not at all.

A 35 year old woman with gestational Your answer:


diabetes mellitus has been managed with A
Question 5 diet alone during pregnancy and has had
a normal vaginal delivery at 39 weeks Correct
with an insulin-glucose regimen. answer: I

A 35 year old woman with gestational Your answer:


diabetes has been treated with insulin A
during the antenatal period. She has a
Question 6
normal vaginal delivery at 38 weeks
gestation and is due for discharge 3 days Correct
post-partum answer: M

| EXPLANATION |

Options for Questions 7-8

A Commence treatment with B Commence treatment with


metformin insulin

C Stop insulin D Commence insulin-glucose


regimen

E Recommend delivery by F Induce labour at 36 weeks


caesarean section at 38 weeks gestation
G Induce labour at 39-40 weeks H Increase insulin dose
gestation

I J Re-introduce pre-
Stop insulin-glucose regimen
pregnancy insulin regimen

K Counsel and offer termination of L Reassure


pregnancy

M Repeat GTT 6-12 weeks post-


partum

Instructions: For each scenario described below, choose the single


most appropriate management from the above list of options. Each
option may be used once, more than once, or not at all.

A 32 year old with IDDM attends the


antenatal clinic at 32 weeks gestation. Your answer:
Her blood glucose concentration on home A
Question 7 monitoring over the preceding week have
been as follows: Pre-breakfast: 9-12mM; Correct
pre-lunch: 8-15mM, pre-supper: 9 - answer: H
12mM; bedtime: 5-12mM

A 35 year old woman with well controlled Your answer:


gestational diabetes has a growth scan at A
38 weeks gestation showing fetal head
Question 8
and abdominal circumferences on the
50th centile. All other maternal and fetal Correct
observations are within normal limits. answer: G

| EXPLANATION |

Options for Questions 9-10

A In-vitro fertilisation B Intra-uterine insemination

C Laparoscopy and dye D Intra-cytoplasmic sperm injection


test
E Clomephene citrate F Gonadotrophin induction of ovulation
G Metformin H Oocyte donation
I Surrogacy J Weight reduction

K Weight gain L Measure serum androgen


concentrations

Instructions: For each of the case histories described below, choose


the single most appropriate initial management from the above list.
Each option may be used once, more than once, or not at all.

A 24 year old woman with her 35 year old


partner has been referred to the infertility
clinic because of 2 years of primary
infertility. The woman has an irregular
menstrual cycle every 2-4 months and Your answer:
her partners semen analysis is normal. A
The womans BMI is 26 and there is no
Question 9
significant past medical history.
Investigations have confirmed a Correct
diagnosis of polycystic ovary. The couple answer: C
have not achieved a pregnancy despite a
6 months course of clomephene citrate
with evidence of ovulation based on day
21 progesterone concentration and
ultrasound scanning.

A 34 year old woman with her 35 year old


partner has been referred to the infertility
clinic because of 2 years of primary
infertility. The woman has a regular 28
day cycle and her periods are heavy with
severe dysmenorrhoea. She also Your answer:
complains of deep dyspareunia and A
intermittent lower abdominal and pelvic
Question 10
pain. Her BMI is 27. Investigations have
shown normal LH, FHS, Thyroid function Correct
tests, prolactin and day 21 progesterone answer: A
concentration confirms ovulatory cycles.
Laparoscopy and dye test shows
evidence of chronic pelvic inflammatory
disease with bilateral tubal occlusion. Her
partners semen analysis is normal.

| EXPLANATION |

Options for Questions 11-12

A Stop heparin on the evening B Commence oral warfarin


before induction

C Await results of V/Q scan then D Await results of venogram


commence treatment then commence treatment

E Await results of D-dimers F Commence therapeutic dose


assay of heparin

G Commence prophylactic dose H Oral aspirin therapy


of heparin post-partum
I Antenatal prophylactic J Antenatal treatment with
treatment with heparin warfarin

K Prophylactic heparin for 6 L Stop heparin therapy


weeks post-partum

Instructions: For each scenario described below, choose the single


most appropriate management from the above list of options. Each
option may be used once, more than once, or not at all.

A 30 year old woman with a previous Your answer:


DVT presents for antenatal care at 15 A
weeks gestation. There is no family
Question 11
history of VTE and her BMI is 25. She is
otherwise fit and well with a negative Correct
thrombophilia screen answer: K

A 30 year old woman with recurrent first


trimester miscarriages is known to have Your answer:
the anti-phospholipid antibody syndrome A
Question 12 and has had a spontaneous vaginal
delivery at 39 weeks gestation. Her BMI Correct
is 27 and she is otherwise fit and well answer: G
with no family history of VTE.

| EXPLANATION |

Options for Questions 13-14


A Prolactinoma B Pre-mature ovarian failure

C Ashermans syndrome D Ovarian hyper-stimulation


syndrome
E Polycystic ovary syndrome F Hypothyroidism
G Androgen secreting tumour H Cervical stenosis
I Hypothalamic amenorrhoea J Hyperthyroidism

K Androgen insensitivity L Pure gonadal dysgenesis


syndrome

Instructions: For each of the case histories described below, choose


the single most likely cause of amenorrhoea from the above list. Each
option may be used once, more than once, or not at all.

An 18 year virgin has been referred to the


gynaecology clinic because she has not Your answer:
started menstruating but she has no other A
symptoms. Clinical examination showed
Question 13
that her height was 1.67m with BMI of 23.
She had normally formed breasts but no Correct
axillary or pubic hair. There was a 2-3cm answer: K
mass palpable in each inguinal canal.

A 17 year old woman with primary Your answer:


amenorrhoea has been investigated with A
the following results: FSH = 9mIU/ml, LH
Question 14
= 20mIU/ml, oestradiol = 200pg/ml, free
testosterone = 5.0ng/dl (1-4.3ng/dl). Correct
Karyotype is 46XY. answer: K

| EXPLANATION |

Options for Questions 15-16

A Amniotic fluid embolism B Placental abruption


C Cardiomyopathy D Placenta praevia
E Chest infection F Pulmonary embolism
G CVA H Pulmonary hypertension
I Endocarditis J Sepsis
K Haemorrhage L Substance misuse
M HELLP syndrome N Thromboembolism
O Myocardial infarcation

Instructions: For each case described below, choose the single most
likely cause of maternal death from the above list of options. Each
option may be used once, more than once, or not at all.

A 42 year old woman presents with a 30


minute history of constant abdominal pain Your answer:
at 38 weeks gestation. Her blood A
pressure is 110/70 with a pulse of 120
Question 15
bpm on admission. The fetal heart is not
detectable. She collapsed 15 minutes Correct
after admission and died despite answer: B
intensive resuscitation

A 32 year old woman with severe pre- Your answer:


Question 16
eclampsia has had an emergency A
caesarean section at 32 weeks gestation
because of IUGR. She became Correct
hypotensive and tachycardic and was answer: N
returned to theatre where 2l of blood was
evacuated from the abdomen and
hysterectomy was performed. Three days
later, she developed sudden onset
severe chest pain and breathlessness
and dies despite resuscitation

| EXPLANATION |

Options for Questions 17-18

A Commence treatment with B Commence treatment with


metformin insulin

C Stop insulin D Commence insulin-glucose


regimen

E Recommend delivery by F Induce labour at 36 weeks


caesarean section at 38 weeks gestation

G Induce labour at 39-40 weeks H Increase insulin dose


gestation

I J Re-introduce pre-
Stop insulin-glucose regimen
pregnancy insulin regimen
K Counsel and offer termination of L Reassure
pregnancy

M Repeat GTT 6-12 weeks post-


partum

Instructions: For each scenario described below, choose the single


most appropriate management from the above list of options. Each
option may be used once, more than once, or not at all.

Your answer:
A 20 year old insulin-dependent diabetic A
Question 17 attends for induction of labour at 38
weeks gestation Correct
answer: D

A 32 year old with adequately controlled Your answer:


IDDM has a growth scan at 36 weeks A
gestation which shows that the fetal
Question 18
abdominal circumference is well above
the 97th centile with an estimated fetal Correct
weight of 4200g. answer: E

| EXPLANATION |

Options for Questions 19-20

A Stop heparin on the evening B Commence oral warfarin


before induction

C Await results of V/Q scan then D Await results of venogram


commence treatment then commence treatment

E Await results of D-dimers F Commence therapeutic dose


assay of heparin
G Commence prophylactic dose H Oral aspirin therapy
of heparin post-partum

I Antenatal prophylactic J Antenatal treatment with


treatment with heparin warfarin

K Prophylactic heparin for 6 L Stop heparin therapy


weeks post-partum

Instructions: For each scenario described below, choose the single


most appropriate management from the above list of options. Each
option may be used once, more than once, or not at all.

A 30 year old woman with a BMI 0f 25


has had a spontaneous vaginal delivery. Your answer:
She has a history of a previous DVT A
Question 19 while on the COCP but the thrombophilia
screen was negative and has been on s/c Correct
heparin since delivery. She is now answer: K
awaiting discharge

A 30 year old woman attends the Your answer:


antenatal clinic at 12 weeks gestation. A
She had a DVT in her previous
Question 20
pregnancy and her sister also had a DVT
during pregnancy. Thrombophilia screen Correct
is negative. answer: I

| EXPLANATION |

Options for Questions 21-22

A In-vitro fertilisation B Intra-uterine insemination

C Laparoscopy and dye D Laparoscopic ovarian drilling


test
E Clomephene citrate F Gonadotrophin induction of ovulation
G Metformin H Carbegolline
I Oocyte donation J Weight reduction

K Weight gain L Measure serum androgen


concentrations

Instructions: For each of the case histories described below, choose


the single most appropriate initial management from the above list.
Each option may be used once, more than once, or not at all.

A 34 year old woman with her 35 year old


partner has been referred to the infertility
clinic because of 2 years of primary
infertility. The woman has a regular 28
day cycle and her periods are heavy with Your answer:
severe dysmenorrhoea. She also A
Question 21 complains of deep dyspareunia and
intermittent lower abdominal and pelvic Correct
pain. Her BMI is 27. Investigations have answer: C
shown normal LH, FHS, Thyroid function
tests, prolactin and day 21 progesterone
concentration confirms ovulatory cycles.
Her partners semen analysis is normal.

A 34 year old woman with her 35 year old Your answer:


Question 22 partner has been referred to the infertility A
clinic because of 2 years of primary
infertility. The woman has a regular and Correct
normal 28 day cycle, her BMI is 26 and answer: A
she has no other symptoms.
Investigations have shown normal LH =
3.5mIU/ml, FHS = 5.0mIU/ml, Thyroid
function tests, prolactin and day 21
progesterone concentration confirms
ovulatory cycles. Her partners semen
analysis is normal. Hystero-salpingogram
confirms bilateral patent fallopian tubes.

| EXPLANATION |

Options for Questions 23-24

A In-vitro fertilisation B Intra-uterine insemination

C Laparoscopy and dye D Laparoscopic ovarian drilling


test
E Clomephene citrate F Gonadotrophin induction of ovulation
G Metformin H Oocyte donation
I Hystero-salpingogram J Weight reduction

K Weight gain L Measure serum androgen


concentrations

Instructions: For each of the case histories described below, choose


the single most appropriate initial management from the above list.
Each option may be used once, more than once, or not at all.

A 41 year old woman with her 35 year old


partner has been referred to the infertility
clinic because of 2 years of primary
infertility. The woman has an irregular Your answer:
menstrual cycle every 2-6 weeks, her A
BMI is 26 and she has no other
Question 23
symptoms. Investigations have shown
normal LH = 7.5mIU/ml, FHS = Correct
22mIU/ml, day 21 progesterone = answer: H
3.5ng/ml. Thyroid function tests, prolactin
and androgen concentrations are normal.
Her partners semen analysis is normal.

A 37 year old woman with her 45 year old


partner has been referred to the infertility
clinic because of 2 years of primary
infertility. The woman has an irregular
menstrual cycle every 2-4 months and Your answer:
her partners semen analysis shows a A
volume of 3ml, concentration of
Question 24
30,million/ml and motility of 60%. The
womans BMI is 26 and there is no Correct
significant past medical history. answer: D
Investigations have confirmed a
diagnosis of polycystic ovary syndrome
which has not responded to treatment
with clomephene citrate over 6 months.

| EXPLANATION |

Options for Questions 25-26


A Prolactinoma B Pre-mature ovarian failure

C Turners syndrome D Ovarian hyper-stimulation


syndrome
E Polycystic ovary syndrome F Hypothyroidism
G Cushings syndrome H Congenital adrenal hyperplasia
I Hypothalamic amenorrhoea J Hyperthyroidism

K Androgen insensitivity L Pure gonadal dysgenesis


syndrome

Instructions: For each of the case histories described below, choose


the single most likely cause of menstrual abnormalities from the above
list. Each option may be used once, more than once, or not at all.

A 36 year old woman and her 40 year old


partner have been referred to the fertility
clinic because of a failure to conceive
after 18 months of unprotected
intercourse. The semen analysis is Your answer:
normal. The woman has irregular periods A
every 3-6 months but no other symptoms
Question 25
and her BMI is 27. The womans results
are as follows: FSH (day 3) = 6mIU/ml, Correct
LH (day 3) = 8mIU/ml, prolactin = 110 answer: E
ng/ml, progesterone (day 21) = 3ng/ml,
testosterone = 4.0pg/ml, DHEA, DHEA-
sulphate and thyroid function tests were
normal.

A 25 year old medical student attends the


gynaecology clinic because she has not
had a period for 12 months. Menarche
was at the age of 13 and she is otherwise
asymptomatic and healthy. Her BMI is 19
and she is a member of the rowing team, Your answer:
practising 5 times a week. She denies A
any recent weight loss or abnormal eating
Question 26
behaviour and has normal secondary
sexual characteristics. Random Correct
endocrine profile is as follows: FSH = answer: I
5.5mIU/ml, LH = 4.0mIU/ml,
progesterone = 1.2ng/ml, prolactin =
56ng/ml, testosterone = 1.2pg/ml. Thyroid
function tests and pelvic ultrasound scan
are normal.

| EXPLANATION |

Options for Questions 27-28

A Cancel IVF cycle B Freeze embryos

C Admit for iv fluids and D Termination of pregnancy


thromboprophylaxis
E Transfer to ITU F Fluid restriction
G Drain pleural effusion H Drain ascites

I J Proceed with embryo


Drain ovarian cysts
replacement

K Avoid unprotected sexual L Diuretics


intercourse
Instructions: For each of the case histories described below, choose
the single most appropriate management from the above list. Each
option may be used once, more than once, or not at all.

A 35 year old woman with PCOS is


undergoing IVF treatment. She attends
for embryo replacement 6 days after
HCG administration and complains of Your answer:
abdominal distension and pain with A
Question 27 nausea and vomiting 2-3 times per day
over the previous 48h. Clinical Correct
examination showed a mildly distended answer: B
abdomen and ultrasound scan confirmed
bilateral ovarian enlargement 8-10cm. All
her blood tests were normal.

A 35 year old woman with primary


infertility is undergoing IVF treatment.
She attends 10 days after embryo
replacement and complains of abdominal Your answer:
distension and pain with nausea and A
persistent severe vomiting over the
Question 28
previous 48h. Clinical examination
showed a markedly distended abdomen Correct
with ascites and pleural effusion. answer: C
Abdominal and pelvic ultrasound scan
confirmed 12cm diameter bilateral cystic
ovaries. All her blood tests were normal.

| EXPLANATION |

Options for Questions 29-30

A Counsel and offer B Counsel and offer detailed


termination of pregnancy anomaly scan
C Reassure D Commence folic acid
0.4mg/day

E Commence folic acid F Maternal vitamin K from 36


4mg/day weeks
G Neonatal oral vitamin K H Amniocentesis for karyotype

I Discontinue anti-epileptic J Monitor anti-epileptic drug


drug therapy levels every month
K Advice against breast- L Advice against pregnancy
feeding
M Neonatal im vitamin K

Instructions: For each scenario described below, choose the single


most appropriate management from the above list of options. Each
option may be used once, more than once, or not at all.

An 18 year old epileptic on phenytoin Your answer:


presents in spontaneous labour A
Question 29 unbooked. She has a spontaneous
vaginal delivery of a baby weighing Correct
3600g. answer: M

A year old epileptic has been seizure-free Your answer:


Question 30
for 18 months on sodium valproate has A
an anomaly scan showing evidence of
fetal spina-bifida Correct
answer: A

| EXPLANATION |

Options for Questions 31-32

A Commence oral warfarin B Await results of D-dimers assay

C Commence therapeutic D Prophylactic dose of heparin for


dose of heparin 3-5 days post-partum

E Antenatal prophylactic F Antenatal treatment with


treatment with heparin warfarin

G Prophylactic heparin for 6 H Continue s/c heparin for 6


weeks post-partum weeks
I J Continue heparin therapy for 6
Stop heparin therapy
months

K No treatment required L Continue heparin prophylaxis


during labour

Instructions: For each scenario described below, choose the single


most appropriate management from the above list of options. Each
option may be used once, more than once, or not at all.

Your answer:
A 35 year old woman with a BMI of 39 A
has been admitted at 8 weeks gestation
Question 31
because of persistent nausea and
vomiting Correct
answer: E

A 28 year old woman with a previous Your answer:


DVT is known to have the anti- A
Question 32 phospholipid antibody syndrome. She
has been referred to the antenatal clinic Correct
at 10 weeks gestation answer: E

| EXPLANATION |

Options for Questions 33-34

A Amniotic fluid embolism B Placental abruption


C Cardiomyopathy D Placenta praevia
E Chest infection F Uterine inversion
G CVA H Pulmonary hypertension
I Endocarditis J Sepsis
K Haemorrhage L Substance misuse
M HELLP syndrome N Thromboembolism
O Myocardial infarcation

Instructions: For each case described below, choose the single most
likely cause of maternal death from the above list of options. Each
option may be used once, more than once, or not at all.

A previously healthy 18-year-old Your answer:


Question 33
primigravida presents at 36 weeks feeling A
unwell and tired. Her brother died
unexpectedly aged 19 years. Her CXR Correct
showed an enlarged heart. While being answer: C
admitted she developed increasing
shortness of breath and died despite
intensive resuscitation.

A 30-year-old woman, 28 weeks'


gestation in her sixth pregnancy presents Your answer:
to A&E with breathlessness and displays A
Question 34 severe anxiety. She had complained of
left-sided pelvic pain for a week. While Correct
being assessed she collapsed and it was answer: N
not possible to resuscitate her.

| EXPLANATION |

Options for Questions 35-36

A Administer regional B Administer regional analgesia if


analgesia if APTT is normal APTT and PT are normal
Administer protamine
C sulphate then regional D Advice that regional analgesia is
contra-indicated
analgesia

E Administer regional F Administer prophylactic dose of


analgesia heparin
Check APTT then remove
G Remove epidural catheter H epidural catheter if result is
normal
Check anti-Xa levels then
I Advise against removal of J administer regional analgesia if
epidural catheter
normal

K Wait for 12h then administer


heparin

Instructions: For each scenario described below, choose the single


most appropriate management from the above list of options. Each
option may be used once, more than once, or not at all.

A 40 year old woman with a BMI of 39


has had an emergency caesarean
section for failure to progress at 6cm Your answer:
dilatation under combined spinal epidural A
(CSE) analgesia. The procedure was
Question 35
uncomplicated with a blood loss of 700ml.
A prophylactic does of LMWH was Correct
administered at noon, 6h after insertion of answer: I
CSE. She requests removal of the
epidural catheter at 18:00h.

A 25 year old woman with a BMI of 38


has an emergency caesarean section at Your answer:
full dilatation under spinal anaesthesia A
because of fetal distress. The procedure
Question 36
was uncomplicated with blood loss of
700ml. She is now 8h post-surgery and Correct
your attention is drawn to instructions on answer: F
post-partum thromboprophylaxis.
| EXPLANATION |

Options for Questions 37-38

A In-vitro fertilisation B Intra-uterine insemination

C Laparoscopy and dye D Laparoscopic ovarian drilling


test
E Clomephene citrate F Gonadotrophin induction of ovulation
G Metformin H Carbegolline
I Oocyte donation J Weight reduction

K Weight gain L Measure serum androgen


concentrations

Instructions: For each of the case histories described below, choose


the single most appropriate initial management from the above list.
Each option may be used once, more than once, or not at all.

A 35 year old woman with her 45 year old


partner has been referred to the infertility
clinic because of 2 years of primary
infertility. The woman has an irregular Your answer:
menstrual cycle every 2-4 months and A
her partners semen analysis shows a
Question 37
volume of 3ml, concentration of
30,million/ml and motility of 60%. The Correct
womans BMI is 24 and there is no answer: E
significant past medical history.
Investigations have confirmed a
diagnosis of polycystic ovary syndrome.

A 24 year old woman with her 35 year old


partner has been referred to the infertility
clinic because of 2 years of primary
infertility. The woman has an irregular Your answer:
menstrual cycle every 2-4 months and A
her partners semen analysis shows a
Question 38
volume of 2.5ml, concentration of
30,million/ml and motility of 65%. The Correct
womans BMI is 34 and there is no answer: J
significant past medical history.
Investigations have confirmed a
diagnosis of polycystic ovary syndrome.

| EXPLANATION |

Options for Questions 39-40

A Prolactinoma B Pre-mature ovarian failure

C Turners syndrome D Ovarian hyper-stimulation


syndrome
E Polycystic ovary syndrome F Hypothyroidism
G Cushings syndrome H Congenital adrenal hyperplasia
I Hypothalamic amenorrhoea J Hyperthyroidism

K Androgen insensitivity L Pure gonadal dysgenesis


syndrome

Instructions: For each of the case histories described below, choose


the single most likely cause of menstrual abnormalities from the above
list. Each option may be used once, more than once, or not at all.

A 35 year old woman and her 40 year old


partner have been referred to the fertility
clinic because of a failure to conceive
after 3 years of unprotected intercourse.
The semen analysis is normal. The Your answer:
woman has irregular periods every 3-6 A
months but no other symptoms and her
Question 39
BMI is 27. The womans results are as
follows: FSH (day 3) = 20mIU/ml, LH (day Correct
3) = 8mIU/ml, prolactin = 110 ng/ml, answer: B
progesterone (day 21) = 1.5ng/ml,
testosterone = 2.0pg/ml, DHEA, DHEA-
sulphate and thyroid function tests were
normal.

A 35 year old woman with one previous


normal pregnancy 3 years earlier attends
the gynaecology clinic because of
irregular periods every 4-6 months. On Your answer:
direct questioning, she complains of A
Question 40 persistent headaches in the last 9 months
which are worse first thing in the morning Correct
and a milky discharge from her right answer: A
breast. Her BMI is 27 and there is a bi-
temporal hemi-anopia on clinical
examination.

Home | Signup | Subscribe | Contact us | Medical forums | Privacy | Legal Notices | Copyright
Busy SpR
Busy SpR Ltd. is a Registered Company of the United Kingdom. Company Registration Number:4407908
Developed by Last Digital
Options for Questions 1-2

A Emergency caesarean section B Ultrasound scan for placental site


C Induction of labour with prostaglandins D Elective caesarean section at 37 weeks
E Oxytocin augmentation of labour F Fetal scalp blood sampling
G Induction of labour with oxytocin H Umbilical artery Doppler
I Elective caesarean section at 39 weeks J Arrange antenatal clinic follow-up
K Expectant management L Transfer to high dependency unit
M Maternal blood transfusion N Vaginal operative delivery

Instructions: For each of the case histories described below, choose the single most appropriate
management from the above list of options. Each option may be used once, more than once, or not at
all.

A 20 year old woman complains of constant abdominal pain


and vaginal bleeding at 34 weeks gestation. Her BP is 130/60 Your answer: A
Question 1 and pulse is 90bpm. CTG shows contractions every 3
minutes with deep variable decelerations. The cervix is Correct answer: A
partially effaced and the os is closed.

A 35 year old woman presents at 41 weeks gestation with Your answer: A


Question 2 spontaneous rupture of the membranes and heavily blood-
stained liquor. Maternal pulse and BP are normal and the Correct answer: G
CTG is reactive. The cervix is partially effaced and 1cm
dilated. There are no uterine contractions.

| EXPLANATION |

Options for Questions 3-4

A Uterine perforation B Laparotomy


C Bladder injury D Bleeding requiring transfusion
E Continuing pregnancy F Infertility
G Wound infection H Failure to identify any pathology
I Failure to gain access to abdominal cavity J Risk of failure of the procedure

Instructions: For each of the case histories described below, choose the single most relevant
complication that you must discuss with the patient when taking consent prior to surgery from the above
list of options. Each option may be used once, more than once, or not at all.

A 34 year old woman with a previous left salpingectomy for Your answer: A
ectopic pregnancy presents with abdominal pain and vaginal
Question 3
bleeding and is found to have an empty uterus with a right
adnexal mass consistent with an ectopic pregnancy. Correct answer: F

A 35 year old woman with a 10 week missed miscarriage is Your answer: A


Question 4 undergoing evacuation of retained products of conception
under general anaesthesia Correct answer: A

| EXPLANATION |

Options for Questions 5-6

A Emergency caesarean section B Ultrasound scan for placental site


C Induction of labour with prostaglandins D Induction of labour by amniotomy
E Oxytocin augmentation of labour F Fetal scalp blood sampling
G Vaginal operative delivery H Maternal blood transfusion
I Transfer to high dependency unit J Expectant management
K Arrange antenatal clinic follow-up L Treatment with tocolytics
M Umbilical artery Doppler N Perform Kleihauer test

Instructions: For each of the case histories described below, choose the single most appropriate
management from the above list of options. Each option may be used once, more than once, or not at
all.

A 35 year old woman presents at 41 weeks gestation with a


history of fresh vaginal bleeding and passing several clots. Your answer: A
The uterus is soft and non-tender with no contractions. The
Question 5
fetal heart is normal and the womans pulse and BP are
normal. The placenta is not low and the cervix is partially Correct answer: D
effaced, 2cm dilated with bulging membranes

A 20 year old woman presents at 28 weeks gestation with


fresh vaginal bleeding. The fetal heart is normal and the Your answer: A
Question 6 womans pulse and BP are normal. The placenta is fundal
and the cervix is 1cm long with a closed internal os. There are Correct answer: J
uterine contractions occurring every 3 minutes.
| EXPLANATION |

Options for Questions 7-8

A Serial beta-HCG assay B Laparoscopy


C Repeat trans-vaginal scan in 24h D Repeat trans-vaginal scan in 7 days
E Counsel and discharge F Offer surgical evacuation of products of conception
G Offer medical treatment with methotrexate H Laparotomy
I Serum progesterone assay J Serum AFP assay
K Refer to molar pregnancy centre L Refer to other specialty

Instructions: Instructions: For each patient described below choose the single most appropriate initial
management option from the list. Each option may be used once, more than once, or not at all.

An asymptomatic 30 year old woman with a previous ectopic


pregnancy presents at 6 weeks amenorrhoea for trans-
vaginal scan to exclude a repeat ectopic pregnancy. The scan Your answer: A
Question 7 is reported to show a 14mm endometrium with no gestation
sac. There are no adnexal masses and no free fluid in the Correct answer: D
pouch of Douglas. HCG concentration is 500IU and 1600IU
48h later

An asymptomatic 30 year old woman with a previous ectopic


pregnancy presents at 6 weeks amenorrhoea for trans-
vaginal scan to exclude a repeat ectopic pregnancy. Her Your answer: A
pregnancy test had been positive 2 weeks earlier. The scan is
Question 8
reported to show a 14mm endometrium with no gestation sac.
There is a 4cm cystic structure in the right adnexum Correct answer: B
suggestive of an ectopic pregnancy but no free fluid in the
pouch of Douglas

| EXPLANATION |

Options for Questions 9-10

A Serial beta-HCG assays B Laparoscopy


C Repeat trans-vaginal scan in 24h D Repeat trans-vaginal scan in 7 days
E Counsel and discharge F Offer surgical evacuation of products of conception
G Offer medical treatment with methotrexate H Laparotomy
I Serum progesterone assay J Serum AFP assay
K Refer to molar pregnancy centre L Refer to other specialty

Instructions: For each patient described below choose the single most appropriate initial management
option from the list. Each option may be used once, more than once, or not at all.

A 42 year old woman presents with 8 weeks of amenorrhoea, Your answer: A


a positive pregnancy test and fresh vaginal bleeding. Trans-
Question 9
vaginal scan shows an enlarged uterus with a snow-storm
appearance consistent with a complete molar pregnancy Correct answer: F

A 24 year old woman with a regular 28 day cycle had


attended 2 weeks earlier with abdominal pain and fresh
vaginal bleeding. Trans-vaginal scan showed a viable 6 week Your answer: A
Question 10 intra-uterine pregnancy. She now re-attends with persistent
vaginal bleeding which has settled over the last 24h. Trans- Correct answer: E
vaginal scan shows an empty uterus with a 3cm
haemorrhagic left ovarian cyst and a small amount of free
fluid in the pouch of Douglas.

| EXPLANATION |

Options for Questions 11-12

A Counsel and offer evacuation of retained products B Counsel and offer support group
of conception

C Counsel and offer laparoscopy & salpingectomy D Counsel and offer investigation for recurrent
miscarriage

E Counsel and offer treatment with methotrexate F Counsel and offer laparotomy &
salpingectomy
G Counsel and offer termination of pregnancy H Perform salpingectomy
I Proceed to laparotomy J Serial HCG assay

K Counsel and refer to early pregnancy assessment L Repeat trans-vaginal scan 7 days later
unit

Instructions: For each patient described below choose the single most appropriate initial management
option from the list. Each option may be used once, more than once, or not at all.

A 25 year old woman had a transvaginal scan following IVF


treatment during which 2 embryos were replaced and was Your answer: A
found to have a 6 week singleton intra-uterine pregnancy.
Question 11
She presents 2 weeks later with abdominal pain and vaginal
bleeding and is found to have an empty uterus with a small Correct answer: B
amount of free fluid in the pouch of Douglas.

A 25 year old woman had a transvaginal scan following IVF


treatment during which 2 embryos were replaced and is found Your answer: A
Question 12 to have a 6 week singleton intra-uterine pregnancy with a
5cm cystic structure adjacent to the right ovary with internal Correct answer: C
echoes consistent with an ectopic pregnancy.

| EXPLANATION |

Options for Questions 13-14

A Serial beta-HCG assays B Laparoscopy


C Repeat trans-vaginal scan in 24h D Repeat trans-vaginal scan in 7 days
E Reassure F Offer surgical evacuation of products of conception
G Offer medical treatment with methotrexate H Laparotomy
I Serum progesterone assay J Serum AFP assay
K Refer to molar pregnancy centre L Refer to other specialty

Instructions: For each patient described below choose the single most appropriate initial management
option from the list. Each option may be used once, more than once, or not at all.

An asymptomatic 24 year old woman with a regular 28 day


cycle presents with 7 weeks amenorrhoea. Her pregnancy Your answer: A
Question 13 test is positive. Trans-vaginal scan shows a gestation sac of
20mm diameter with a 12mm (~7 weeks size) fetal pole but Correct answer: F
no fetal heart activity.

A 24 year old woman with an irregular menstrual cycle Your answer: A


Question 14 presents with 7 weeks amenorrhoea and fresh vaginal
bleeding. Her pregnancy test is positive. Trans-vaginal scan
shows a gestation sac of 12mm diameter (~5 weeks) with a Correct answer: D
yolk sac but no fetal pole

| EXPLANATION |

Options for Questions 15-16

A Emergency caesarean section B Ultrasound scan for placental site


C Induction of labour with prostaglandins D Induction of labour by amniotomy
E Oxytocin augmentation of labour F Fetal scalp blood sampling
G Vaginal operative delivery H Maternal blood transfusion
I Transfer to high dependency unit J Expectant management
K Arrange antenatal clinic follow-up L Treatment with tocolytics
M Umbilical artery Doppler N Perform Kleihauer test

Instructions: For each of the case histories described below, choose the single most appropriate
management from the above list of options. Each option may be used once, more than once, or not at
all.

A 34 year old woman is brought in by ambulance because of Your answer: A


severe constant abdominal pain and vaginal bleeding at 35
Question 15
weeks gestation. On admission, her pulse is 120bpm, BP
80/40 and she is unresponsive. The fetal heart is 80bpm Correct answer: H

A 25 year old woman with major placenta previa presents at Your answer: A
Question 16 32 weeks gestation with fresh vaginal bleeding. Her pulse is
70bpm and BP 120/80. The fetal heart rate is normal. Correct answer: J

| EXPLANATION |

Options for Questions 17-18

A Serial beta-HCG assays B Laparoscopy


C Repeat trans-vaginal scan in 24h D Repeat trans-vaginal scan in 7 days
E Reassure F Offer surgical evacuation of products of conception
G Offer medical treatment with methotrexate H Laparotomy
I Serum progesterone assay J Serum AFP assay
K Refer to molar pregnancy centre L Refer to other specialty

Instructions: Instructions: For each patient described below choose the single most appropriate initial
management option from the list. Each option may be used once, more than once, or not at all.

An asymptomatic 24 year old woman with a regular 28 day


cycle presents with 7 weeks amenorrhoea. Her pregnancy Your answer: A
test was positive 2 weeks earlier and she has been referred
Question 17
because of a history of previous ectopic pregnancy. Trans-
vaginal scan shows an empty uterus with no adnexal masses Correct answer: A
and no free fluid in the pouch of Douglas.

A 24 year old woman with one previous ectopic pregnancy


and a regular 28 day cycle presents with 7 weeks Your answer: A
amenorrhoea and a 6h history of constant abdominal pain.
Question 18
Her pregnancy test was positive 2 weeks earlier. Trans-
vaginal scan shows an empty uterus with no adnexal masses Correct answer: H
and some free fluid in the pouch of Douglas. She collapses
shortly after the ultrasound scan.

| EXPLANATION |

Options for Questions 19-20

A Counsel and offer evacuation of retained products B Counsel and offer support group
of conception

C Counsel and offer laparoscopy & salpingectomy D Counsel and offer investigation for recurrent
miscarriage

E Counsel and offer treatment with methotrexate F Counsel and offer laparotomy &
salpingectomy
G Counsel and offer termination of pregnancy H Perform salpingectomy
I Proceed to laparotomy J Serial HCG assay

K Counsel and refer to early pregnancy assessment L Repeat trans-vaginal scan 7 days later
unit

Instructions: For each patient described below choose the single most appropriate initial management
option from the list. Each option may be used once, more than once, or not at all.

A 25 year old woman has a transvaginal scan following IVF Your answer: A
treatment during which 2 embryos were replaced and is found
Question 19
to have a 6 week singleton intra-uterine pregnancy with no
fetal heart activity. Correct answer: A

A 25 year old nulliparous woman has had 2 previous first Your answer: A
trimester miscarriages presents at 8 weeks gestation for an
Question 20
early pregnancy scan. She is found to have a 7 week missed
miscarriage. Correct answer: A

| EXPLANATION |

Options for Questions 21-22

A Serial beta-HCG assay B Laparoscopy


C Repeat trans-vaginal scan in 24h D Repeat trans-vaginal scan in 7 days
E Counsel and discharge F Offer surgical evacuation of products of conception
G Offer medical treatment with methotrexate H Laparotomy
I Serum progesterone assay J Serum AFP assay
K Refer to molar pregnancy centre L Refer to other specialty

Instructions: Instructions: For each patient described below choose the single most appropriate initial
management option from the list. Each option may be used once, more than once, or not at all.

An asymptomatic 30 year old woman with a previous ectopic


pregnancy presents at 6 weeks amenorrhoea for trans-
vaginal scan to exclude a repeat ectopic pregnancy. The scan Your answer: A
Question 21 is reported to show a 14mm endometrium with no gestation
sac. There are no adnexal masses and no free fluid in the Correct answer: D
pouch of Douglas. HCG concentration is 500IU and 1600IU
48h later

An asymptomatic 30 year old woman with a previous ectopic Your answer: A


pregnancy presents at 6 weeks amenorrhoea for trans-
Question 22
vaginal scan to exclude a repeat ectopic pregnancy. Her
pregnancy test had been positive 2 weeks earlier. The scan is Correct answer: B
reported to show a 14mm endometrium with no gestation sac.
There is a 4cm cystic structure in the right adnexum
suggestive of an ectopic pregnancy but no free fluid in the
pouch of Douglas

| EXPLANATION |

Options for Questions 23-24

A Emergency caesarean section B Ultrasound scan for placental site


C Induction of labour with prostaglandins D Induction of labour by amniotomy
E Oxytocin augmentation of labour F Fetal scalp blood sampling
G Vaginal operative delivery H Maternal blood transfusion
I Transfer to high dependency unit J Expectant management
K Arrange antenatal clinic follow-up L Treatment with tocolytics
M Umbilical artery Doppler N Perform Kleihauer test

Instructions: For each of the case histories described below, choose the single most appropriate
management from the above list of options. Each option may be used once, more than once, or not at
all.

A 42 year old woman had an elective caesarean section at 37 Your answer: A


weeks gestation for major placenta previa. She returned to
Question 23
theatre 6h later because of suspected intra-abdominal
bleeding and has undergone a total abdominal hysterectomy Correct answer: I

A 35 year old Rhesus positive woman presented with fresh


vaginal bleeding and intermittent abdominal pain at 30 weeks Your answer: A
gestation. Maternal pulse on admission was 90bpm with BP
Question 24
120/80. The fetal heart rate was normal. She has been in
hospital for 48h and complains of a slight brown discharge but Correct answer: K
no other symptoms and has had no further bleeding.

| EXPLANATION |

Options for Questions 25-26

A Emergency caesarean section B Ultrasound scan for placental site


C Induction of labour with prostaglandins D Elective caesarean section at 37 weeks
E Oxytocin augmentation of labour F Fetal scalp blood sampling
G Induction of labour with oxytocin H Umbilical artery Doppler
I Elective caesarean section at 39 weeks J Arrange antenatal clinic follow-up
K Expectant management L Transfer to high dependency unit
M Maternal blood transfusion N Vaginal operative delivery

Instructions: For each of the case histories described below, choose the single most appropriate
management from the above list of options. Each option may be used once, more than once, or not at
all.

A 35 year old woman with major placenta previa has been in Your answer: A
hospital since 32 weeks gestation when she presented with a
Question 25
small antepartum haemorrhage. She has had no further
bleeding and is now 35 weeks pregnant Correct answer: D

Question 26 A 36 year old woman presents in spontaneous labour at 39 Your answer: A


weeks gestation and progresses rapidly to full dilatation with a
direct occipito-anterior position 1cm below the ischial spines. Correct answer: N
She complains of sudden onset constant abdominal pain and
fresh vaginal bleeding and the CTG shows deep
decelerations

Home | Signup | Subscribe | Contact us | Medical forums

You might also like